Patho Set 2

Pataasin ang iyong marka sa homework at exams ngayon gamit ang Quizwiz!

Which of the following interventions would be considered a nonpharmacologic method of pain control? Select all that apply.

-Distraction by knitting -Guided imagery -Biofeedback

Which of the following principles should underlie the pain control strategy in the care of a child with a diagnosis of cancer?

-Dosing and timing should aim for a steady serum level of the prescribed drug.

Which of the following pathophysiologic phenomena may result in a diagnosis of Cushing disease?

-Excess ACTH production by a pituitary tumor

Which of the following assessment findings of a male client constitutes a criterion for a diagnosis of metabolic syndrome? The client:

-Has blood pressure that is consistently in the range of 150/92 mm Hg

A high school senior sustained a concussion during a football game. Which of the following signs and symptoms are indicative of postconcussion syndrome in the weeks following his injury?

-Headaches and memory lapses

A late indicator of increased intracranial pressure is:

-High mean arterial pressure

While working on the med-surg floor, the nurse has a client who is experiencing an insulin reaction. The client is conscious and can follow directions. The most appropriate intervention would be:

-Immediately administer 15 g of glucose (preferably via oral route if the client is alert enough to swallow) and wait for 15 minutes. Then repeat this if necessary.

LDL 172 mg/dL

A nurse is reviewing the laboratory results of a male adult client who is at risk for peripheral arterial disease from atherosclerosis. The nurse should identify that which of the following results places the client at risk?

The client has a traumatic complete spinal cord transection at the C5 level. Based on this injury, the health care worker can expect the client to have control of which body function/part?

-Diaphragm

osteopenia

Older adults are at risk and benefit from vitamin D supplementation (necessary for calcium absorption) to prevent osteopenia; osteoporosis; pseudofractures; and electrolyte imbalances

osteoporosis

Older adults are at risk and benefit from vitamin D supplementation (necessary for calcium absorption) to prevent osteopenia; osteoporosis; pseudofractures; and electrolyte imbalances

pseudofractures

Older adults are at risk and benefit from vitamin D supplementation (necessary for calcium absorption) to prevent osteopenia; osteoporosis; pseudofractures; and electrolyte imbalances

Myelinated neurons found in the central nervous system are covered by which of the following cells?

Oligodendrocytes

Bone formation occurs in two stages. Which of the following stages involves the formation of prebone or unmineralized bone?

Ossification

An adult female client visits with her health care provider about pain in her hand. She describes it as an audible grinding and cracking sound, especially in her thumb. "I had to buy an automatic jar opener...I just can't grasp and open a jar...it just hurts too badly." The health care provider suspects the client has a degenerative form of joint disease that is often evidenced by:

Osteoarthritis

An elderly female is admitted with a hip fracture. Her lab test reveals an increase in serum levels of alkaline phosphatase. The nurse determines which of the following is causing this lab level to rise?

Osteoblast activity

Multiple sclerosis is characterized by what type of neuron damage?

Polyneuropathy

axial skeleton

Portion of the skeletal system that consists of the skull, rib cage, and vertebral column

Each joint capsule has tendons and ligaments. What are the tendons and ligaments of the joint capsule sensitive to?

Position and movement

The nurse is performing an assessment for a client suspected of having symptoms that correlate with carpal tunnel disorder. The client states he feels a tingling sensation that radiates into the palm of the hand when the nurse lightly percusses over the median nerve. How would the nurse document this finding?

Positive Tinel sign

Duchenne muscular dystrophy usually does not produce any signs or symptoms until between the ages of 2 and 3. What muscles are usually first to be affected in Duchenne muscular dystrophy?

Postural muscles of the hip and shoulder

The more complex patterns of movements, such as throwing a balling or picking up a fork, are controlled by the _____ motor cortex in the frontal lobe.

Premotor

A young adult male client presents to the orthopedic clinic complaining of "stiffening of the spine." The health care provider orders some diagnostic lab work. Which lab result leads the health care worker to diagnose ankylosing spondylitis?

Presence of HLA-B27 allele marker

A 70 year-old male has been diagnosed with a stroke that resulted in an infarct to his cerebellum. Which of the following clinical findings would be most closely associated with cerebellar insult?

Unsteady gait and difficulty speaking and swallowing

A patient is devastated to receive a diagnosis of amyotrophic lateral sclerosis (ALS). The symptomatology of this disease is a result of its effects on:

UPPER and LOWER motor neurons

A nurse is caring for a patient who has developed vasculitis as a result of a rheumatology disorder. Which of the following factors does the nurse expect to uncover in assessing this patient?

Ulcers of the lower extremities

When teaching a patient recently diagnosed with systemic lupus erythematosus (SLE), which of the following does the nurse teach the client to avoid to prevent exacerbations?

Ultraviolet light (UV)

A late indicator of increased intracranial pressure is:

Unequal pupil response

Which signs and symptoms is most suggestive of acute cholecystitis

Upper right quadrant or epigastric pain

When a nurse is caring for a patient with gout, which of the following diagnostic studies supporting the presence of the disease does the nurse monitor?

Uric acid levels

order of complexity for nervous system that control motor function (cord, brain stem, motor cortex

cord (least- reflexes) brainstem (bridge) motor cortex (most complex...responsible for voluntary muscle activity

Decreased GFR leads to increased plasma ___ and ___

creatinine urea (BUN)

dorsal column (medial lemniscal pathway)

crosses at base of medulla

Which test provides a way to monitor fluctuations of blood glucose levels over the previous 6 to 12 weeks? a. glucose tolerance test b. fasting blood glucose c. capillary blood glucose d. glycosylated hemoglobin

d. Glycosylated hemoglobin

A long bone, such as the humerus of the upper arm, has which of the following structural characteristics? A) A perichondrium that overlies most of the bone surface B) A durable outer shell made of cancellous bone C) A diaphysis at each end D) An endosteum composed of osteogenic cells

d. an endosteum composed of osteogenic cells

Type 1 diabetes melitus results from destruction of the pancreatic beta cells by two mechanisms. The more common mechanism for Type I diabetes is _____ destruction of the beta cells. a. genetic b. resistant c. idiopathic d. autoimmune

d. autoimmune

The major adrenal cortical hormones are steroids and are synthesized from acetate and a. ACTH b. albumin c. amino acids d. cholesterol

d. cholesterol

A change in the circadian rhythm for glucose tolerance and an inappropriate increase in counterregulatory hormones can lead to ____ in diabetics a. hypoglycemia b. Somogyi effect c. hyperinsulinemia d. dawn phenomenon

d. dawn phenomenon

Persons with non-insulin dependent diabetes (Type II) are said to be ketosis-resistant because they a. are unable to convert fatty acids to ketone b. have insufficient fat stores c. have sufficient insulin to permit glucose transport into fat cells d. have sufficient insulin to prevent fat breakdown

d. have sufficient insulin to prevent fat breakdown

Extreme cerebral edema may cause the brain to herniate into another compartment. Upward herniation from the infratentorial compartment against the aqueduct of Sylvius causes:

hydrocephalus

A characteristic manifestation of polycystic kidney disease is:

hypertension

cluster (headache)

hypothalamus is believed to play a key role in this pathology

cluster headache

hypothalamus is believed to play a key role in this pathology

A clinician is assessing the muscle tone of a patient who has been diagnosed with a lower motor neuron (LMN) lesion. Which of the following assessment findings is congruent with the patients diagnosis?

hypotonia

discriminative touch

identify size/shape of object moving across skin surface

TMJ (temporomandibular joint pain)

imbalance in joint movement because of poor bite, bruxism (teeth grinding), inflammation, trauma or degenerative changes

how to osteoblasts build bone matrix

in alkaline env.

The most recent assessment of a patient with a diagnosis of type 1 diabetes indicates a heightened risk of diabetic nephropathy. Which of the following assessment findings is most suggestive of this increased risk?

microalbuminuria

Carpal tunnel syndrome is a __ neuropathy between the ___ neuron

mono median

migraine headache

more common in women, may have a hereditary linkage

Disorders of the pyramidal tracts, such as a stroke, are characterized by: paralysis. hypotonia. muscle rigidity. involuntary movements.

paralysis.

sacral ___ bladder

parasympathetic

if there is a spinal sacral problem, __ is damaged. You will have an ____ situation and leads to ___ bladder

parasympathetic overflow areflex ( can't pee)

substantia nigra- not enough dopamine associated with __

parkinsons

sesamoid bone

patella

The most common cause of knee pain is:

patellofemoral pain syndrome

The Helicobacter pylori protobacteria cause __ __ by producing __

peptic ulceration toxins

some cartilage lacks

perichorndrium such as articular surfaces and moving joints

Coup and contrecoup cerebral contusion caused by blunt head trauma against a fixed object results in:

permanent brain tissue damage

chronic

persistent pain that may be accompanied by loss of appetite, depression, sleep disturbances & other debilitating responses

Common manifestations of acute meningococcal meningitis, a highly contagious and lethal form of meningitis, include:

petechiae

A common manifestation of acute meningococcal meningitis, a highly contagious and lethal form of meningitis, is

petechiae (May be an early sign of DIC)

The kidney assists in blood pressure regulation by releasing_______ from the juxtaglomerular cells in response to decreased renal blood flow, resulting in efferent arteriole vasoconstriction.

renin

arthroplasty

repair of a joint

A patient with a long history of cigarette smoking and poorly controlled hypertension has experienced recent psychomotor deficits as a result of hemorrhagic brain damage. The patients psychomotor deficits are likely the result of:

vascular dementia

Autonomic hyperreflexia is characterized by

vasospams and hypertension

Autonomic dysreflexia (autonomic hyperreflexia) is characterized by ___ and ___

vasospasms hypertension

Autonomic dysreflexia (autonomic hyperreflexia) is characterized by:

vasospasms and hypertension

dorsal column (medial lemniscal pathway)

uses three neurons to transmit information: 1. primary dorsal root ganglion neuron → dorsal column neuron 2.the medial lemniscus → thalamic neuron

Autonomic dysreflexia (autonomic hyperreflexia) is characterized by: severe spinal shock. tachycardia and pale skin. lack of sweat above injury level. vasospasms and hypertension.

vasospasms and hypertension.

BOTH prehepatic and posthepatic causes of portal hypertension include the formation of a. fibrous nodules b. venous thrombosis c. collateral circulation d. portosystemic shunts

venous thrombosis

planar joints

veterbrocostal joints

Pacinian corpuscles

within hypodermis

general visceral (afferent neurons)

within visceral structures

Unlike disorders of the motor cortex and corticospinal (pyramidal) tract, lesions of the basal ganglia disrupt movement ___ ___ ___

without causing paralysis

Unlike disorders of the motor cortex and corticospinal (pyramidal) tract, lesions of the basal ganglia disrupt movement:

without causing paralysis

Following a length series of diagnostic tests, a patient's chronic hip pain has been attributed to advanced osteonecrosis. What treatment is this patient most likely to require? A. Joint replacement surgery B. Intravenous antibiotics C. Injections of corticosteroids into the synovial space. D. transfusion of packed red blood cells.

A

In contrast to synarthroses joints, synovial joints are linked to the bone by: A. A joint capsule B. hyaline cartilage C. connective tissue D. interosseous ligaments

A

In growing bone, severe vitamin C deficiency slows bone growth by impairing: A. organic matrix formation B. calcification of new bone C. growth plate separation D. widening of the cortex

A

The distinguishing characteristic of chronic osteomyelitis is the presence of: A. Sequestrum bone. B. Abscess formation C. Severe bone pain D. External drainage

A

Which of the following joints are classified as a synarthrosis? A. the joint between 2 vertebrae B. the joint between the femur and the pelvis. C. the joint between the humerus & the radius & ulna. D. an interphalangeal joint of the hand (knuckle)

A

Which individual is likely to have the best prognosis for recovery from his or her insult to the peripheral nervous system?

A 32-year-old male who had his forearm partially crushed by gears during an industrial accident.

Which of the following individuals has the highest chance of having a medulloblastoma?

A 4-year-old child who has become uncoordinated in recent months

Which of the following individuals demonstrates a health problem with his or her axial skeleton?

A 40 year-old man who has a contusion to the left temporal bone of his skull following a motor vehicle accident.

Which of the following clinical findings among older adults is most unlikely to warrant further investigation and possible treatment?

A 78-year-old female's GFR has been steadily declining over several years.

The health care provider has determined that a client has developed rheumatoid arthritis. The provider is aware that the client may experience joint inflammation that involves of immunologic mediation triggered by:

A T-cell mediated response to an immunologic trigger, such as a microbial agent

periosteum

A dense fibrous membrane covering the surface of bones (except at their extremities) and serving as an attachment for tendons and muscles.

An ultrasound has confirmed appendicitis as the cause of a 20-year-old man's sudden abdominal pain. Which of the following etiologic processes is implicated in the development of appendicitis? Obstruction of the intestinal lumen Elimination of normal intestinal flora Sloughing of the intestinal mucosa Increased osmolality of intestinal contents

A due to fecal mass

What does the nurse expect to assess in a client that has a torn ligament?

A high degree of pain

located on left pedal pulse

A nurse is assessing a client who had left femoral cardiac angiography. Identify where the nurse will palpate to assess the most distal pulse on the affected side. (Selectable areas, or "Hot Spots," are outlined in the artwork below. Select only the outlined area that corresponds to your answer.)

Hyperpigmentation

A nurse is assessing a client who has Cushing's syndrome. Which of the following findings should the nurse expect?

Muscle spasms

A nurse is assessing a client who has a hip fracture. Which of the following findings should the nurse expect?

Increased BUN

A nurse is assessing a client who has fluid volume deficit. The nurse should expect which of the following findings?

Client report of numbness in his hands

A nurse is assessing a client who has hypoparathyroidism. Which of the following findings should the nurse expect?

Weight gain

A nurse is assessing a client who has hypothyroidism. The nurse should expect which of the following findings?

Dyspnea

A nurse is assessing a client who has infective endocarditis. Which of the following findings should be the priority for the nurse to report to the provider?

airway obstruction

A nurse is caring for a client admitted to the emergency department with extensive partial and full-thickness burns of the head, neck, and chest. While planning the client's care, the nurse should be aware that initially the client is at greatest risk for

Elevate the head of the bed to 30&deg

A nurse is caring for a client following surgical treatment for a supratentorial brain tumor. Which of the following interventions should the nurse take?

Weigh the client daily.

A nurse is caring for a client who has Addison's disease and is at risk for Addisonian crisis. Which of the following actions should the nurse take?

-Moon face -Purple striations -Buffalo hump

A nurse is caring for a client who has Cushing's syndrome. The nurse should recognize that which of the following are manifestations of Cushing's syndrome? (Select all that apply.)

Keep the drainage system below the level of the client's chest at all times

A nurse is caring for a client who has a chest tube connected to a closed drainage system and needs to be transported to the x-ray department. Which of the following actions should the nurse take?

Keep the drainage system below the level of the client's chest at all times.

A nurse is caring for a client who has a chest tube connected to a closed drainage system and needs to be transported to the x-ray department. Which of the following actions should the nurse take?

Instruct the client to cough and deep breathe.

A nurse is caring for a client who has increased intracranial pressure (ICP) following a closed-head injury. Which of the following actions should the nurse take?

Dyspnea

A nurse is caring for a client who has infective endocarditis. Which of the following manifestations is the priority for the nurse to monitor for?

Place the client in a high-Fowler's position

A nurse is caring for a client who has quadriplegia from a spinal cord injury and reports having a severe headache. The nurse obtains a blood pressure reading of 210/108 mm Hg and suspects the client is experiencing autonomic dysreflexia. Which of the following actions should the nurse take first?

Ensure the airway is open

A nurse is caring for a client who has respiratory failure, is intubated, and is mechanically ventilated. The client pulls out the endotracheal tube. Which of the following is a priority nursing action?

Decreased level of consciousness

A nurse is caring for a client who has sustained a traumatic brain injury. The nurse should monitor the client for which of the following manifestations of increased intracranial pressure?

Verify that the suction regulator is on and check the tubing for leaks.

A nurse is caring for a client who is 1-day postoperative following a left lower lobectomy of the lung. When assessing the client's closed chest drainage system, the nurse notes that there is no bubbling in the suction control chamber. Which of the following actions should the nurse take?

Combing her hair

A nurse is caring for a client who is 4 days postoperative following a right radical mastectomy. Which of the following activities should the nurse anticipate being the most difficult for this client to perform with her right hand?

Reduce edema of the brain.

A nurse is preparing to administer an osmotic diuretic IV to a client with increased intracranial pressure. Which of the following should the nurse identify as the purpose of the medication?

Heart rate 56/mi

A nurse is preparing to administer atenolol (Tenormin) to a client. Which of the following should prompt the nurse to withhold the medication?

Ten years

A nurse is teaching a group of middle adult clients about early detection of colorectal cancer. The nurse should include the American Cancer Society recommendation that men and women beginning at age 50 who are at average risk should have a fecal occult blood test (FOBT) and a colonoscopy at which of the following intervals?

"Disaster drills should be held on a regular basis."

A nurse is teaching a group of newly hired nurses about the requirements for disaster planning. Which of the following statements by one of the newly hired nurses indicates an understanding of the teaching?

"I will give my son the enzymes between meals."

A nurse is teaching the mother of a 5-year-old child who has cystic fibrosis about pancreatic enzymes. The nurse should understand that further teaching is necessary when the mother states which of the following?

Plantar flexion of the legs

An acute care nurse receives shift report for a client who has increased intracranial pressure. The nurse is told that the client demonstrates decorticate posturing. Which of the following findings should the nurse expect to observe when assessing the client?

Allow the drainage to drip onto a sterile gauze pad

An emergency room nurse is caring for a client following an automobile crash. Upon assessment the nurse observes bleeding from the client's nose. Which of the following interventions is appropriate?

A long bone, such as the humerus of the upper arm, has which of the following structural characteristics?

An endosteum composed of osteogenic cells

Which one of the following is a manifestation of a serious condition that causes new-onset back pain in persons 50 years of age or older?

Aortic aneurysm

For many patients, the first indication that they have osteoporosis is: A. Bone pain that is not alleviated by rest B. A bone fracture C. Craving high-calcium foods D. Decrease in range of motion in the hip and knee joints

B

For many patients, the first indication that they have osteoporosis is: A) bone pain that is not alleviated by rest. B) a bone fracture. C) craving high-calcium foods. D) decreased range of motion in the hip and knee joints.

B) a bone fracture.

Whereas PTH increases blood calcium levels, the hormone _____ lowers blood calcium levels and decreases bone resorption. A) vitamin D B) calcitonin C) prolactin D) phosphate

B) calcitonin

Rheumatoid arthritis is a systemic inflammatory disease with joint manifestations described as: A) dysplasia. B) polyarticular. C) asymmetrical. D) osteophytes.

B) polyarticular.

To maintain adequate serum calcium levels, parathyroid hormone reduces: A) activation of vitamin D. B) serum phosphate levels. C) calcium release from bone. D) intestinal absorption of calcium.

B) serum phosphate levels.

A pathologic stress fracture occurs in bones subjected to: A) sudden direct force. B) weakening by disease. C) repeated excessive use. D) massive muscle contraction.

B) weakening by disease.

What are the closed sacs that are formed from the synovial membrane but are not part of the joint called?

Bursae

A patient is devastated to receive a diagnosis of amyotrophic lateral sclerosis (ALS). The symptomatology of this disease is a result of its effects on: A. Upper motor neurons. B. The vestibulocerebellar system C. Upper & lower motor neurons. D. Neuromuscular junctions

C

A vitamin _____ deficiency impairs the formation of the organic matrix, slowing growth at the epiphyseal plate.

C

The tendons and ligaments of the joint capsule are sensitive to position and movement as a result of having _______ to help maintain muscle support. A) autonomic nerve fibers B) supporting bursa sacs C) reflexive proprioception D) elastic articular cartilage

C) reflexive proprioception

Loss of diaphragm control would most likely occur with a spinal injury at which level?

C1-2

A patient with a history of a seizure disorder has been observed suddenly and repetitively patting his knee. After stopping this repetitive action, the patient appears confused and is oriented to person and place but not time. What type of seizure did this patient most likely experience?

Complex partial seizure

A client is concerned about bone strength and development and asks the nurse if dietary sources of vitamin D will cause immediate activity within the skeletal system. The best response by the nurse would be:

"Vitamin D has little or no activity until it has been converted to physiologically active compounds."

A 70 year-old woman's family physician has recommended a vitamin D supplement. The woman states that she tries hard to take as few pills as possible and questions her physician on the rationale and necessity of the hormone. How can the physician most accurately reply to the client's concerns?

"Vitamin D is important in order for your body to absorb the calcium that you consume in your diet."

The nurse determines that additional patient education is needed when a patient with gout makes which one of the following statements?

"When I have an exacerbation of my symptoms, a glass of red wine will be helpful."

A hospital client has been complaining of increasing fatigue for several hours, and his nurse has entered his room to find him unarousable. The nurse immediately checked the client's blood glucose level (and reverified with a second blood glucose meter), which is 22 mg/dL (1.2 mmol/L). The nurse should prepare to administer which of the following?

-A 50% glucose solution intravenously

The nurse contacts the physician regarding a client's early signs of diminishing level of consciousness based on which of the following? Select all that apply.

• Disorientation • Blunted responsiveness • Inattention

Which are efficient dietary ways to aid in intestinal absorption of calcium? Select all that apply.

• Eating fish on a regular basis • Drinking irradiated milk

While talking about their migraine headaches, two women have found that they have some common triggers for their migraines, which may include: Select all that apply.

-At the time of their menstrual cycle -Consuming chocolate

A family brings their elderly mom to the emergency department. The client had a short period of time where she was confused, had slurred speech and appeared to have a weak arm. Now she is back to her normal self. Suspecting a transient ischemic attacks (TIAs), the health care provider will order diagnostic testing looking for:

-Atherosclerotic lesions in cerebral vessels

A client with a history of diabetes presents to the emergency department following several days of polyuria and polydipsia with nausea/vomiting. On admission, the client labs show a blood glucose level of 480 mg/dL and bicarbonate level of 7.8 mEq/dL. The nurse suspects the client has diabetic ketoacidosis (DKA). The priority intervention should include:

-Begin a loading dose of IV regular insulin followed by a continuous insulin infusion.

A client has suffered a stroke that has affected his speech. The physician has identified the client as having expressive aphasia. Later in the day, the family asks the nurse to explain what this means. The most accurate response would be aphasia that is:

-Characterized by an inability to easily communicate spontaneously or translate thoughts or ideas into meaningful speech or writing

Impaired and delayed healing in a person with diabetes is caused by long-term complications that include:

-Chronic neuropathies

A client tells his health care provider that his body is changing. It used to be normal for his blood glucose to be higher during the latter part of the morning. However, now his fasting blood glucose level is elevated in the early AM (07:00). The health care provider recognizes the client may be experiencing:

-Dawn phenomenon

A diabetic client's most recent blood work indicated a decreased glomerular filtration rate and urine testing revealed + microalbuminuria. Which priority self-care measures should the client's care team prescribe for this client?

-Diet, exercise, and prescriptions to lower blood pressure below 140/80 mm Hg

Complex regional pain syndrome is characterized by:

-Disproportionate pain with mobility

Stretch-sensitive receptors in the skin (Ruffini end organs, Pacinian corpuscles, and Merkel cells) help signal postural information and are processed through the:

-Dorsal column-medial lemniscus pathway

Which of the following clinical manifestations would support the medical diagnosis of Cushing syndrome? Select all that apply.

-Excessive facial hair growth -Blood glucose level in 200 mg/dL range -Blood pressure reading less than 90/70

The health care provider for a client with degenerative Alzheimer-type dementia recognizes the client has moved from the initial stage to the moderate stage based on which of the following clinical manifestations? Select all that apply.

-Extreme confusion and disorientation -Need for direct supervision for ADLs -Inability to problem solve simple tasks

A sudden traumatic complete transection of the spinal cord results in what type of injury below the site?

-Flaccid paralysis

While explaining the somatosensory cortex to a group of nursing students, the instructor asks, "What is involved in the final processing of somatosensory information?" The correct response includes: Select all that apply.

-Full localization -Discrimination of intensity -Interpretation of somatosensory stimuli

Loss of pituitary function can result in deficiencies/loss of which of the following hormones' secretions? Select all that apply.

-Growth hormone -Luteinizing hormone -Follicle stimulating hormone -Prolactin

Diabetic retinopathy, the leading cause of acquired blindness in the United States, is characterized by retinal:

-Hemorrhages

perichrondrium

-In hyaline cartilage -Dense irregular connective tissue -Covers cartilage and helps maintain its shape

While trying to explain the physiology behind type 2 diabetes to a group of nursing students, the instructor will mention which of the following accurate information?

-In skeletal muscle, insulin resistance prompts decreased uptake of glucose. Following meals (postprandial), glucose levels are higher due to diminished efficiency of glucose clearance.

Following a head injury on the football field, the medical team is assessing the player for injury. One of the earliest signs of decreased level of consciousness to assess for would be:

-Inattention

While lecturing to a group of physiology students, the instructor asks, "What metabolic factors cause vasodilation of cerebral vessels thereby increasing cerebral blood flow to the brain?" The student with the best response would be:

-Increased carbon dioxide level

Which of the following pathophysiologic processes occurs in cases of bacterial meningitis?

-Inflammation allows pathogens to cross into the cerebrospinal fluid.

Which of the following insulin administration regimens is most likely to result in stable blood glucose levels for a client with a diagnosis of type 1 diabetes?

-Intermediate-acting insulin at 8:00 AM and 8:00 PM with rapid-acting insulin before each meal

Following a head injury, a client is diagnosed with a possible intracranial epidural hematoma. During the initial assessment, the client suddenly becomes unconscious. Other clinical manifestations that correlate with this diagnosis may include:

-Ipsilateral pupil dilation

Which of the following pregnant women likely faces the greatest risk of developing gestational diabetes? A client who:

-Is morbidly obese defined as greater than 100 pounds over ideal weight

A client's emergency magnetic resonance imaging (MRI) has been examined by the physician and tissue plasminogen activator (tPA) has been administered to the client. What was this client's most likely diagnosis?

-Ischemic stroke

The iatrogenic form of Cushing syndrome is caused by:

-Long-term cortisone therapy

Nociceptors are sensory receptors that are activated by:

-Noxious stimuli

The most common cause of thyrotoxicosis is Graves disease. When assessing this client, the nurse should put priority on which of the following signs/symptoms?

-Ophthalmopathy

What pain theory proposes that pain receptors share pathways with other sensory modalities and that different activity patterns of the same neurons can be used to signal painful or non painful stimuli?

-Pattern

Opioid receptors are highly concentrated in which region of the CNS and produce pain relief through the release of endogenous opioids?

-Periaqueductal gray (PAG) region

Common manifestations of acute meningococcal meningitis, a highly contagious and lethal form of meningitis, include:

-Petechiae

A client's recent computed tomography (CT) scan has revealed the presence of hydrocephalus. Which of the following treatment measures is most likely to resolve this health problem?

-Placement of a shunt

A hospital client with a diagnosis of type 1 diabetes has been administered a scheduled dose of regular insulin. Which of the following effects will result from the action of insulin?

-Promotion of glucose uptake by target cells

A client has just undergone a diagnostic cardiac angiogram. As part of their ordered labs, the physician has ordered a thyroid panel. The physiological principle behind ordering this lab tests includes which of the following correlations? Hyperthyroidism can cause: Select all that apply.

-Rise in oxygen consumption -Increase in cardiac output

When a person is stung on the index finger by a bee, the thalamus interprets the pain as:

-Somewhere on the hand

A client who has been taking 80 mg of prednisone, a glucocorticoid, each day has been warned by his primary care provider to carefully follow a plan for the gradual reduction of the dose rather than stopping the drug suddenly. What is the rationale for this directive?

-Stopping the drug suddenly may cause acute adrenal insufficiency.

When lecturing about heart attacks (myocardial infarctions), the instructor will emphasize the client may present with: Select all that apply.

-Substernal chest pain -Neck pain -Pain that radiates to the left arm

Following an automobile accident that resulted in a traumatic amputation of the right lower leg, the client complains of feeling tingling, heaviness, and shooting pain in the amputated limb. The health care providers treat phantom limb pain by which of the following interventions? Select all that apply.

-TENS of the large myelinated afferents innervating the area -Hypnosis -Relaxation techniques

Which of the following clinical manifestations following thyroidectomy would alert the nurse that the client is going into a life-threatening thyroid storm? Select all that apply.

-Temperature of 104.2°F -Telemetry showing heart rate of 184 -Extremely agitated

While playing outside in the snow, a young child complained of painful fingertips since he would not keep his gloves on. In the emergency department, the nurse knows this painful sensation is a result of which transmission of proprioceptive somatosensory information?

-Type C dorsal root ganglion neurons

Which of the following characteristics differentiates a migraine with aura from a migraine without aura?

-Visual symptoms that precede the headache

A client has been diagnosed with a cerebral aneurysm and placed under close observation before treatment commences. Which of the following pathophysiologic conditions has contributed to this client's diagnosis?

-Weakness in the muscular wall of an artery

A student nurse is taking a test on the endocrine system. From the following list of clinical manifestations, she needs to select the ones she would see in hypothyroidism. Which answers should she select? Select all that apply.

-Weight gain despite loss of appetite -Coarse brittle hair -Puffy face with swollen eyelids

Match each of the descriptive statements with the appropriate categories of lung disease. Cystic fibrosis is an example A. Obstructive lung Dz Asthma, emphysema, or B. Restrictive Lung Dz chronic bronchitis C. Both May be due to neurological D. Neither or neuromuscular defects

1.Cystic fibrosis is an example of Obstructive Lung Dz 2.Asthma, emphysema, or chronic bronchitis are obstructive lung Dz 3. Restrictive lung Dz may be due to neurological or neuromuscular defects

When teaching a community education class on osteoarthritis (OA), the nurse should include which of the following in discussing which individuals are at risk for this problem? Select all that apply. a. Asian women b. Elderly individuals c. Older women d. Obese people e. People with autoimmune disorders

Correct response: b. Elderly individuals c. Older women d. Obese people Explanation: Eighty-five percent of people with OA are in their seventies. Men are affected more commonly at a younger age, however, by 55 years of age; women are the more frequent gender affected. Obesity is a risk; Chinese individuals have a lower incidence of OA than Europeans.

When caring for a patient with ankylosing spondylitis, the nurse tells the patient that stiffness may be relieved by which of the following interventions? a. Bed rest b. Gentle exercise c. Calcium supplements d. Acetaminophen

Correct response: b. Gentle exercise Explanation: The pain of ankylosing spondylitis becomes worse when resting, particularly when lying in bed, and may be relieved with physical activity. Lumbosacral pain may also be present, with discomfort in the buttocks and hip areas. Prolonged stiffness is present in the morning and after periods of rest. Mild physical activity or a hot shower helps reduce pain and stiffness.

A 55-year-old male client has reported joint pain in his feet. Which of the following blood work results should prompt further testing to rule out primary gout? a. Increased C-reactive protein (CRP) b. Increased serum uric acid c. Increased polymorphonuclear leukocytes d. Increased serum cortisol

Correct response: b. Increased serum uric acid Explanation: Although hyperuricemia is not diagnostic of gout, it is suggestive and should prompt further assessment. Increases in CRP, polymorphonuclear leukocytes, and cortisol levels are not as closely associated with the body's response to gout.

A basketball player fell awkwardly when attempting to claim a rebound, a mishap that resulted in a tear to the anterior cruciate ligament of his left knee. What characteristics of ligaments make them particularly susceptible to injury? A. ligaments are incapable of accommodating lateral movement. B. ligaments are exclusive to the knee, which is the most frequently injured joint. C. ligaments are superficial to the synovial joint capsule & are consequently vulnerable to impacts. D. ligaments are incapable of stretching when exposed to unusual stress.

D

A long bone, such as the humerus of the upper arm, has which of the following structural characteristics? A. a perichondrium that overlies most of the bone surface. B. a durable outer shell made of cancellous bone. C. a diaphysis at each end. D. an endosteum composed of osteogenic cells.

D

A patient's clacicular fracture has healed in the weeks following a bicycle accident. Which of the following events tales place in the remodeling stage of bone healing? A. Formation of granulation tissue B. Development of fibrocartilage that resembles the appearance of the original bone. C. Deposition of mineral salts into callus. D. Reduction in the size of the callus.

D

Autonomic dysreflexia (autonomic hyperreflexia) is characterized by: A. Severe spinal shock. B. Tachycardia & pale skin. C. Lack of sweat above injury level. D. Vasospasms & hypertension.

D

Cancellous bone is relatively light, it its lattice-like structure gives it: A. Rigidity B. Thickness C. A growth plate D. Tensile strength

D

Characteristics of a patient that is at high risk for falling & fracturing a hip include: A. Obesity & short stature B. Excessive sugar intake C. Hearing impairment D. Chronic osteoporosis

D

Osteonecrosis is most often cause by: A. Stress fractures B. Bacterial infection C. Synovial inflammation D. Bone marrow ischemia

D

Which of the following neoplasms of the skeletal system is likely to require the most timely & aggressive treatment? A. Esostosis B. Osteochondroma C. Endochondroma D. Osteosarcoma

D

Antibody testing has confirmed that a man is positive for hepatitis A virus (HAV). Which of the patient's statements suggests that he understands his new diagnosis? A) I guess I'm an example of why you should always use condoms." B) I'm embarrassed that I'll be a carrier of hepatitis from now on." C) I'm still trying to deal with the fact that this will forever change my life. D) I don't know why I didn't bother to get vaccinated against this

D (I don't know why I didn't bother to get vaccinated against this)

Which of the following neoplasms of the skeletal system is likely to require the most timely and aggressive treatment? A) Exostosis B) Osteochondroma C) Endochondroma D) Osteosarcoma

D) Osteosarcoma

Magnetic resonance imaging of a patients knee has revealed the presence of bursitis. What is the primary purpose of bursae? A) To maintain close articulation between the two long bones at a synovial joint B) To strengthen the attachment between skeletal muscles and bones C) To strengthen the integrity of the articulating capsule D) To prevent friction at a tendon in a synovial joint

D) To prevent friction at a tendon in a synovial joint

HLA-B27 antigen may be linked to other genes that determine the pathologic autoimmune phenomenon in: A) gout syndrome. B) rheumatoid arthritis. C) osteoarthritis syndrome. D) ankylosing spondylitis.

D) ankylosing spondylitis.

Osteonecrosis is most often caused by: A) stress fractures. B) bacterial infection. C) synovial inflammation. D) bone marrow ischemia.

D) bone marrow ischemia.

Metastatic bone disease is most closely associated with: A) cervical cancer and ovarian cancer. B) acute myelogenous leukemia (AML) and malignant melanoma. C) non-Hodgkin lymphoma and bladder cancer. D) breast cancer and prostate cancer.

D) breast cancer and prostate cancer.

Characteristics of a patient that is at high risk for falling and fracturing a hip include: A) obesity and short stature. B) excessive sugar intake. C) hearing impairment. D) chronic osteoporosis.

D) chronic osteoporosis.

A dermatologist is explaining to a client the advantages of using a lotion that can draw out water to the skin surface. The nurse knows the technical term for this lotion is a/an:

D. Occlusive

vit D in liver

25-hydroxylase (p-450) converts precursor Vit D to 25-hydroxyvitamin D (25-OH D; calcidiol)

The nurse is caring for a 31-year-old trauma victim admitted to the neurologic intensive care unit. While doing the initial assessment, the nurse finds that the client is flexing the arms, wrists, and fingers. There is adduction of the upper extremities with internal rotation and plantar flexion of the lower extremities. How would the nurse describe this in the notes?

Decorticate posturing Decorticate (flexion) posturing is characterized by flexion of the arms, wrists, and fingers, with adduction of the upper extremities, internal rotation, and plantar flexion of the lower extremities.

A feature of rheumatoid arthritis that differentiates it from other forms of inflammatory arthritis is the development of: A. pannus tissue. B. tophi deposits. C. subluxations D. autoantibodies.

A

A patient has a fractured tibia. After the cast is applied, he is at high risk for compartment syndrome caused by: A. Inflammation B. Joint immobility C. Muscle atrophy D. Extremity elevation

A

A patient presents to the clinic complaining, "I have something in my eye." When questioned, the patient admits to a scratching and burning sensation and light sensitivity. The health care provider suspects the patient has developed: A. Conjunctivitis B. Retinal detachment C. Acute glaucoma D. Corneal edema

A

A patient with a spinal cord injury at T8 would likely retain normal motor & somatosensory function to her: A. Arms. B. Bowels. C. Bladder. D. Perineal musculature.

A

A patient's recent diagnosis of Parkinson disease has prompted his care provider to promptly begin pharmacologic therapy. The drugs that are selected will likely influence the patient's levels of: A. Dopamine. B. Acetylcholine. C. Serotonin. D. Adenosine.

A

A sudden complete transection of the spinal cord results in _____ below the level of injury. A. Flaccid paralysis B. Vasoconstriction C. Deep visceral pain D. 3+ tendon reflexes

A

Despite differences in onset, involvement. & symptomatology, al of the spondyloarthropathies involve: A. sacroiliitis. B. calcinosis. C. excessive bone turnover. D. autoimmune etiology.

A

Developmental dysplasia of the hip (DDH), formerly known as congenital hip dislocation, is suspected when an infant has: A. Gluteal fold asymmetry B. Lengthening of the thigh C. Joint capsule tightness D. Delay of knee crawling

A

Disorder that affect cortical bone typically result in: A. fractures of long bones. B. impaired collagen synthesis. C. infection. D. vertebral fractures.

A

Disorders of the pyramidal tracts, such as a stroke, are characterized by: A. Paralysis B. Hypotonia C. Muscle rigidity D. Involuntary movements

A

Select the statement that best describes a synovial or diarthrodial joint. The joint:

Moves freely

flat bones

Flat bones: composed of a layer of cancellous bone between two layers of compact bone ~ found in areas such as the skull and rib cage, where extensive protection of underlying structure is needed ~ or ~ in the scapula, where a broad surface for muscle attachment must be provided

A client who is diagnosed with seizures describes feeling confused after experiencing a seizure. The family members report that the client has been smacking his lips prior to having a seizure. The client most likely experienced which type of seizure?

Focal Focal seizures with impairment of consciousness sometimes referred to as psychomotor seizures are often accompanied by automatisms or repetitive nonpurposeful activities such as lip smacking, grimacing, patting, or rubbing clothing. Confusion during the postictal period (after a seizure) is common.

can be inferred since the client is in critical condition

Following a suicide bombing at a shopping mall, an unidentified, unconscious client is admitted to the emergency department with an acute intra-abdominal hemorrhage. The nurse should recognize that consent for the surgery

In which of the following does the highest level of motor function occur?

Frontal cortex of the brain

thyroidcalcitonin

In contrast to PTH (which increases blood calcium levels), calcitonin (thyroidcalcitonin) is a thyroid-derived hormone that decreases blood calcium levels

metaphysis

In growing bones, the part of the bone shaft that funnels out as it approaches the epiphysis is called metaphysis ~ composed of bony trabeculae that have cartilage cores

The practitioner expects to see which of the following lab results in a client with a recent femur fracture?

Increased alkaline phosphatase

A patient's emergency magnetic resonance imaging (MRI) has been examined by the physician and tPA has been administered to the patient. What was this patient's most likely diagnosis?

Ischemic stroke

A patients emergency magnetic resonance imaging (MRI) has been examined by the physician and tPA has been administered to the patient. What was this patients most likely diagnosis?

Ischemic stroke

A client has a tendonitis. What will the nurse teach the client about this disorder?

It is caused by overuse.

Which statement is correct about hyaline cartilage?

It joins the ribs to the sternum.

Results of Peripheral Neuropathy:

Muscle weakness, with or without atrophy and sensory changes.

Muscular dystrophy is characterized by what pathophysiologic changes?

Muscular necrosis, with replacement with fat and connective tissue

The basal ganglia play a role in coordinated movements. Part of the basal ganglia system is the striatum, which involves local cholinergic interneurons. What disease is thought to be related to the destruction of the cholinergic interneurons?

Myasthenia gravis

Which one of the following is the most common primary intracranial tumor in adults?

Neurogliomas or neoplasms of astrocytic origin Neurogliomas or neoplasms of astrocytic origin are the most common type of primary brain tumor in adults.

alkaline phosphatase

Osteoblasts secrete an enzyme, alkaline phosphatase, acts locally in bone tissue to increase calcium and phosphate levels to the point at which precipitation occurs Osteoblast activity contributes to the increase in alkaline phosphatase serum levels following bone injury and fractures

lacuna

Osteocytes lie in a small lake filled with extracellular fluid, called lacuna, and are surrounded by an intracellular matrix (see Fig. 56.3)

osteons

Osteons: consist of concentric lamellae of bone matrix surrounding a central canal, the Haversian canal, which contains the blood vessels and nerve supply for the osteon (see Fig. 56.3)

Which bone cell is responsible for continual replacement of worn out bone tissue?

Osteoprogenitor

Which of the following bone cells are located in the growth plate?

Osteoprogenitor

tendonitis

Overuse can cause tendonitis or inflammation of the tendons

pain transmission

PAG region also interacts with pontile noradrenergic neurons and the medullary nucleus raphe magnus (serotonin acts as NT) to block what?

The nurse recognizes that parathyroid Hormone (PTH) has what relationship to bone formation?

PTH is a regulator of calcium and phosphate.

In the kidney, _____ stimulates tubular reabsorption of calcium while reducing reabsorption of phosphate.

PTH(parathyroid hormone)

fibrocartilage

Pads between vertebrae that are shock absorbers symphysis pubis menisci TMJ IV discs

A "collage of matrix madness" describes:

Paget Disease

The nurse is assessing a client and notes the client is now displaying decerebrate posturing. The position would be documented as:

Rigidity of the arms with palms of the hands turned away from the body and with stiffly extended legs and plantar flexion of the feet Decerebrate (extensor) posturing results from increased muscle excitability. It is characterized by rigidity of the arms with palms of the hands turned away from the body and with stiffly extended legs and plantar flexion of the feet.

When teaching a group of nursing students about rheumatic disorders, a nurse emphasizes which of the following important differences when caring for the older adult?

Risk for falls

In _____, the "blush" eventually becomes a permanent dark red erythema.

Rosacea

_____ is often referred to as sixth disease.

Roseola Infantum

A common cause of bacterial pneumonia in BOTH the community and hospital setting is a. E. coli b. S. aureus c. Enterobacter d. S. pneumoniae

S. Aureus

A client presents to the pain clinic for a steroid injection into the spine due to increasing pain around the joints. The health care provider tells the client, "You have inflammation where your tendons/ligaments insert into the bone. This injection should help." The nurse assisting with the procedure recognizes this to be characteristic of:

Sacroiliitis

The most common cause of an ischemic stroke is which of the following? a) Thrombosis b) Cardiogenic embolus c) Intracerebral arterial vasculitis d) Vasospasm

Thrombosis Thrombi are the most common cause of ischemic strokes, usually occurring in atherosclerotic blood vessels.

The most common cause of ischemic stroke is:

Thrombosis. Ischemic stroke includes those caused by large artery thrombosis (20%), small artery thrombosis (25%), cardiogenic embolism (20%) and cryptogenic (undetermined cause)(30%), making thrombosis the most common cause (45%).

The health care provider is assessing a client for carpal tunnel syndrome. The health care provider performs light percussion over the median nerve at the wrist. This assessment is known as:

Tinel sign

Diagnosis of Carpal Tunnel:

Tinned sign--the development of tingling sensation radiating into the palm of the hand that is elicited by light percussion over the median nerve of the wrist.

During a routine 2-month checkup at the pediatric clinic, a mother expresses concern that her son looks "cross-eyed." She asks if she need to put patches over his good eye. Assessment reveals full eye movement, and the child uses each eye independently. The health care provider explains that the best treatment for the infant's eye problem is:

To prepare for some surgery to correct this problem early on to correct the eye muscle disorder.

The H. Pylori protobacteria causes peptic ulceration by a. acids b. toxins c. ischemia d. bleeding

Toxins

The metaphysis is the part of the bone that fans out toward the epiphysis. What is the metaphysis composed of?

Trabeculae

In contrast to synarthroses joints, synovial joints are linked to the bone by: A) a joint capsule. B) hyaline cartilage. C) connective tissue. D) interosseous ligaments.

a. a joint capsule

Which of the following is the primary role of fibrocartilage? A) Absorption of physical shock B) Provision of flexibility C) Facilitation of long bone growth D) Tissue hydration

a. absorption of physical shock

Which of the following statements most accurately describes the anatomy and physiology of the bone marrow? A) Hematopoiesis takes place in red bone marrow. B) Yellow bone marrow predominates in infants. C) By adulthood, all red bone marrow has been replaced by yellow bone marrow. D) Yellow bone marrow is hematopoietically active in infants, but not in adults.

a. hematopoiesis takes place in red bone marrow

The iatrogenic form of Cushing syndrome is caused by a. long term corticosteroid therapy b. pituitary tumor-secreting ACTH c. benign or malignant adrenal tumor d. ectopic ACTH-secreting lung tumor

a. long-term corticosteroid

In growing bone, severe vitamin C deficiency slows bone growth by impairing: A) organic matrix formation. B) calcification of new bone. C) growth plate separation. D) widening of the cortex.

a. organic matrix formation

Which of the following joints is classified as a synarthrosis? A) The joint between two vertebrae B) The joint between the femur and the pelvis C) The joint between the humerus and the radius and ulna D) An interphalangeal joint of the hand (knuckle)

a. the joint between two vertebrae

calcitonin is secreted by

c cells of thyroid

An intracranial epidural hematoma causes focal symptoms that can include ___ __ __

ipsilateral pupil dilation

An intracranial epidural hematoma causes focal symptoms that can include:

ipsilateral pupil dilation

cluster (headache)

is accompanied by conjunctival injection (redness) or lacrimation (tearing), ipsilateral nasal congestion, eyelid edema, or forehead/facial sweating

As chronic renal failure progresses, the second stage (renal insufficiency) is identified by

isotonic polyuria

Abnormally high accumulation of bilirubin in the blood causes a. jaundice b. cholestasis c. xanthomas d. biliary cirrhosis

jaundice

___ makes bile salts/bilirubin and the __ stores them

liver gallbladder

special somatic (afferent neurons)

located in muscles, tendons & joints

shapes of compact bones

long, short, flat, irregular

A nurse caring for a patient in myasthenic crisis identifies a priority concern as:

maintenance of airway and respiration.

headache

many factors may precipitate a ________ including: foods/food additives (MSG, aged cheese, chocolate), missed meals, menstral periods (estrogen levels), certain medications

phantom limb pain

may be the result of regenerating nerve tissue being trapped within scar tissue at the amputation site or the spontaneous firing of spinal cord neurons that have lost their sensory input from the amputated limb

tension-type

may result from oromandibular dysfunction, stress, anxiety, depression, overuse of analgesics or caffeine

myelinated

mechanical or thermal stimuli

endosteum

membranous lining of the hollow cavity of the bone

primary (headache)

migraine, cluster, tension-type

PAG (periaqueductal gray) region

modulates ascending pain impulses by producing a state of analgesia = endogenous analgesia center

primary (somatosensory) cortex

most of surface concerns fingers, lips & tongue

Alcohol liver disease manifests in three stages. The intermediate stage, alcoholic hepatitis, is characterized by liver cell a. necrosis b. nodules c. atrophy d. hypertrophy

necrosis

The functional unit of the kidney is:

nephron

Do osteoclasts have PTH receptors?

no ostebolasts stimulate osteoclasts so if you need calcium in blood. the pth hormone

synostoses

non-movable joints in which the surfaces of the bones are joined by dense connective tissue or bone

substantia nigra

not enough dopamine

Calcium and vitamin D

nurse is teaching a client who is to begin long-term therapy with prednisone to treat rheumatoid arthritis. The nurse should instruct the client to take which of the following supplements while taking this medication?

specificity theory

pain is a separate sensory modality evoked by specific receptors that transit information to the forebrain where pain is experienced

allodynia

pain that follows a non-noxious stimulus to apparently normal skin

cutaneous

pain that is bright, sharp, burning pain with origin in skin or subcutaneous tissues

visceral (splanchnic)

pain that is diffuse, poorly-defined pain that results from stretching, distention, or ischemia of tissues in a body organ

The most common sign of epithelial cell bladder cancer is

painless hematuria

Disorders of the pyramidal tracts, such as a stroke, are characterized by ___

paralysis

pyramidal- expect __ and ___

paralysis spastic ( stiff, more muscle tone/mass longer)

Complete spinal cord sever...T12 and above. ____ from waist down and sympathetic wouldn't work...bladder won't work

paralyzed

starts at the feet ascending paralysis __ neuropathy demylenation will it resolve?

poly yes (happens quickly...worried about diaphragm paralysis) (Guillen-Barre is PNS...MS is CNS)

The late manifestations of cirrhosis are related to liver failure and ___ ___

portal hypertension

Proprioception

position sense

pain

possesses an urgent, primitive quality

A patient has wrist inflammation causing compression of the median nerve in the carpal tunnel. Manifestations of this syndrome include ___ ___ ___

precision grip weakness

A patient has wrist inflammation causing compression of the median nerve in the carpal tunnel. Manifestations of this syndrome include: little finger numbness. forearm paresthesia. loss of tendon reflexes. precision grip weakness.

precision grip weakness.

More complex patterns of movements, such as throwing a ball or picking up a fork, are controlled by the ______ cortex in the frontal lobe.

premotor

TMJ (temporomandibular joint pain)

presents as facial muscle pain, headache, neck ache or earache

central axon

projects to the CNS

When Acute Tubular Necrosis (ATN) has occurred instead of prerenal azotemia, lab test findings show the presence of excessive urine

protein

increased GFR leads to ___ and ___

proteinuria hypoalbuminurea

_____ originates in the skin's free nerve endings, causing scratching in an attempt to get relief.

pruritus

Unlike Crohn type of inflammatory bowel disease, the ulcerative colitits type is characterized by a. skip lesions b. steatorrhea c. gastric ulcers d. pseudopolyps

pseudopolyps

types of bone marrow

red and yellow

complex regional pain syndrome

reflex sympathetic dystrophy

dorsal column (medial lemniscal pathway)

relays precise information regarding spatial orientation

prostaglandins

released from inflamed tissues enhance the sensitivity of pain endings but they don't directly stimulate them

A hallmark of irritable bowel syndrome is abdominal pain __ __ ___

relieved by defecation

The nurse observes a new nurse performing the test for Kernig's sign on a client. The new nurse performs the test by providing resistance to flexion of the knees while the client is lying with the hip flexed at a right angle. The nurse should explain to the new nurse that: a) the sign elicited was the Budzinski's sign. b) the sign elicited was the obturator sign. c) resistance should be provided with the knee in a flexed position. d) the client should be in a sitting position.

resistance should be provided with the knee in a flexed position. The test for Kernig's sign for meningeal irritation is performed by providing resistance to flexion of the knees while the client is lying with the hip flexed at a right angle.

Transient ischemic attacks (TIAs) are characterized by ischemic cerebral neurologic deficits that:

resolve within one hour of onset

A patient with a diagnosis of myasthenia gravis has required a mastectomy for the treatment of breast cancer. The surgery has been deemed a success, but the patient has gone into a myasthenic crisis on postoperative day one. Which of the following measures should the care team prioritize in this patients immediate care?

respiratory support and protection of the patients airways

cold receptors

respond to < 34°C

warmth receptors

respond to > 34°C

nociceptors

respond to mechanical, chemical, & thermal stimuli

bursae

sacs containing synovial fluid not apart of the joint prevent friction

The edema that develops in persons with glomerulonephritis and nephrotic syndrome reflects:

salt and water retention

Candida albicans fungal infections can be differentiated from a tinea fungal infection by the presence of:

satellite lesions

flat bone

scapula and sternum

stiffness and spasticity

sclerosis

In response to the rise in sex hormones at puberty, _____ glands enlarge.

sebaceous

A thermal burn described as involving the entire epidermis and dermis is classified as a _____ burn.

second-degree full thickness

unmyelinated

slow-wave pain

trigeminal neuralgia

tic douloureux

temperature

warmth & cold

tactile

when _____ information is lost → total anesthesia (numbness)

most common tumor in kids

wilm's

pain threshold

~ same for all individuals

When administering a corticosteroid to a patient with rheumatoid arthritis, a nurse includes which of the following in educating the patient about the medication? Select all that apply.

• "Corticosteroids do not prevent joint destruction." • "Pain is relieved with use of corticosteroid medications." • "These drugs are for short-term use."

Which of the following patients would the nurse expect to find woven bone scaffolding in place to facilitate healing?

• 22 year old patient recovering from an auto accident where they incurred a fracture of their femur. • 55 year old prostate cancer patient undergoing radiation therapy for bone metastasis.

A client has developed bursitis of the right knee. Select all that apply.

• Bursitis develops where pressure is exerted. • Movement may be limited. • Swelling may occur. • Pain may develop.

A client may be at risk for the development of hypoxia. Select the conditions that would place a client at risk. Select all that apply.

• Carbon monoxide poisoning • Increased oxygenation by the lungs • Severe anemia Hypoxia usually is seen in conditions such as exposure to reduced atmospheric pressure, carbon monoxide poisoning, severe anemia, and failure of the lungs to oxygenate the blood.

A client may be at risk for the development of hypoxia. Select the conditions that would place a client at risk. Select all that apply. a) Increased atmospheric pressure b) Increased oxygenation by the lungs c) Thrombocytopenia d) Age e) Severe anemia f) Carbon monoxide poisoning

• Carbon monoxide poisoning • Increased oxygenation by the lungs • Severe anemia Hypoxia usually is seen in conditions such as exposure to reduced atmospheric pressure, carbon monoxide poisoning, severe anemia, and failure of the lungs to oxygenate the blood. The other options will not alter oxygen levels.

Which type of materials are not included in a cancellous bone? Select all that apply.

• Cartilaginous • Compact • Densely calcified

A 68 year-old woman has had her mobility and quality of life severely affected by rheumatoid arthritis (RA). Place the following pathophysiological events involved in her health problem in the correct order that they most likely occurred. Use all the options.

• Inflammatory response • Interaction between rheumatoid factor (RF) and IgG • T-cell mediated immune response • Pannus invasion • Destruction of articular cartilage

Which statement are true regarding synarthroses? Select all that apply.

• It is connected to bone by hyaline cartilage. • It has little or no range of motion. • It is found primarily in the lower extremities.

Which statements are true regarding red bone marrow? Select all that apply.

• It is the primary type of bone marrow found in infants. • It is primarily present in the vertebrae, ribs, and sternum. • It is the site of blood cell formation.

Which symptoms would support the diagnosis of a stroke involving the posterior cerebral artery? Select all that apply. a) Repeating of verbal responses b) Loss of central vision c) Contralateral hemiplegia d) Denial of paralyzed side e) Aphasia

• Loss of central vision • Repeating of verbal responses Posterior cerebral artery stroke would produce visual defects and the repeat of verbal and motor responses. -The other options are seen in middle cerebral artery strokes.

The nurse reading the results of a lumbar puncture cerebral spinal fluid analysis anticipates that the client's meningitis will be self-limiting in nature because of which of the following findings? Select all that apply.

• Lymphocytes • Moderately increased protein Viral meningitis, which presents with lymphocytes, moderately increased protein and normal glucose levels in the cerebral spinal fluid upon lumbar puncture, is self-limiting in nature.

A nurse in the emergency room is assessing a client's level of consciousness. The client appears very drowsy but is able to follow simple commands and respond to painful stimuli appropriately. The nurse should document the client's level of consciousness as which of the following? a) Confusion b) Obtundation c) Lethargy

• Obtundation Characteristics of obtundation include responding verbally with a word, arousable with stimulation, responds appropriately to painful stimuli, follows simple commands, and appears very drowsy.

The intercellular matrix of bone is composed of two types of substances: organic matter and inorganic salts. Which of the following are organic matter? Select all that apply.

• Osteoblasts • Nerves • Blood vessels

A post-menopausal female client diagnosed with osteoporosis asks the nurse how a decrease in estrogen can cause problems with bone density. Which of the following statements is the best response from the nurse?

"A decrease in estrogen levels, which occurs at menopause, results in increased resorption of bone."

During a flu shot clinic, one of the questions the student nurse asks relates to whether the patient has had Guillain-Barre? syndrome in their medical history. The patient asks, "What is that?" How should the nursing student reply?

"A type of paralysis that affects movement on both sides of the body. It may even involve the respiratory muscles."

10. The synovial membrane is innervated only by _____ that control blood flow.

autonomic fibers

cartilage

avascular consists of chondrocytes a firm but flexible type of connective tissue

thalamic neuron

axon reaches the somatosensory cortex of parietal lobe

The most common indicator of acute renal failure is

azotemia

all of the protein breakdown products together

azotemia

The cardinal symptoms of Parkinson disease include:

bradykinesia

The cardinal symptoms of Parkinson disease include

bradykinesia (slow movement)

The cardinal symptoms of Parkinson disease include: hypotonia. bradykinesia. paresthesia. lack of sweating.

bradykinesia.

nociceptors

bradykinin, histamine, serotonin, & K activate & sensitize

in order, the ways to moniter blood ph: ___, ___ , finally ___

buffers respiration kidneys

medial lemniscus

crosses at medulla & reaches thalamus on opposite side of brain where sensation began

Pacinian corpuscles

detect tissue vibration

free nerve endings

detect touch & pressure

The most common cause of an ischemic stroke is which of the following?

Thrombosis Thrombi are the most common cause of ischemic strokes, usually occurring in atherosclerotic blood vessels.

Acute pancreatitis involves activated pancreatic enzymes that escape into surrounding tissues to cause _______.

autodigestion

A sudden traumatic complete transection of the spinal cord results in _______ below the level of injury. flaccid paralysis vasoconstriction deep visceral pain 3+ tendon reflexes

flaccid paralysis

When educating a client with possible glucocorticoid dysfunction, the nurse will explain that the CRH controls the release of ACTH. The best time to perform the blood test to measure peak ACTH levels would be:

-06:00 to 08:00 AM

In contrast to synarthroses joints, synovial joints are linked to the bone by: A) a joint capsule. B) hyaline cartilage. C) connective tissue. D) interosseous ligaments.

A) a joint capsule.

Premature osteoporosis is being seen increasingly in female athletes because of an increased prevalence of: A) amenorrhea. B) high protein intake. C) abnormal body fat. D) osteoarthritis.

A) amenorrhea.

The most commonly occurring early symptoms of systemic lupus erythematosus (SLE) include: A) arthralgia. B) tendon rupture. C) facial hair growth. D) pyelonephritis.

A) arthralgia.

Systemic sclerosis (scleroderma) is an autoimmune disease of connective tissue characterized by: A) fibrosis. B) thin fragile skin. C) collagen deficiency. D) avascular necrosis.

A) fibrosis.

Disorders that affect cortical bone typically result in: A) fractures of long bones. B) impaired collagen synthesis. C) infection. D) vertebral fractures.

A) fractures of long bones.

Reflex activity involves which of the following?

All of the above

synovial joints

All tissues of the synovial joints (except the articulating surfaces of the articulating cartilage) receive nourishment either directly or indirectly from blood vessels

Global and focal brain injuries manifest differently. What is almost always a manifestation of a global brain injury?

Altered level of consciousness

A daughter is concerned because her elderly parent has been diagnosed with osteomalacia. The daughter asks the nurse why this happened. The best response would be that:

Intestinal absorption slows as a natural aging process

A patient is asked to stand with feet together, eyes open and hands by the sides. Then the patient is asked to close their eyes while the nurse observes for a full minute. What assessment is the nurse performing?

Proprioception.

Hallmark of nephrotic syndrome: ___ ___ and ___

generalized edema and massive proteinuria (decreased capillary colloidal (osmotic) pressure...if losing proteins, fluid moves into the tissue)

A client begins to exhibit signs and symptoms of a stroke at a community health fair. Emergency care for the client includes:

going to the nearest stroke center.

osteoclasts

"Bone chewing" cells that function in the resorption of bone, removing the mineral content and the organic matrix

An adult patient has been diagnosed with polycystic kidney disease. Which of the patient's following statements demonstrates an accurate understanding of this diagnosis?

"I suppose I should be tested to see if my children might inherit this."

A 42 year old male has been diagnosed with renal failure secondary to diabetes mellitus and is scheduled to begin dialysis soon. Which of the following statements by the client reflects and accurate understanding of the process of hemodialysis?

"I won't be able to go about my normal routine during treatment."

Which of the following statements by the husband of a patient with Alzheimer's demonstrates an accurate understanding of his wife's medication regimen?

"I'm really hoping these medications will slow down her mental losses."

A 64 year-old man was diagnosed 19 months ago with bilateral osteoarthritis (OA) in his knees, and has come to his family physician for a checkup. The client and his physician are discussing the affects of his health problem and the measures that the man has taken to accommodate and treat his OA in his daily routines. Which of the following statements by the client would necessitate further teaching?

"I've been avoiding painkillers because I know they can mask damage that I might be inflicting on my knees."

A client diagnosed with Paget disease asks the health care provider how the disease developed. The best response would be:

"It is thought to have a probable association with a viral infection: paramyxovirus."

The spouse of a patient diagnosed with Alzheimer's disease asks the nurse why the patient often neglects to take a shower. The spouse states that the patient was always diligent with hygiene in the past; however, over the past few months that has not been the case. Which of the following is the nurse's best response?

"You should remind the patient to shower." The patient should be reminded to shower because most likely he or she has difficulty remembering to do so. In the moderate stage of Alzheimer's disease, which can last for several years, it is not unusual for hygiene to be neglected because the person may just not remember if he or she did or did not shower.

astereogenesis

*Dorsal Column Medial Lemniscal Pathway* if medial lemniscal pathway is functional but somatosensory cortex is damaged, you can describe object but not recognize the specific object type

According to the Glasgow Coma Scale, opening one's eyes to only painful stimuli would receive which score?

2 Only opening eyes to painful stimulation is scored as a 2. Spontaneously opening eyes is scored as a 4; opening eyes to speech is scored as a 3; no opening is scored as a 1.

thalamus

2 parallel pathways transmit information from spinal cord to the _______: 1.dorsal column medial lemniscal pathway 2.anterolateral pathway

Denial

A young adult client with a new diagnosis of rheumatoid arthritis states, "The pain in my joints is just a temporary thing. If I keep eating right and exercising, it'll go away." The nurse should identify the client is exhibiting which of the following defense mechanisms?

articulations

Articulations (or joints): areas where two or more bones meet

A compression of the medial nerve:

Carpal Tunnel Syndrome

collagen

Collagen is an inelastic and insoluble fibrous protein

Juvenile dermatomyositis is a chronic inflammatory myopathy that commonly manifests systemically. What is the treatment of choice for this myopathy?

Corticosteroids

Select the laboratory blood test that would be a used to suggest a diagnosis of muscular dystrophy (MD).

Creatine kinase

Peripheral neuropathy occurs most commonly with which one of the following disorders?

Diabetes

An emergency room nurse receives a report that a client's Glasgow Coma Scale (GCS) is 3. The nurse prepares to care for a client with which of the following? a) Normal flexion b) Confused conversation c) Flaccid motor response d) Spontaneous eye opening

Flaccid motor response A score of 3 on the Glasgow Coma Scale indicates the lowest possible score in each of the three scoring categories (eye opening, motor response, and verbal response) and includes flaccid or no motor response, no verbal response and the inability to open the eyes.

Most common uncomplicated urinary tract infections are caused by _____ that enter through the urethra

E. Coli

endosteum

Endosteum: membrane that lines the spaces of spongy bone, the marrow cavities, and the Haversian canals of compact bone Composed mainly of osteoprogenitor cells that contribute to the growth and remodeling of bone; and are necessary for bone repair

The CT scan report identified that a client with a skull fracture has developed a hematoma that resulted from a torn artery. The report would be interpreted as:

Epidural hematoma

The CT scan report identified that a client with a skull fracture has developed a hematoma that resulted from a torn artery. The report would be interpreted as: a) Epidural hematoma b) Intracranial hematoma c) Subdural hematoma d) Chronic subdural hematoma

Epidural hematoma An epidural hematoma is one that develops between the inner side of the skull and the dura, usually resulting from a tear in an artery, most often the middle meningeal, usually in association with a head injury in which the skull is fractured. -A subdural hematoma results from a torn vein; -chronic subdural hematoma is common in older persons: -brain atrophy causes the brain to shrink away from the dura and to stretch fragile bridging veins. - An intracranial hematoma occurs when a blood vessel ruptures within your brain or between your skull and your brain.

During the first two decades of life, the skeleton undergoes general overall growth. Select the structure that promotes this process.

Epiphyseal growth plate

A seronegative inflammatory arthropathy is psoriatic arthritis. What drug has been found to be beneficial in controlling both the psoriasis and the arthritis in these clients?

Etanercept

Following surgery for a large malignant brain tumor, the nurse should anticipate discussing which further treatment option with the family that may ensure that any remaining cancer cells will be killed? a) Gamma knife radiation b) Immunotherapy c) Chemotherapy d) Stem cell transplant

Gamma knife radiation Most malignant brain tumors respond to external irradiation. Irradiation can increase longevity and sometimes can allay symptoms when tumors recur. The treatment dose depends on the tumor's histologic type, responsiveness to radiation, and anatomic site and on the level of tolerance of the surrounding tissue. A newer technique called gamma knife combines stereotactic localization of the tumor with radiosurgery, allowing delivery of high-dose radiation to deep tumors while sparing the surrounding brain.

fibrocartilage is made of

High collagen, low elastic

A nurse is assessing a client with symptoms of botulism. The nurse will question the client regarding ingestion of which of the following?

Home-grown and canned vegetables

A nurse working in a busy orthopedic clinic is asked to perform the Tinel sign on a patient having problems in their hand/wrist. In order to test Tinel sign, the nurse should give the patient which of the following directions?

I'm going to tap (percuss) over the median nerve in your wrist, tell me what sensation you feel while I am doing this. Does the sensation stay in the wrist or go anywhere else?

Select the major stimulus for calcitonin synthesis and release.

Increased serum calcium

Cells that bind and process antigens in the epidermis are known as _____ cells.

Langerhans

calcitonin

Lowers blood calcium levels

A clinic nurse plans care for a newly diagnosed osteoarthritis client. Which of the following items should this client be provided with educational materials? Select all that apply.

Physical measures are aimed at improving the supporting structures of the joint and strengthening opposing muscle groups involved in cushioning weight-bearing forces. This includes a balance of rest and exercise, use of splints to protect and rest the joint, use of heat and cold to relieve pain and muscle spasm, and adjusting the activities of daily living. The involved joint should not be further abused, and steps should be taken to protect and rest it. These include weight reduction (when weight-bearing surfaces are involved) and the use of a cane or walker if the hips and knees are involved. A recent multicenter trial funded by the National Institutes of Health found that glucosamine and chondroitin (alone or in combination) were no better than placebo in reducing pain in the total group of persons with knee pain. Narcotics are usually not the pain medication of choice for OA.

A patients recent computed tomography (CT) scan has revealed the presence of hydrocephalus. Which of the following treatment measures is most likely to resolve this health problem?

Placement of a shunt

The initiating event in the development of nephrotic syndrome in the glomerular membrane that causes increased permeability to:

Plasma proteins

Giant cell arteritis is a comorbid condition of which of the following?

Polymalgia rheumatica

An elderly female complains about waking up one morning with pain/stiffness in her neck/shoulders. Lab work reveals an elevated erythrocyte sedimentation rate (ESR). The physician gives the client a 3-day trial of prednisone, which significantly improves the pain. The health care provider correlates this information and diagnoses which of the following disorders?

Polymyalgia rheumatica is a common syndrome of older clients, rarely occurring before 50 years and usually after 60 years of age. Reiter syndrome, psoriatic arthritis, and ankylosing spondylitis may occur at younger ages.

Guillain-Barré syndrome is characterized by which of the following forms of neuron damage?

Polyneuropathy

The parent of a toddler with Duchenne muscular dystrophy reports that the child has an increase in muscle size but a decrease in strength. The nurse documents this using which of the following medical terms?

Pseudohypertrophy

A nurse caring for a client with multiple sclerosis notes that the client has mood swings. Which of the following can best explain this?

Psychological manifestation due to involvement of white matter of cerebral cortex

A nurse is caring for a patient with rheumatoid arthritis (RA) who states she "caught" this from her mother. Which of the following statements is the correct explanation of how this type of arthritis develops?

RA has an autoimmune and genetic predisposition.

Almost all people with diffuse scleroderma develop:

Raynaud phenomenon

A patient has been diagnosed with scleroderma. Which of the following assessment findings does the nurse expect?

Raynaud's phenomenon

The nurse assessing a patient with scleroderma with CREST variant would include an assessment for which of the following?

Raynaud's phenomenon

Nystagmus due to cerebellar dysfunction would most likely interfere with which activity?

Reading

A nurse at a long term care facility provides care for an 85 year-old man who has had recent transient ischemic attacks (TIAs). Which of the following statements best identifies future complications associated with TIAs? TIA's:

Resolve rapidly but may place the client at an increased risk for stroke. TIAs can be considered a warning sign for future strokes. They are not hemorrhagic in nature and their effects are not normally cumulative. They may require treatment medically or surgically.

Transient ischemic attacks (TIA) are characterized by ischemic cerebral neurological deficits that:

Resolve within one hour of onset

Death caused by muscular dystrophy in early adulthood is usually due to which of the following?

Respiratory and cardiac muscle involvement

The earliest signs of decreased level of consciousness include:

Restless, agitation

When the glomerular transport maximum for a substance such as blood glucose is exceeded and its renal threshold has been reached, the substance will:

Spill into the urine

The health care provider is assessing a client with a history of ankylosing spondylitis, to note progression of the disease. The most important area for the provider to assess would be:

Spine

A patient with ankylosing spondylitis wants to begin an exercise program. Which of the following does the nurse recommend?

Swimming

synarthroses

Synarthroses: Joints that lack a joint cavity; move little or not at all

diarthrodial joint

Synovial (diarthrodial) joints: Freely movable joints ~ most joints in the body are included in this category Range of movement varies: Almost none (e.g., sacroiliac joint) Simple hinge movement (e.g., interphalangeal joint) Movement in many planes (e.g., shoulder or hip joint) Bony surfaces of synovial joints are covered with thin layers of articular cartilage; and the cartilaginous surfaces of these joints slide past each other during movement Synovial joints are the one most frequently affected by rheumatoid arthritis

Which of the following substances helps maintain a smooth surface in joint cartilage?

Synovial fluid

While reviewing the following diagnostic findings on a group of patients with joint complaints, which finding would be a priority for further investigation and possible medical intervention?

Synovial fluid aspiration indicates the presence of monosodium urate crystals.

A client has experienced trauma of the musculoskeletal structure that connects muscle to bone. The nurse interprets this as an injury of the:

Tendon

A nurse is caring for a client who injured her knee while playing basketball. Tests reveal a torn connective tissue that connects muscles to bones. Based on this finding, the nurse prepares to teach the client about which of the following anatomical structure that is injured?

Tendons

A client has tendon damage. What will the nurse teach the client about the function of tendons?

Tendons connect muscles and bones

tubular bone

The compact (cortical) bone of the long tubular bones is provided with blood supply from two sources: Nutrient arteries: enter the bone through a nutrient foramen; and supply the marrow space and the internal one half of the cortex Perforating arteries: small arteries that extend inward from the periosteum and anastomose in the cortex with branches of the nutrient arteries coming from the bone marrow

What part of the bone assists in protecting the bone?

The compact bone

During class, the instructor asks students to explain the pathophysiology behind development of multiple sclerosis. Which student gave the most accurate description?

The demyelination and subsequent degeneration of nerve fibers and decreased oligodendrocytes, which interfere with nerve conduction

Which of the following descriptions is true of peritoneal dialysis?

Treatment involves the introduction of a sterile dialyzing solution, which is drained after a specified time.

A weak urinary stream, postvoid dribbling, frequency of urination, and nocturia are classic symptoms of an enlarged prostate.

True

Bone may take up toxic substances from the circulation, which can be viewed as a protective mechanism.

True

The nurse is teaching a patient with rheumatoid arthritis about pannus, which develops in the affected joint area. Which of the following does the nurse include to describe pannus?

Vascular granulation tissue that destroys cartilage and bone

A patient in the intensive care unit who has a brain tumor has experienced a sharp decline. The care team suspects that water and protein have crossed the blood-brain barrier and been transferred from the vascular space into the client's interstitial space. Which of the following diagnoses best captures this pathophysiology?

Vasogenic edema Vasogenic edema occurs with conditions that impair the function of the blood-brain barrier and allow transfer of water and protein from the vascular into the interstitial space. It occurs in conditions such as tumors, prolonged ischemia, hemorrhage, brain injury, and infectious processes.

The nurse has just completed an assessment on a client admitted with Guillain-Barré syndrome. The nurse determines that a priority of care will be:

Ventilatory assessment and support

A topical corticosteroid has been prescribed for short-term use on an infant's skin. When planning the use of a topical medication for an infant, what principle should guide the nurse's actions?

With topical medications, cautious use is recommended with infants and young children due to the fact they have more permeable skin and are more likely to absorb the topical drugs. Infants have significant subcutaneous fat, but this does not necessitate higher concentrations of drugs. Topical drugs do not necessarily have to be covered with a barrier following administration.

analgesia

absence of pain

Which of the following conditions is caused by autoimmune antibodies binding to TSH receptors and stimulating thyroid hormone secretion? a. Cretinism b. Grave's disease c. Hashimoto's thyroiditis d. Myxedema e. Thyrotoxicosis

b. Grave's disease

The most common invasive cancer in humans is:

basal cell carcinoma

The typical attack of _____ in monoarticular and usually affects the first metatarsophalangeal joint.

gout

Crohn's type of inflammatory bowel disease is characterized by ___ ___

granulomatous lesions (skip)

two types of bone tissue

laminar bone woven bone

osteoblasts

lay down new bone

osteocytes

live in mature bone and maintain the ecm

The adverse effects of emboli on the pulmonary circulation and airways include a. excess surfactant b. area bronchodilation c. local vasoconstriction d. lower lobe consolidation

local vasoconstriction

trigger points

localized points on skin or mucous membranes that produce immediate intense pain when stimulated by light tactile stimulation

postherpetic neuralgia

localized recurrent infection by the varicella virus that has remained latent in the dorsal root ganglia since the initial attack of chickenpox

pain tolerance

maximum duration & intensity of pain that a person can endure

Headaches

may be primary or secondary

pain

can be classified according to location, referral, & duration

migraine headache

can occur without or with aura

two types of mature bone

cancellous compact

sensory unit

cell body of dorsal root ganglion neuron + its receptor + its central axon

Merkels disks

continuous determination of touch against the skin

aspirin (& NSAIDs)

control pain because they block enzyme needed for PG synthesis

special somatic (afferent neurons)

convey information on position & movement of body

general somatic (afferent neurons)

convey sensations of pain, touch & temperature

general visceral (afferent neurons)

convey sense of fullness & discomfort

secondary (headache)

due to meningitis, tumor or cerebral aneurysm

tension-type

dull, aching, diffuse, nondescript headaches occurring in a hatband distribution around the head

Premature osteoporosis is being seen increasingly in female athletes because of an increased prevalence of:

eating disorders and amenorrhea

Regardless of the cause, chronic kidney disease results in progressive permanent loss of nephrons, glomerular filtration and renal:

endocrine functions

NTs

endogenous analgesic mechanisms involve three families of opioid peptides that can function as ____ 1.enkephalins 2.endorphins 3.dynorphins

cluster headache

episodic occurrences

trigger for red blood cell production ___...If not working can make the patient __

erythropoietin anemic

A patient is taking ibuprofen (Motrin) for knee pain. The patient is admitted to the hospital with abdominal pain. Which of the following assessments should the nurse prioritize?

Nonsteroidal anti-inflammatory agents that block COX-1 and COX-2 place the patient at risk for gastrointestinal bleed. Patients who have symptoms of abdominal pain and are taking NSAIDs should be assessed for signs and symptoms of gastrointestinal bleed. Assessing the patient for diarrhea is not related to ibuprofen (Motrin) administration. Assessing the patient for hematuria or hemoptysis is not a priority.

The nurse caring for a client with a newly diagnosed intracranial tumor anticipates that the neoplasm will be which of the following?

Astrocytic neoplasms Collectively, astrocytic neoplasms are the most common type of primary brain tumor in adults; therefore the nurse anticipates that this is the cause of the client's intracranial tumor.

The nurse caring for a client with a newly diagnosed intracranial tumor anticipates that the neoplasm will be which of the following? a) Oligodendroglioma b) Meningioma c) Astrocytic neoplasms d) Metastatic carcinoma

Astrocytic neoplasms Collectively, astrocytic neoplasms are the most common type of primary brain tumor in adults; therefore the nurse anticipates that this is the cause of te client's intracranial tumor.

When acetylsalicylic acid (aspirin) is administered in low doses, it blocks the synthesis of thromboxane A2. What physiological effect results from this action?

At low doses, aspirin blocks the synthesis of thromboxane A2 to inhibit platelet aggregation; this lasts for the life of the platelet.

When a nurse is caring for a patient who has systemic lupus erythematosis (SLE), which of the following causes the disease?

Autoimmune process

When teaching a patient with rheumatoid arthritis (RA), which of the following factors does the nurse explain is an underlying cause of this disease?

Autologous antibodies

A gymnastics student lands awkwardly and hurts her ankle. After MRI scan, it was revealed that she has a torn cartilage. The health care worker states it may take up to 3 to 4 months for this injury to heal. The basic physiologic reason behind the prolonged recovery is due to the fact that cartilage is/has primarily:

Avascular

The most common indicator for acute renal failure is:

Azotemia

A 55-year-old male patient has reported joint pain in his feet. Which of the following blood work results should prompt further testing to rule out primary gout? A. Increased C-reactive protein (CRP) B. Increased serum uric acid C. Increased polymorphonuclear leukocytes D. Increased serum cortisol

B

A child has been hospitalized for the treatment of hematogenous osteomyelitis. The defining characteristic of this type of osteomyelitis is: A. the presence of dead bone tissue. B. introduction of microorganisms from the bloodstream. C. bacterial proliferation in the absence of the classic sings of infection. D. destruction of the vascular network in the endosteum.

B

A nurse is caring for a child with systemic-onset juvenile idiopathic arthritis (JIA). On which of following symptoms should the nurse focus?

Fever

Laparoscopic knee surgery on a 22 year-old basketball player has necessitated entry into the synovial cavity. The surgeon performing the procedure would be aware of which of the following relevant characteristics of synovial tissue?

Few pain receptors are located in the synovial membrane.

Our bodies contain three types of cartilage: elastic, hyaline, and fibrocartilage. Which of these types of cartilage is found in the symphysis pubis?

Fibrocartilage

fibrocartilage

Fibrocartilage: has characteristics that are intermediate between dense connective tissue and hyaline cartilage Found in the intervertebral discs; in areas where tendons are connected to bone; and in the symphysis pubis "Slipped disc" back injuries are fibrocartilage injuries

intervertebral discs

Fibrocartilage: has characteristics that are intermediate between dense connective tissue and hyaline cartilage Found in the intervertebral discs; in areas where tendons are connected to bone; and in the symphysis pubis "Slipped disc" back injuries are fibrocartilage injuries

Gout, or gouty arthritis, cannot be diagnosed on the basis of hyperuricemia. What is the diagnostic criterion for gout?

Finding of monosodium urate crystals in the synovial fluid

An emergency room nurse receives a report that a client's Glasgow Coma Scale (GCS) is 3. The nurse prepares to care for a client with which of the following?

Flaccid motor response A score of 3 on the Glasgow Coma Scale indicates the lowest possible score in each of the three scoring categories (eye opening, motor response, and verbal response) and includes flaccid or no motor response, no verbal response and the inability to open the eyes.

The nurse is caring for a client with Guillain Barre' syndrome. Which of the following are expected clinical manifestations? Select all that apply.

Flaccid paralysis of extremities Pain Loss of tendon reflexes

A client with a history of a seizure disorder has been observed suddenly and repetitively patting his knee. After stopping this repetitive action, the client appears confused but is oriented to person and place but not time. What type of seizure did this client most likely experience?

Focal seizure with impairment to consciousness

syndesmosis

an immovable joint in which bones are joined by connective tissue (e.g., between the fibula and tibia at the ankle).

tension-type

analgesic relief provided by aspirin and NSAIDs

The parent of an infant who developed hydrocephalus while in utero is very concerned that the child will have significant intellectual dysfunction. The best response to the parent would be which of the following?

"Because the skull sutures are not fused there may be no brain damage." When hydrocephalus develops in utero, before the cranial sutures have fused, the head can swell and decrease intracranial pressure, thereby decreasing the amount of brain tissue that is compressed.

The parent of an infant who developed hydrocephalus while in utero is very concerned that the child will have significant intellectual dysfunction. The best response to the parent would be which of the following? a) "The cranial sutures are fused and decrease brain damage." b) "Infants never have symptoms from hydrocephalus." c) "Because the skull sutures are not fused there may be no brain damage." d) "Unfortunately, there usually is significant brain dysfunction."

"Because the skull sutures are not fused there may be no brain damage." When hydrocephalus develops in utero, before the cranial sutures have fused, the head can swell and decrease intracranial pressure, thereby decreasing the amount of brain tissue that is compressed.

The parents of a child diagnosed with rheumatic disease are shocked by the diagnosis and tell the nurse that they did not think children could acquire the disease. The best response would be:

"Children can be affected with almost all of the rheumatic diseases that occur in adults."

A patient is having difficulty with sleeping and has also been experiencing marital difficulties over the past couple of months. The patient tells the nurse at the physician's office that all this started after he had a car accident earlier that year. Which of the following would be the most important question for the nurse to ask?

"Did you sustain any injuries in the accident?" Postconcussion syndrome can interfere with daily living and also with relationships and can continue for months. The syndrome can include amnesia, insomnia, headache, difficulty concentrating, and irritability. In this situation, it would be very important to determine if the patient sustained a head injury to rule out postconcussion syndrome.

A patient is having difficulty with sleeping and has also been experiencing marital difficulties over the past couple of months. The patient tells the nurse at the physician's office that all this started after he had a car accident earlier that year. Which of the following would be the most important question for the nurse to ask? a) "What is your normal routine before bedtime?" b) "Did you go to the hospital following the accident?" c) "How long have you been married?" d) "Did you sustain any injuries in the accident?"

"Did you sustain any injuries in the accident?" Postconcussion syndrome can interfere with daily living and also with relationships and can continue for months. The syndrome can include amnesia, insomnia, headache, difficulty concentrating, and irritability. In this situation, it would be very important to determine if the patient sustained a head injury to rule out postconcussion syndrome.

A patient recently diagnosed with rheumatoid arthritis (RA) tells the nurse she is glad there is nothing "really wrong with her" but some joint swelling. Which of the following information should the nurse tell the patient about RA?

"Extra-articular manifestations may include anemia and deformities of the affected joints."

A patient has been diagnosed with rheumatoid arthritis (RA). Which of the following will the nurse tell the patient about this disorder's etiology?

"Genetic predisposition is very likely."

The unique clinical presentation of a 3 month-old infant in the emergency department leads the care team to suspect botulism. Which of the following assessment questions posed to the parents is likely to be most useful in the differential diagnosis?

"Have you ever given your child any honey or honey-containing products?"

A patient has just been diagnosed with acute glomerulonephritis. Which question should the nurse ask this client in attempting to establish a cause?

"Have you ever had any type of infection within the last two weeks?"

A female client with rheumatoid arthritis has taken high doses of aspirin for several years to control inflammatory pain. Which of the following statements leads the health care provider to suspect the client has developed ototoxicity?

"I've been getting dizzy and light-headed. I seem to have a constant ringing in my ear."

A patient with a recent diagnosis of renal failure that will require hemodialysis is being educated in the dietary management of the disease. Which of the patient's following statements shows an accurate understanding of this component of treatment?

"I've made a list of high-phosphate foods so that I can try to avoid them."

A client's primary care provider has ordered an oral glucose tolerance test (OGTT) as a screening measure for diabetes. Which of the following instructions should the client be given?

-"The lab tech will give you a sugar solution and then measure your blood sugar levels at specified intervals."

the greatest risk of developing Clostridium difficile colitis

antibitiotics

The physical therapy student is studying how the knee moves. The instructor knows the student understands the structure when she states which of the following?

"Ligaments join one bone to its articulating mate."

The nurse determines further teaching is necessary when a client diagnosed with osteoarthritis states:

"Men are more likely to have osteoarthritis than women."

A nurse is teaching a group of nursing students about the presentation of systemic lupus erythematosus (SLE). Which of the following statements does the nurse is likely to make?

"More women than men are affected by lupus."

A nurse educator is performing client education with a 51 year old man who has been recently diagnosed with chronic kidney disease. Which of the following statements by the client would the nurse most likely want to correct or clarify?

"My kidney problems increase my chance of developing high blood pressure or diabetes."

The nurse is caring for a patient diagnosed with osteoarthritis (OA). Which of the following does the nurse teach the patient about the disease?

"OA is a disease of the weight-bearing joints."

The faculty member knows the student understands the function of bone cells when the student states which of the following?

"Osteoblasts are bone-building cells that synthesize and secrete the organic matrix of bone."

A nursing instructor is teaching students about metabolic bone diseases. She realizes a need for further instruction when one student states which of the following?

"Osteopenia is a condition that is only common in the elderly."

A nursing faculty member is explaining the function of red bone marrow to students. Which of the following statements made by a student indicates that teaching was successful?

"Red bone marrow is the site of blood cell formation."

A client recently diagnosed with an injury to the articular cartilage of the knee asks the nurse how long it will take the injury to heal. Which of the following statements is the best response from the nurse?

"Regeneration of most cartilage is slow. These injuries can take a long time to heal."

A hospital client has been reluctant to accept morphine sulfate despite visible signs of pain. Upon questioning, the client reveals that he is afraid of becoming addicted to the drug. How can a member of the care team best respond to the client's concern?

-"There's only a minute chance that you will become addicted to these painkillers."

Glomerulonephritis is usually caused by

antigen-antibody complexes

Following a progressive onset of fatigue, aching and joint stiffness over the last two years, a 69 year-old male has recently been diagnosed with rheumatoid arthritis (RA). Which of the following teaching points should his primary care physician include during the office visit in which this diagnosis is communicated to the client?

"Steroids and anti-inflammatory drugs that I'll prescribe will likely bring some relief to your symptoms."

The endocrinologist knows the student understands calcitonin when the student states which of the following?

"The major stimulus for calcitonin synthesis and release is an increase in serum calcium."

A nurse is teaching a patient diagnosed with osteoarthritis about the joint capsule. Which of the following statements made by the patient indicates that teaching was successful?

"The synovial fluid in my joint capsule acts as a lubricant and helps the movement of the joint's articulating surfaces."

A 36 year-old female who has experienced diverse symptoms for several years has finally had her health problems attributed to scleroderma (systemic sclerosis), and has committed herself to learning as much about the disease as she can. Which of her following statements would her nurse want to correct or clarify?

"The worst part of this so far has been learning that there aren't any treatments for scleroderma."

While explaining to the parents of a 5-year old why the child's fractured femur has so much swelling and bruising, the nurse will include which of the following statements?

"This bone is hollowed out and the inside of the shaft produces marrow where red blood cells are formed."

A young child develops type 1A diabetes. The parents ask, "They tell us this is genetic. Does that mean our other children will get diabetes?" The best response by the health care provider would be:

-"This autoimmune disorder causes destruction of the beta cells, placing your children at high risk of developing diabetes."

A patient discharged from the hospital 5 days ago following a stroke has come to the emergency department with facial droop that progressed with hemiplegia and aphasia. The patient's spouse is extremely upset because the physician stated that the patient cannot receive thrombolytic medications to reestablish cerebral circulation and the spouse asks the nurse why. Which of the following is the nurse's most accurate response?

"Thrombolytics may cause cerebral hemorrhage." A previous stroke, occurring within 3 months of the administration of thrombolytics, significantly increases the risk of intracranial hemorrhage.

A patient discharged from the hospital 5 days ago following a stroke has come to the emergency department with facial droop that progressed with hemiplegia and aphasia. The patient's spouse is extremely upset because the physician stated that the patient cannot receive thrombolytic medications to reestablish cerebral circulation and the spouse asks the nurse why. Which of the following is the nurse's most accurate response? a) "All the brain tissue damage is already done." b) "Thrombolytics may cause cerebral hemorrhage." c) "The medications do not work with subsequent strokes." d) "The stroke is hemorrhagic, not thrombotic."

"Thrombolytics may cause cerebral hemorrhage." A previous stroke, occurring within 3 months of the administration of thrombolytics, significantly increases the risk of intracranial hemorrhage.

A nurse is caring for a patient with a fractured elbow. Which of the following instructions is important to give the patient to prevent cartilage degeneration while the elbow is immobilized?

"To prevent cartilage atrophy, slowly and gradually resume exercising."

The clinical nurse educator on a nephrology unit of a large, urban hospital is orientating recent nursing graduates to the unit. Which of the following teaching points about acute tubular necrosis (ATN) should the educator include in the orientation session?

"Trauma, burns, and major surgery are common precursors to ATN."

The spouse of a patient diagnosed with Alzheimer's disease asks the nurse why the patient often neglects to take a shower. The spouse states that the patient was always diligent with hygiene in the past; however, over the past few months that has not been the case. Which of the following is the nurse's best response? a) "The patient would be fine without showering." b) "You should remind the patient to shower." c) "The patient is experiencing a temporary relapse." d) "The patient just does not care anymore."

"You should remind the patient to shower." The patient should be reminded to shower because most likely he or she has difficulty remembering to do so. In the moderate stage of Alzheimer's disease, which can last for several years, it is not unusual for hygiene to be neglected because the person may just not remember if he or she did or did not shower. There is no information in the question to support the remaining responses.

A 20 year-old has been diagnosed with an astrocytic brain tumor located in the brainstem. Which of the following statements by the oncologist treating the client is most accurate? a) "Your prognosis will depend on whether we can surgically resect your tumor." b) "This is likely a result of a combination of heredity and lifestyle." c) "The major risk that you face is metastases to your lungs, liver or bones." d) "Our treatment plan will depend on whether your tumor is malignant or benign."

"Your prognosis will depend on whether we can surgically resect your tumor." The prognosis of people with pilocytic astrocytomas is influenced primarily by their location. The prognosis is usually better for people with surgically resectable tumors, such as those located in the cerebellar cortex, than for people with less accessible tumors, such as those involving the hypothalamus or brain stem. -Because of infiltration of brain tissue that prevents total resection, surgery rarely cures brain tumors. -The binary of malignant and benign is not used to characterize brain tumors and the etiology and substantive risk factors are largely unknown. - Brain tumors rarely metastasize outside the CNS.

stereogenesis

*Dorsal Column Medial Lemniscal Pathway* conveys information on size/shape of object without visual input

without aura

*Migraine* increased sensitivity to light & sound

without aura

*Migraine* may be accompanied by nausea & vomiting

without aura

*Migraine* pulsating, throbbing, unilateral headache that lasts 1-2 days and is aggravated by physical activity

with aura

*Migraine* same symptoms as without aura + visual or neurologic symptoms that precede the headache cluster headache

chickenpox (varicella)

*Postherpetic Neuralgia* herpes zoster (shingles) is caused by the same herpesvirus that causes...

A client with chronic low back pain presents to the clinic. In addition to a detailed pain assessment, which of the following questions would be appropriate to ask? Select all that apply.

-"Can you financially afford your medicine?" -"What kind of stressors are you experiencing?" -"Are you having trouble sleeping?"

A 44-year-old woman has sought care for the treatment of headaches that have been increasing in severity and frequency and has been subsequently diagnosed with migraines. Which of the following teaching points should her care provider emphasize?

-"It would be helpful for you to take control of your diet, sleep schedule, and stress levels."

A diabetic client presents to a clinic for routine visit. Blood work reveals a HbA1C of 11.0% (high)? Which response by the patient may account for this abnormal lab result?

-"My meter broke so I have not been checking my blood glucose levels for a while."

Which of the following clients are at risk for developing hypothyroidism? Select all that apply.

-A client who is prescribed amiodarone for frequent dysrhythmias -A client who has precancerous thyroid lesions who underwent ablation with radiation -A female experiencing an autoimmune disorder called thyroiditis -A bipolar client prescribed lithium carbonate

Which of the following clients may be experiencing the effects of neuropathic pain?

-A man with pain secondary to his poorly controlled diabetes

Which of the following clients would be considered to be exhibiting manifestations of "prediabetes"?

-A middle-aged overweight adult with a fasting plasma glucose level of 122 with follow-up OGTT of 189 mg/dL.

A lung cancer client with small cell carcinoma may secrete an excess of which hormone causing an ectopic form of Cushing syndrome due to a non pituitary tumor?

-ACTH

The signs and symptoms of abrupt cessation of pharmacologic glucocorticoids closely resemble those of:

-Addison disease

An otherwise healthy client has been referred to a pain clinic because she claims to experience exquisite pain from the friction of her clothes on her torso. This client is likely to be diagnosed with which of the following health problems?

-Allodynia

Which of the following residents of a long-term facility is exhibiting clinical manifestations of hypothyroidism?

-An 80-year-old woman who has uncharacteristically lost her appetite and often complains of feeling cold

Pain assessment is likely to be most challenging when providing care for which of the following older adult clients?

-An 87-year-old man with vascular dementia and other health problems like heart failure

Which of the following individuals displays the precursors to acromegaly?

-An adult with an excess of growth hormone due to an adenoma

A client with a spinal cord injury at T8 would likely retain normal motor and somatosensory function of her:

-Arms

A 33-year-old client has been admitted to the hospital for the treatment of Graves disease. Which of the following assessments should the client's care team prioritize?

-Assessment of the client's vision and oculomotor function

The results of a 44-year-old obese man's recent diagnostic workup have culminated in a new diagnosis of type 2 diabetes. Which of the following pathophysiologic processes underlies the client's new diagnosis?

-Beta cell exhaustion due to long-standing insulin resistance

A woman has cut her finger while dicing onions in the kitchen, causing her to drop her knife in pain. Which of the following components of this pain signal was transmitted by a third-order neuron? The neurons:

-Between the thalamus and the cortex

Which of the following comorbidities represent the greatest risk for the development of foot ulcers in a diabetic client? Select all that apply.

-Bilateral distal loss of pain sensation -Motor neuropathy related to improperly fitted shoes -Smoking history averaging 2 packs/day

A diabetic client presents to the clinic. He is concerned his lower legs are "feeling funny." Which of the following assessment findings lead the health care provider to suspect the client may have developed somatic neuropathy? Select all that apply.

-Both legs appear to be the same as far as numbness is involved. -Bilateral cool ankles and feet. -With eyes closed, the client cannot identify where the HCP is touching his feet.

A teenager has been in a car accident and experienced acceleration-deceleration head injury. Initially, the client was stable but then started to develop neurological signs/symptoms. The nurse caring for this client should be assessing for which type of possible complication?

-Brain contusions and hematomas

The nurse is caring for a spinal cord injury client. Assessment reveals shallow breath sounds with a very weak cough effort. The nurse correlates this with which level of injury on the spinal column?

-C5

A client with type 1 diabetes has started a new exercise routine. Knowing there may be some increase risks associated with exercise, the health care provider should encourage the client to:

-Carry a snack with carbs to prevent profound hypoglycemia

An elderly male client has been brought to the emergency department after experiencing stroke-like symptoms a few hours ago, and has been subsequently diagnosed with an ischemic stroke. The care team is eager to restore cerebral perfusion despite the likely death of the brain cells directly affected by the stroke. What is the rationale for the care team's emphasis on restoring circulation?

-Cells of the penumbra may be saved from hypoxic damage if blood flow is promptly restored.

The intracranial volume that is most capable of compensating for increasing intracranial pressure is the:

-Cerebrospinal fluid

A client who has had a spinal injury now has sensory changes on the distal forearm and fourth and fifth fingers. The nurse can predict that this client has experienced an injury to the: Select all that apply.

-Cervical (C) 8 -Thoracic (T) 1 dorsal root

A woman who is exhibiting clinical manifestations of a pituitary adenoma will likely complain of: Select all that apply.

-Cessation of menses -Unusual milk secretion unrelated to pregnancy -Infertility

A traumatic brain injury client has developed extreme cerebral edema. The nurse is monitoring the client closely for signs of brain herniation. Which clinical manifestations would correlate to upward herniation of the midbrain from the infratentorial compartment? Select all that apply.

-Deep coma -Respiratory rate of 8 with intermittent sighs -Bilateral small, fixed pupils

glomerulonephritis is usually caused by:

antigen-antibody complexes

A family brings a client to the emergency department with increasing lethargy and disorientation. They think the client had a seizure on the drive over to the hospital. The client has been sick with a "cold virus" for the last few days. On admission, the clients' temperature is 102°F. Which other clinical manifestations may lead to the diagnosis of encephalitis?

-Impaired neck flexion resulting from muscle spasm

The immune suppressive and anti-inflammatory effects of cortisol cause:

-Inhibition of prostaglandin synthesis

A newly diagnosed type 2 diabetic client has been prescribed metformin. When explaining the actions of this medication, the nurse should include which statement? This medication:

-Inhibits hepatic glucose production and increases the sensitivity of peripheral tissues to the actions of insulin

A client with excessive production of growth hormone level will likely exhibit which clinical manifestations? Select all that apply.

-Large hands and feet due to increased production of GH -Excess thirst and urination due to decreased glucose uptake -Difficulty chewing food

A diabetic client was visiting the endocrinologist for annual checkup. The client's blood work reveals an increased level of which lab result that reveals early signs of diabetic nephropathy?

-Microalbuminuria

The somatosensory system consists of three types of sensory neurons. The special somatic type of afferent sensory neurons has receptors that sense:

-Muscle position

Which of the following physiologic processes is a direct effect of the release of growth hormone by the anterior pituitary?

-Production of insulin-like growth factors (IGFs) by the liver

A client has started having uncontrolled seizures that are not responding to usual medications. Nursing working with the client must pay special attention to which of the following priority aspects of this clients care? Assessment of:

-Respiratory status and oxygen saturation

While teaching a class of nursing students about spinal cord injury, the instructor mentions that male SCI clients will be able to have a sexual response if their injury is at which level on the spinal column?

-S4

A recently injured (3 months ago) client with a spinal cord injury at T4 to T5 is experiencing a complication. He looks extremely ill. The nurse recognizes this as autonomic dysreflexia (autonomic hyperreflexia). His BP is 210/108; skin very pale; gooseflesh noted on arms. The priority nursing intervention would be to:

-Scan his bladder to make sure it is empty

Primary adrenal insufficiency is manifested by:

-Serum sodium level of 120 mmol/L (low) and blood glucose level of 48 mg/dL (low)

A client has developed the facial appearance that is characteristic of myxedema, along with an enlarged tongue, bradycardia, and voice changes. Which of the following treatment modalities is most likely to benefit this client?

-Synthetic preparations of T3 or T4

A client with a diagnosis of lung cancer has developed bone metastases resulting in severe and protracted pain. Which of the following assessment components should the nurse prioritize when assessing the client's pain?

-The client's subjective report of the character and severity of pain

A client with long-standing type 2 diabetes is surprised at his high blood sugar readings while recovering from an emergency surgery. Which of the following factors may have contributed to the client's inordinately elevated blood glucose levels?

-The stress of the event caused the release of cortisol.

A client with type 2 diabetes has routine lab work, which reveals elevated free fatty acids (FFA). The client asks, "Why is this significant?" The most accurate response would be: Select all that apply.

-This may increase the amount of triglyceride (a form of fat) stored in your liver or around your heart. -Your pancreas is affected by increased fat (lipotoxicity), which causes beta cell dysfunction, leading to the need for insulin.

A client is admitted in the ICU with diagnosis of hyperglycemic hyperosmolar state (HHS). The nurse caring for the client knows that the client's elevated serum osmolality has pulled water out of this brain cells based on which of the following assessment findings? Select all that apply.

-Weakness one side of the body -After the sole of the foot has been firmly stroked, the toes flex and flare out -Unable to respond verbally to questions -Uncontrollable twitching of a muscle group

Which of the following individuals is experiencing the effects of a primary endocrine disorder? A client:

-Who has low calcium levels because of the loss of his parathyroid gland

Which of the following clinical manifestations lead the health care worker to suspect the client is at the end-stage expression of hypothyroidism? A client: Select all that apply.

-Who takes analgesics for chronic pain that goes into a coma -Brought to the emergency department with hypothermia who presents with low serum sodium levels

A client with trigeminal neuralgia usually complains of excruciating pain. Which of the following activities may trigger an acute pain attack? Select all that may apply.

-Working in the office that has an air duct located directly overhead -Applying lipstick -Walking outside on a windy day

A 60-year-old male client has presented to his primary care provider to follow up with his ongoing treatment for peptic ulcer disease. What is the most likely goal of this client's pharmacologic treatment? Inhibiting gastric acid production Promoting hypertrophy of the gastric mucosa Increasing the rate of gastric emptying Increasing muscle tone of the cardiac sphincter

A

proprioceptive receptors

-muscle spindle fibers -Golgi tendon organs

A nurse is monitoring the intracranial pressure (ICP) of a client. The nurse would consider the client to have a normal reading when the results identify:

0 to 15 mm Hg The cranial cavity contains blood, CSF, and brain tissue. Each of these three volumes contributes to ICP, which is normally maintained within a range of 0 to 15 mm Hg when measured in the lateral ventricles.

Carpal Tunnel Treatments (2):

1. Avoid movements that causes pain. 2. Splinting and anti-inflammatory medication.

Mononeuropathies (4):

1. Caused by localized conditions such as trauma, compression, or infections that affect a single spinal nerve, plexus, or peripheral nerve trunk. 2. Fractured bones may lacerate or compress nerves. 3. Excessively tight tourniquets may injure nerves directly or produce ischemic injury. 4. Infections such as herpes zoster may affect a single segmental afferent nerve distribution.

Leg pain at rest

A nurse is assessing a male client who has advanced peripheral artery disease (PAD). Which of the following findings should the nurse expect?

Heat intolerance

A nurse is assessing an adolescent who has an exacerbation of Graves' disease. Which of the following findings should the nurse expect?

Following his annual influenza vaccination, a patient begins to feel achy, like he has developed the flu. An hour later, the patient is rushed to the emergency department. Diagnosis of Guillain-Barré syndrome was made based on which of the following assessment findings? Choose all that apply.

1.Rapid deterioration of respiratory status 2. Flaccid paralysis of limbs 3. Pale, cool, dry skin

A clinician is assessing the muscle tone of a patient who has been diagnosed with a lower motor neuron (LMN) lesion. Which of the following assessment findings is congruent with the patient's diagnosis? A. Hypotonia B. Spasticity C. Tetany D. Rigidity

A

-Inspect the electrode pads -Instruct the client not talk during the test

A nurse is assisting with obtaining an electrocardiogram (ECG) for a client who has atrial fibrillation. Which of the ollowing actions should the nurse take? (Select all that apply.)

A college baseball player has seen his season cut short by a rotator cuff injury. Rotator cuff injuries are frequent because of: A. the inherent instability of the shoulder. B. the absence of ligaments at the glenohumeral joint. C. the vulnerability of the shoulder menisci. D. the large mass of the humeral head.

A

The fourth heart sound (S4)

A nurse is auscultating a client's heart sounds and hears an extra heart sound before what should be considered the first heart sound S1. The nurse should document this finding as which of the following heart sounds?

Which of the following characteristics differentiates cartilage from bone? A. secretion of an extracellular matrix B. avascularity C. low tensile strength D. rapid healing

B

Increase in the heart rate from 88 to 110/min.

A nurse is caring for a client who returns to the nursing unit from the recovery room after a sigmoid colon resection for adenocarcinoma. The client had an episode of intraoperative bleeding. Which finding indicates to the nurse that the client may be developing hypovolemic shock?

The client develops a life-threatening situation.

A nurse is caring for a client who sustained a femur fracture in an automobile accident and is placed into skeletal traction. The nurse may remove the weights from the traction device if which of the following occurs?

The nurse is reviewing the health histories of four clients. Select the client most at risk for developing secondary osteoporosis.

A 60-year-old female taking prednisone for asthma

Which of the following disorders of neuromuscular function typically has the most rapid onset? A. Duchenne muscular dystrophy (DMD). B. Guillain-Barre syndrome C. Parkinson disease D. Myasthenia gravis

B

elastic cartilage

Elastic cartilage: contains some elastin in tis intercellular substance ~ found in areas where some flexibility is important such as the ear

Which of the following individuals who have recently presented to a hospital emergency department is displaying an injury that involves his or her fibrocartilage?

A 78 year-old man who has fallen and is suspected of having a "slipped disc" in his back.

A 5-year-old girl has been presented for care by her father due to her recent development of macules on her trunk, extremities, & mucous membranes. The child is mildly febrile but her primary symptom is extreme pruritus. What disorder of the skin should the clinician who is assessing the child first suspect? A. Varicella B. Lichen planus C. Rosacea D. Impetigo

A

In a child, the epiphyses and metaphysis are separated by which of the following?

A growth plate

Which of the following clients may be experiencing a sensory focal seizure that has sent an abnormal cortical discharge to the Autonomic Nervous System (ANS)? a) 44 year old patient complaining of constant movement and pain in the legs that gets worse when they try to sleep. b) 85 year old patient experiencing drooping of the right side of face and numbness in right arm and leg. c) 56 year old complaining of tingling sensations and has both an elevated pulse and BP. d) 22 year old complaining of a stiff neck and achiness, along with some nausea and vomiting.

56 year old complaining of tingling sensations and has both an elevated pulse and BP. Sensory symptoms correlate with the location of seizure activity on the contralateral side of the brain and may involve somatic sensory disturbance (tingling). With abnormal cortical discharge stimulating ANS, see tachycardia, diaphoresis, hypo- or hypertension, or papillary changes. Distractor A is associated with restless leg syndrome (RLS). Distractor B is associated with stroke (CVA). Distractor D is associated with meningitis.

Assess the perineal drainage and incision

59. A 71-yr-old patient had an abdominal-perineal resection for colon cancer. Which nursing action is most important to include in the plan of care for the day after surgery?

The regulation of cerebral blood flow is accomplished through both autoregulation and local regulation. This allows for the brain to meet its metabolic needs. What is the low parameter for arterial blood pressure before cerebral blood flow becomes severely compromised?

60 mm Hg If blood pressure falls below 60 mm Hg, cerebral blood flow becomes severely compromised, and if it rises above the upper limit of autoregulation, blood flow increases rapidly and overstretches the cerebral vessels.

severe intrapulmonary shunt

62-year-old female is admitted to the unit with respiratory failure following hip replacement surgery. She complains of shortness of breath and orthopnea. She has the following laboratory data available: PaO2 61 SaO2 0.90 SvO2 0.65 PaCO2 39 pH 7.35 FIO2 60%. Which condition is likely to be developing?

A couple has just learned that their newborn has been diagnosed with osteogenesis imperfecta, & they have responded by seeking out as much information as possible about their child's diagnosis. What should the clinician teach the couple about their child's health problem? A. "This is something that your child may have inherited from one or both of you." B. "This might have been caused by something you were exposed to during the early part of your pregnancy." C. "You'll have to be vigilant of your child's safety for the next few years, but the disease often resolves spntaneously." D. "With aggressive treatment, most children with osteogenesis imperfecta are cured within several months."

A

A family brings their father to his primary care physician for a checkup. Since their last visit, they note their dad has developed a tremor in his hands and feet. He also rolls his fingers like he has a marble in his hand. The primary physician suspects the onset of Parkinson disease when he notes which of the following abnormalities in the patient's gait? A. Slow to start walking and has difficulty when asked to "stop" suddenly B. Difficulty putting weight on soles of feet and tends to walk on tiptoes C. Hyperactive leg motions like he just can't stand still D. Takes large, exaggerated strides and swings arms/hands wildly

A

During class, the instructor asks students to explain the pathophysiology behind development of multiple sclerosis. Which student gave the most accurate description? A. The demyelination and subsequent degeneration of nerve fibers and decreased oligodendrocytes, which interfere with nerve conduction B. Muscle necrosis with resultant increase in fat/connective tissue replacing the muscle fibers C. Atherosclerotic destruction of circulation to the brain resulting in lactic acid buildup that affects nerve transmission D. Autoimmune disease where antibody loss of acetylcholine receptors at the neuromuscular junction causes decrease motor response

A

More complex patterns of movements, such as throwing a ball or picking up a fork, are controlled by the ____ cortex in the frontal lobe. A. Premotor B. Primary Motor C. Reflexive D. Supplementary

A

Premature osteoporosis is being seen increasingly in female athletes because of increased prevalence of: A. amenorrhea. B. high protein intake. C. abnormal body fat. D. osteoarthritis.

A

Systemic sclerosis (scleroderma) is an autoimmune disease of connective tissue characterized by: A. fibrosis. B. thin fragile skin. C. collagen deficiency. D. avascular necrosis.

A

The demyelination & degeneration of nerve fibers characteristic of multiple sclerosis is the result of: A. Decreased oligodendrocytes B. Corticospinal injuries C. Atherosclerotic destruction D. Oligodendrocytic infection

A

The most common early sysmptoms of systemic lupus erythematosus (SLE) include: A. arthralgia. B. tendon rupture. C. facial hair growth D. pyelonephritis.

A

The patient has a fractured tibia. After the cast is applied he is at high risk for compartment syndrome caused by: A. Inflammation B. Joint immobility C. Muscle atrophy D. Extremity elevation

A

Unlike disorders of the motor cortex & corticospinal (pyramidal) tract, lesions of the basal ganglia disrupt movement: A. Without causing paralysis B. Posture & muscle tone. C. & cortical responses. D. Of upper motor neurons.

A

When explaining acute pancreatitis to a newly diagnosed client, the nurse will emphasize the pathogenesis begins with an inflammatory process whereby: A. Activated pancreatic enzymes escape into surrounding tissues, causing autodigestion of pancreatic tissue. B. The pancreas is irreversibly damaged and will not recover to normal functioning (chronic). C. The pancreas will hypertrophy (enlarge) to the point of causing bowel obstruction. D. Stones will develop in the common bile duct, resulting in acute jaundice.

A

Which of the following disorders of the skeletal system occurs exclusively in older adults? A. Polymyalgia rheumatica. B. Psoriatic arthritis C. Reiter syndrome D. Ankylosing spondylitis

A

Which of the following is the primary role of fibrocartilage? A. absorption of physical shock B. provision of flexibility C. facilitation of long bone growth D. tissue hydration

A

Which of the following signs and symptoms is most indicative of Meniere disease? A. Rotary vertigo and tinnitus B. Nausea and vomiting C. Progressive hearing loss and frequent falls D. Otalgia and recurrent otitis media

A

Which of the following statements most accurately describes the anatomy & physiology of the bone marrow? A. hematopoiesis takes place in red bone marrow. B. yellow bone marrow predominates in infants. C. by adulthood, all red bone marrow has been replaced by yellow bone marrow. D. yellow bone marrow is hematopoietically active in infants, but not in adults.

A

Following his annual influenza vaccination, a patient begins to feel achy, like he has developed the flu. An hour later, the patient is rushed to the emergency department. Diagnosis of Guillain-Barré syndrome was made based on which of the following assessment findings? Choose all that apply. A. Rapid deterioration of respiratory status B. Lack of any physical pain C. Flaccid paralysis of limbs D. BP 90/62 E. Pale, cool, dry skin

A (weak diapragm), C, D

Lifelong anticoagulant therapy is needed after mechanical valve replacement.

A 21-yr-old woman is scheduled for percutaneous transluminal balloon valvuloplasty to treat mitral stenosis. Which information should the nurse include when explaining the advantages of valvuloplasty over valve replacement to the patient?

Respiratory acidosis

A client is admitted to the emergency room with a respiratory rate of seven per min. Arterial blood gases (ABG) reveal the following values. Which of the following is an appropriate analysis of the ABGs? pH 7.22 PaCO2 68 mm Hg Base excess -2 PaO2 78 mm HG Saturation 80% Bicarbonate 28 mEq/L

Systolic blood pressure is increased

A nurse is assessing a client who is receiving dopamine IV to treat left ventricular failure. Which of the following findings should indicate to the nurse that the medication is having a therapeutic effect?

Increased bruising

A nurse is assessing a client who was admitted with Cushing's syndrome. Which of the following manifestations should the nurse expect?

Inspect mouth for signs of inhalation injuries

A group of college students was attending a weekend football rally when one of the students stumbled and fell into the bonfire. Although several friends quickly intervened, the client sustained partial-thickness burns to both lower legs, chest, and both forearms. Which of the following is the priority nursing action when the client is brought into the emergency room?

The Kaccha (holy underwear) should be kept on one leg throughout labor and delivery

A labor and delivery nurse is working with a pregnant Sikh woman. The nurse understands that based on religious preference, she should ensure which of the following during labor?

Urine specific gravity 1.015

A nurse administers desmopressin to a client who has a diagnosis of diabetes insipidus. The nurse recognizes that which the following laboratory findings indicate a therapeutic effect of the medication?

"Large incisions will be made in the eschar to improve circulation."

A nurse in a burn treatment center is caring for a client who is admitted with severe burns to both lower extremities and is pending an escharotomy. The client's spouse asks the nurse what the procedure entails. Which of the following nursing statements is appropriate?

Obtain a prescription for the appropriate anti-venom

A nurse in an emergency department is assessing a client who was bitten on the left leg by a poisonous snake. The client has placed elastic bandages snuggly above and below the bite marks and is in no apparent distress. Which of the following actions should the nurse take?

Hemorrhagic stroke

A nurse in an emergency department is caring for a client who had a seizure and became unresponsive after stating she had a sudden, severe headache and vomiting. The client's vital signs are as follows: blood pressure of 198/110 mm Hg, pulse of 82/min, respirations of 24/min, and a temperature of 38.2° C (100.8° F). Which of the following neurologic disorders should the nurse suspect?

Administer oxygen via nasal cannula.

A nurse in an emergency department is caring for a client who has a sucking chest wound resulting from a gunshot. The client has a blood pressure of 100/60 mm Hg, a weak pulse rate of 118/min, and a respiratory rate of 40/min. Which of the following actions should the nurse take?

54 percent

A nurse in an emergency department is caring for a client who has burns on the front and back of both his legs and arms. Using the rule of nines the nurse should document burns to which percentage of the client's total body surface area (TBSA)?

Inspect the mouth for signs of inhalation injuries.

A nurse in an emergency room is caring a the client who sustained partial-thickness burns to both lower legs, chest, face, and both forearms. Which of the following is the priority action the nurse should take?

Airway obstruction

A nurse in the emergency department is caring for a client who has extensive partial and full-thickness burns of the head, neck, and chest. While planning the client's care, the nurse should identify which of the following risks as the priority for assessment and intervention?

anterior: torso, whole left arm, right forearm posterior: whole left arm, right forearm 31.5%

A nurse is admitting a client who has sustained severe burn injuries in a grease fire. The nurse shades in a diagram indicating the burned surface areas. Using the Rule of Nines, the nurse should estimate that the client has burned what percentage of body surface area? (Round the answer to the nearest tenth. Use a leading zero if it applies. Do not use a trailing zero.)

Elevated temperature

A nurse is admitting a young adult client who has suspected bacterial meningitis. The nurse should closely monitor the client for increased intracranial pressure (ICP) as indicated by which of the following findings?

Pain

A nurse is assessing a child who is in sickle cell crisis. Which of the following findings should the nurse expect?

-Erythema -Throbbing -Warmth at insertion site -Streak formation

A nurse is assessing a client who has a peripheral IV with a continuous infusion. Which of the following findings is a manifestation of phlebitis? (Select all that apply.)

Movement of the trachea toward the unaffected side

A nurse is assessing a client who has a pneumothorax with a chest tube in place. For which of the following findings should the nurse notify the provider?

Tachycardia

A nurse is assessing a client who has diabetes insipidus. Which of the following findings is a manifestation of this diagnosis?

Dehydration

A nurse is assessing a client who has diabetes insipidus. Which of the following findings should the nurse expect?

Urine specific gravity 1.002

A nurse is assessing a client who has diabetes insipidus. Which of the following findings should the nurse expect?

Hyperpigmentation

A nurse is assessing a client who is admitted for elective surgery and has a history of Addison's disease. Which of the following findings should the nurse expect?

1370 mL

A nurse is calculating the output of a client at the end of the shift. The nurse notes the following: client voided 400 mL at 1100 and 350 mL at 1430. The closed chest drainage system was previously marked at 155 mL and is now at 175 mL. The NG tube has 575 mL in drainage container, and 25 mL is emptied out of the Jackson-Pratt drainage tube. How many mL should the nurse record in the medical record as the client's output?

Teach the parents about cortisol replacement therapy

A nurse is caring for a child who has Addison's disease. Which of the following actions should the nurse take?

Defibrillation

A nurse is caring for a client who develops a ventricular fibrillation rhythm. The client is unresponsive, pulseless, and apneic. Which of the following actions is the nurse's priority?

To prevent fluid from accumulating in the wound

A nurse is caring for a client who has a Jackson-Pratt (JP) drain in place after surgery for an open reduction and internal fixation. The nurse should understand that the JP drain was placed for which of the following purposes?

No fluctuations in the water seal chamber

A nurse is caring for a client who has a chest tube in place to a closed chest drainage system. Which of the following findings should indicate to the nurse that the client's lung has re-expanded?

Restlessness

A nurse is caring for a client who has a traumatic brain injury. Which of the following findings should the nurse identify as an indication of increased intracranial pressure (ICP)?

Mannitol 25%

A nurse is caring for a client who has a traumatic head injury and is exhibiting signs of increasing intracranial pressure. Which of the following medications should the nurse plan to administer?

pH 7.26, HCO3 14, PaCO2 30

A nurse is caring for a client who has acute kidney injury (AKI). Which of the following arterial blood gas values would the nurse expect this client to have?

Administer antibiotics to the client.

A nurse is caring for a client who has acute osteomyelitis. Which of the following interventions is the nurse's priority?

Coffee-ground drainage

A nurse is caring for a client who has an active upper gastrointestinal bleed. After inserting a NG tube into the client, which of the following findings should the nurse anticipate?

-Confusion -Nonreactive dilated pupils -Slurred speech

A nurse is caring for a client who has an intracranial pressure (ICP) reading of 40 mm Hg. Which assessment should the nurse recognize as a late sign of ICP? (Select all that apply.)

Apical and radial pulses

A nurse is caring for a client who has been admitted with atrial fibrillation. Which of the following assessments will provide provide valuable client data to the nurse in relation to this diagnosis

"Warfarin takes several days to work, so the IV heparin will be used until the warfarin reaches a therapeutic level."

A nurse is caring for a client who has deep vein thrombosis and has been on heparin continuous infusion for 5 days. The provider prescribes warfarin PO without discontinuing the heparin. The client asks the nurse why both anticoagulants are necessary. Which of the following statements should the nurse make?

A decrease in urine output

A nurse is caring for a client who has diabetes insipidus and is receiving vasopressin. The nurse should identify which of the following findings as an indication that the medication is effective?

Assess orthostatic blood pressure

A nurse is caring for a client who has gastrointestinal bleeding. Which of the following actions should the nurse take first?

Paraplegia

A nurse is caring for a client who has had a spinal cord injury at the level of the T2-T3 vertebrae. When planning care, the nurse should anticipate which of the following types of disability?

Elevate the head of the bed 20&deg.

A nurse is caring for a client who has increased intracranial pressure. Which of the following interventions should the nurse take?

Different apical and radial pulses.

A nurse is caring for a client who has pericarditis and reports feeling a new onset of palpitations and shortness of breath. Which of the following assessments should indicate to the nurse that the client may have developed atrial fibrillation?

Place the extremity in a dependent position

A nurse is caring for a client who is brought into the emergency department immediately following a snake bite to his forearm. The client suspects the snake to be venomous. Which of the following interventions should the nurse take?

Mannitol 25%

A nurse is caring for a client who is experiencing Cushing's Triad following a subdural hematoma. Which of the following medications should the nurse plan to administer?

"The Coumadin takes several days to work, so the IV heparin will be used until the Coumadin reaches a therapeutic level."

A nurse is caring for a client who is hospitalized with deep vein thrombosis and has been on IV heparin for 5 days. The provider prescribes oral warfarin (Coumadin) without discontinuing the heparin. The client asks the nurse why both anticoagulants are necessary. Which of the following is an appropriate nursing response?

Vitamin K

A nurse is caring for a client who is on warfarin therapy for atrial fibrillation. The client's INR is 5.2. Which of the following medications should the nurse prepare to administer?

Massaging her legs

A nurse is caring for a client who is postoperative and is at risk for developing venous thromboembolism (VTE). The nurse should instruct the client to avoid which of the following unsafe actions?

Crossing the legs

A nurse is caring for a client who is postoperative and is at risk for development of thrombophlebitis. The nurse should instruct the client to avoid which of the following unsafe actions while sitting in a chair?

Prothrombin time (PT)

A nurse is caring for a client who is prescribed warfarin therapy for an artificial heart valve. Which of the following laboratory values should the nurse monitor for a therapeutic effect of warfarin?

Pacemaker spikes before each QRS complex

A nurse is caring for a client with a ventricular pacemaker who is on ECG monitoring. The nurse understands that the pacemaker is functioning properly when which of the following appears on the monitor strip?

Elevate head of bed to 30 to 45 degrees

A nurse is caring for a patient who is orally intubated and receiving mechanical ventilation. To decrease the risk for ventilator-associated pneumonia, which action will the nurse include in the plan of care?

Perform a neurovascular assessment

A nurse is caring for an adolescent client who has a newly applied fiberglass cast for a fractured tibia. Which of the following is the priority action for the nurse to take?

Peripherally inserted central catheter

A nurse is completing discharge planning for a client who has bacterial endocarditis. The client will need to receive 12 weeks of antibiotic therapy. Which of the following venous access devices should the nurse identify as appropriate for the client?

Talk with the client during wound care

A nurse is developing a plan of care for a client who is rehabilitating from major burns. Which of the following interventions should the nurse include to provide emotional support?

"Offer fluids to your child multiple times every day."

A nurse is discharging a child who has sickle cell anemia after an acute crisis episode. Which of the following instructions should the nurse include in the teaching?

May expect to have swelling of the legs

A nurse is educating a client who is being treated for metastatic colorectal cancer with bevacizumab (Avastin). Which of the following information should the nurse include?

A client who has a piece of wood punctured into the chest wall and has an audible hissing sound coming from the wound site

A nurse is helping to triage a group of clients at a mass casualty incident who were involved in an explosion at a local factory. Which of the following clients should the nurse tag to be the priority for care?

"I will take my medications at the first sign of an attack."

A nurse is instructing a client who has a new diagnosis of Raynaud's disease about preventing the onset of manifestations. Which of the following client statements should indicate to the nurse the need for additional teaching?

-Headache -Slurred speech -Pupillary changes -Disorientation

A nurse is monitoring a client who has a leaking cerebral aneurysm. Which of the following manifestations should indicate to the nurse the client is experiencing an increase in intracranial pressure (ICP)? (Select all that apply.)

Respirations are unlabored

A nurse is monitoring a client who received epinephrine for angioedema after a first dose of losartan. Which of the following data indicates a therapeutic response to the epinephrine?

Continue to monitor the client's respiratory status.

A nurse is observing the closed chest drainage system of a client who is 24 hr post thoracotomy. The nurse notes slow, steady bubbling in the suction control chamber. Which of the following actions should the nurse take?

A client experiencing a tension pneumothorax

A nurse is performing triage for a group of clients following a mass casualty incident (MCI). Which of the following clients should the nurse plan to care for first?

Check urine specific gravity

A nurse is planning care for a client who has a new diagnosis of diabetes insipidus. Which of the following interventions should the nurse include in the plan of care?

To reduce the risk of stroke in clients who have atrial fibrillation

A nurse is preparing to administer dabigatran to a client who has atrial fibrillation. The nurse should explain that the purpose of this medication is which of the following?

Administer a saline solution after injection.

A nurse is preparing to administer phenytoin IV to a client who has a seizure disorder. Which of the following actions should the nurse plan to take?

"We will give our child pancreatic enzymes with snacks and meals."

A nurse is providing discharge teaching about nutrition to the parents of a child who has cystic fibrosis (CF). Which of the following responses by the parents indicates an understanding of the teaching?

Levothyroxine

A nurse is providing teaching to a client who has a new diagnosis of hypothyroidism. On which of the following medications should the nurse prepare to instruct the client?

"I'll wrap a warm, wet towel around my right calf every 4 hours."

A nurse is providing teaching to a client who has a prescription for heat therapy for treatment of cellulitis of the right lower leg. Which of the following client statements indicates an understanding of the teaching?

It facilitates the client's deep breathing.

A nurse is providing teaching to a client who is postoperative following coronary artery bypass graft (CABG) surgery and is receiving opioid medications to manage discomfort. Aside from managing pain, which of the following desired effects of medications should the nurse identify as most important for the client's recovery?

pH 7.25, HCO3- 19 mEq/L, PaCO2 30 mm Hg

A nurse is reviewing the arterial blood gas values of a client who has chronic kidney disease. Which of the following sets of values should the nurse expect?

The client who has an increased magnesium level

A nurse is reviewing the laboratory results for four clients. The nurse should recognize that which of the following clients has a manifestation of primary hyperparathyroidism?

A client who has a phosphate of 5.7 mg/dL

A nurse is reviewing the laboratory results for four clients. The nurse should recognize which of the following clients has a manifestation of hypoparathyroidism?

Urine specific gravity 1.035

A nurse is reviewing the laboratory results of a client who has fluid volume deficit. The nurse would expect which of the following findings?

When assessing the motor system, on which of the following will the nurse focus? Select all that apply.

Body position Presence of involuntary muscle movement Muscle strength Coordination

pre-oxygenation prior to suctioning.

A nurse is suctioning the endotracheal tube of a client who is on a ventilator. The client's heart rate increases from 86/min to 110/min and becomes irregular. The nurse should know that the client requires

Perform pre-oxygenation prior to suctioning

A nurse is suctioning the endotracheal tube of a client who is on a ventilator. The client's heart rate increases from 86/min to 110/min and becomes irregular. Which of the following actions should the nurse take?

Immobilize the affected extremity with a splint.

A nurse is teaching a group of clients about emergency care for a snake bite. Which of the following information should the nurse include in the teaching?

the first wave before the QRS

A nurse is teaching a newly licensed nurse about evaluating a cardiac rhythm. Which of the following options should the nurse identify as the P wave in the ECG complex? (You will find hot spots to select in the artwork below. Select only the hot spot that corresponds to your answer.)

"Move objects away from the client."

A nurse is teaching the family of a client who has a new diagnosis of epilepsy about actions to take if the client experiences a seizure. Which of the following instructions should the nurse include in the teaching?

"My child will take the enzymes to help digest the fat in foods."

A nurse is teaching the mother of a child who has cystic fibrosis and has a prescription for pancreatic enzymes three times per day. Which of the following statements indicates that the mother understands the teaching?

"Monitor for compression fractures of the back and neck."

A nurse is teaching with a client about taking high doses of oral glucocorticoids for an extended period of time to treat rheumatoid arthritis. Which of the following instructions should the nurse include in the teaching?

An unconscious adult client who has a sucking chest wound, respirations of 38/min, and capillary refill of < 2 seconds

A nurse is the triage officer in the emergency department when four clients arrive following a factory explosion. Which of the following clients should the nurse care for first?

Focus on providing care that prevents life-threatening emergencies

A nurse is working with a limited staff because of a severe storm in the area. The facility incident commander has initiated disaster protocols. Which of the following actions should the nurse take?

Irregular pulsations

A nurse on a telemetry unit is caring for a client who has premature ventricular contractions (PVCs). While sitting in a chair, the client feeling reports feeling lightheaded. If the client is having PVCs, which of the following findings should the nurse expect when auscultating the client's apical pulse?

Restlessness

A nurse suspects that a client admitted for treatment of bacterial meningitis is experiencing increased intracranial pressure (ICP). Which of the following assessment findings by the nurse supports this suspicion?

Hand hygiene, oral care, and head-of-bed elevated

A nurse understands there are three care actions to decrease ventilator-associated pneumonia called a "ventilator bundle". Which of the following three nursing actions are included in the clients' care bundle?

Restrict sodium intake

A nursing is providing dietary teaching for a client who has Cushing's disease. Which of the following recommendations should nurse include in the teaching?

Statins

A patient in the outpatient clinic has a new diagnosis of peripheral artery disease (PAD). Which group of drugs will the nurse plan to include when teaching about PAD management?

monitor the tumor status after surgery.

A patient preparing to undergo a colon resection for cancer of the colon asks about the elevated carcinoembryonic antigen (CEA) test result. The nurse explains that the test is used to:

patients should the nurse observe most closely for the signs and symptoms of paralytic ileus?

A patient who is postoperative day 1 following gall bladder surgery (due to anesthesia)

A soccer player has been diagnosed with a brain contusion after being injured in a game. The best explanation of the injury by the nurse would be:

Bruising on the surface of the brain occurred. Contusions represent bruising on the surface of the brain, and lacerations are a tearing of brain tissue.

Which of the following individuals likely faces the greatest risk for the development of chronic kidney disease?

A patient with a recent diagnosis of type 2 diabetes who does not monitor his blood sugars or control his diet

"It is caused by the lack of production of aldosterone by the adrenal gland."

A staff nurse is teaching a client who has Addison's disease about the disease process. The client asks the nurse what causes Addison's disease. Which of the following responses should the nurse make?

A 26 year-old female is resting after a one-minute episode during which she lost consciousness while her muscles contracted and extremities extended. This was followed by rhythmic contraction and relaxation of her extremities. On regaining consciousness, she found herself to have been incontinent of urine. What has the woman most likely experienced?

A tonic-clonic seizure. A tonic-clonic seizure often begins with tonic contraction of the muscles with extension of the extremities and immediate loss of consciousness. Incontinence of bladder and bowel is common. Cyanosis may occur from contraction of airway and respiratory muscles. The tonic phase is followed by the clonic phase, which involves rhythmic bilateral contraction and relaxation of the extremities.

A college baseball player has seen his season cut short by a rotator cuff injury. Rotator cuff injuries are frequent because of: A) the inherent instability of the shoulder. B) the absence of ligaments at the glenohumeral joint. C) the vulnerability of the shoulder menisci. D) the large mass of the humeral head.

A) the inherent instability of the shoulder.

A 26 year-old female is resting after a one-minute episode during which she lost consciousness while her muscles contracted and extremities extended. This was followed by rhythmic contraction and relaxation of her extremities. On regaining consciousness, she found herself to have been incontinent of urine. What has the woman most likely experienced? a) An absence seizure b) A myoclonic seizure c) A complex partial seizure. d) A tonic-clonic seizure.

A tonic-clonic seizure. A tonic-clonic seizure often begins with tonic contraction of the muscles with extension of the extremities and immediate loss of consciousness. Incontinence of bladder and bowel is common. Cyanosis may occur from contraction of airway and respiratory muscles. The tonic phase is followed by the clonic phase, which involves rhythmic bilateral contraction and relaxation of the extremities. -A myoclonic seizure involves bilateral jerking of muscles, generalized or confined to the face, trunk, or one or more extremities. -Absence seizures are nonconvulsive and -complex partial seizures are accompanied by automatisms (aimless and apparently undirected behavior that is not under conscious control and is performed without conscious knowledge).

Administer oxygen via nasal cannula

A triage nurse in an emergency department is caring for a client who has a gunshot wound to the right side of her chest. The nurse notes a thick dressing on the chest and a sucking noise coming from the wound. The client has a blood pressure of 100/60 mm Hg, a weak pulse rate of 118/min, and a respiratory rate of 40/min. Which of the following actions should the nurse take initially?

A patient who experienced a traumatic head injury from a severe blow to the back of his head now lives with numerous function deficits, including an inability to maintain steady posture while he is in a standing position, although he is steadier when walking. Which of the following disorders most likely resulted from his injury?

A vestibulocerebellar disorder

A patient who experienced a traumatic head injury from a severe blow to the back of his head now lives with numerous function deficits, including an inability to maintain steady posture while he is in a standing position, although he is steadier when walking. Which of the following disorders most likely resulted from his injury? Cerebellar dystaxia Cerebellar tremor A lower motor neuron lesion A vestibulocerebellar disorder

A vestibulocerebellar disorder

Which of the following is the primary role of fibrocartilage? A) Absorption of physical shock B) Provision of flexibility C) Facilitation of long bone growth D) Tissue hydration

A) Absorption of physical shock

Which of the following statements most accurately describes the anatomy and physiology of the bone marrow? A) Hematopoiesis takes place in red bone marrow. B) Yellow bone marrow predominates in infants. C) By adulthood, all red bone marrow has been replaced by yellow bone marrow. D) Yellow bone marrow is hematopoietically active in infants, but not in adults.

A) Hematopoiesis takes place in red bone marrow.

Following a lengthy series of diagnostic tests, a patients chronic hip pain has been attributed to advanced osteonecrosis. What treatment is this patient most likely to require? A) Joint replacement surgery B) Intravenous antibiotics C) Injections of corticosteroids into the synovial space D) Transfusion of packed red blood cells

A) Joint replacement surgery

Which of the following disorders of the skeletal system occurs exclusively in older adults? A) Polymyalgia rheumatica B) Psoriatic arthritis C) Reiter syndrome D) Ankylosing spondylitis

A) Polymyalgia rheumatica

Which of the following joints is classified as a synarthrosis? A) The joint between two vertebrae B) The joint between the femur and the pelvis C) The joint between the humerus and the radius and ulna D) An interphalangeal joint of the hand (knuckle)

A) The joint between two vertebrae

A couple has just learned that their newborn infant has been diagnosed with osteogenesis imperfecta, and they have responded by seeking out as much information as possible about their childs diagnosis. What should the clinician teach the couple about their childs health problem? A) This is something that your child may have inherited from one or both or you. B) This might have been caused by something you were exposed to during the early part of your pregnancy. C) Youll have to be vigilant of your childs safety for the next few years, but the disease often resolves spontaneously. D) With aggressive treatment, most children with osteogenesis imperfecta are cured within several months.

A) This is something that your child may have inherited from one or both or you.

Developmental dysplasia of the hip (DDH), formerly known as congenital hip dislocation, is suspected when an infant has: A) gluteal fold asymmetry. B) lengthening of the thigh. C) joint capsule tightness. D) delay of knee crawling.

A) gluteal fold asymmetry.

The patient has a fractured tibia. After the cast is applied he is at high risk for compartment syndrome caused by: A) inflammation. B) joint immobility. C) muscle atrophy. D) extremity elevation.

A) inflammation.

In growing bone, severe vitamin C deficiency slows bone growth by impairing: A) organic matrix formation. B) calcification of new bone. C) growth plate separation. D) widening of the cortex.

A) organic matrix formation.

A feature of rheumatoid arthritis that differentiates it from other forms of inflammatory arthritis is the development of: A) pannus tissue. B) tophi deposits. C) subluxations. D) autoantibodies.

A) pannus tissue.

Despite differences in onset, involvement, and symptomatology, all of the spondyloarthropathies involve: A) sacroiliitis. B) calcinosis. C) excessive bone turnover. D) autoimmune etiology.

A) sacroiliitis.

The distinguishing characteristic of chronic osteomyelitis is the presence of: A) sequestrum bone. B) abscess formation. C) severe bone pain. D) external drainage.

A) sequestrum bone.

In some diarthrotic joints, the synovial membrane forms sacs filled with synovial fluid that prevent friction on a tendon. These sacs are known as which of the following?

Bursae

A child in gymnastics class has fallen off the balance beam and hurt her ankle. X-rays are negative for fracture, so the health care provider has diagnosed a severe sprain. Which of the following treatment measures should be taught to the family and child? Choose all that apply. A. Immobilization B. Elevate the ankle on pillows C. Apply ice packs to the ankle D. Perform active and passive range-of-motion exercises daily

A, B, C REST sprain or strain the same treatment

A client with hepatitis B asks the nurse, "How did I get this hepatitis?" The nurse responds that the mode of transmission is predominantly by: (Select all that apply.) A. Intravenous drug use B. Fecal-oral route C. Unprotected sexual intercourse D. Sharing oral secretions by kissing E. Inhalation of airborne droplets

A, C

A new mother brings her infant to the clinic reporting that the child is not sleeping or eating much. Upon assessment, the health care provider notes that the infant's ear canal is reddened with a bulging tympanic membrane. Which other data collected would lead to the diagnosis of acute otitis media (AOM)? Select all that apply. A. "Yes, he has been pulling at his ear." B. "We like to throw him up in the air hoping any water in his ear will drain." C. "He's been very irritable and fussy the past couple of days." D. "When I dropped a pan on the floor, he jumped." E. "He jabbers all the time usually."

A, C

A student nurse is taking a test on the endocrine system. From the following list of clinical manifestations, she needs to select the ones she would see in hypothyroidism. Which answers should she select? Select all that apply. A. Weight gain despite loss of appetite B. Nervousness with fine muscle tremors C. Coarse brittle hair D. Heat intolerance E. Puffy face with swollen eyelids

A, C, E

A client has visited the health care provider following several days of nausea/vomiting and abdominal pain. The provider thinks the client may have Helicobacter pylori (H. pylori) infection. As part of the education provided, the client should be informed that which of the following complications can occur if this infection is not eradicated? Select all that apply. GI bleeding due to peptic ulcer formation Failure to thrive due to malabsorption Pyloric stenosis due to inability to empty the stomach Gastric cancer due to metaplasia changes in the cells

A, D

A health care provider suspects a female patient (who has had vague complaints over the past several months) may be developing systemic lupus erythematosus (SLE). Which clinical manifestations would correlate with this diagnosis? Select all that apply. A. Arthralgia B. Oral ulcer C. Facial hair growth D. Uncontrolled hypertension due to pyelonephritis E. Chest pain that increases with each deep breath

A,B, E can develop due to pleuritis

Which of the following lab results may be associated with metastatic bone disease? Choose all that apply. A. Elevated alkaline phosphatase B. Decreased creatinine levels C. High serum calcium levels D. Lower serum phosphate levels

A,C

Which of the following clinical manifestations would support the medical diagnosis of Cushing syndrome? Select all that apply. A. Excessive facial hair growth B. Muscle hypertrophy C. Blood glucose level in 200 mg/dL range D. "Buffalo hump" on back E. Blood pressure reading less than 90/70

A,C,D

Following a fracture, the nurse is educating the patient and his family about bone healing. The nurse begins by stating: A. "In order to initiate the cellular events essential to bone healing, within a day or so, your body will develop a blood clot at the fracture site." B. "The first thing that will happen is your body will form a soft tissue callus around the fracture site." C. "Your body will first absorb any excess bone fragments that occurred as a result of the break." D. "Just like on your hands after hard work, your body will begin the healing process by forming a callus at the fracture site."

A- first thing hematoma-blood clot formation

The ulcerative colitis client should be assessed by the health care provider for which of the following clinical manifestations? Choose all that apply. Persistent diarrhea Steatorrhea Stool containing blood (hematochezia) External hemorrhoids Prolapsed colon

A: persistent diarrhea C stool containing blood (hematochezia)

neurotransmitter needed for muscle movement

ACh

What best explains symptoms of ALS including dysphagia, muscle weakness and spasticity, and dysphonia?

ALS is caused by both an upper motor neuron and lower motor neuron disturbance.

To maintain hematocrit levels in people with kidney failure, the nurse should be prepared to:

Administer a subcutaneous injection of recombinant human erythropoietin

bursa

Bursae: synovial membrane sacs that are not part of the joint and form in some diarthrotic joints

Myasthenia Gravis, an autoimmune disease, is characterized by muscle weakness caused by antibody mediated destruction of

Acetylcholine receptors

myasthenia gravis is characterized by muscle weakness caused by antibody-mediated destruction of:

Acetylcholine receptors

The family of an older adult reports increasing inability to perform basic activities of daily living. After evaluation, the client is diagnosed with Alzheimer's disease. What intervention will be implemented to slow cognitive decline?

Acetylcholinesterase inhibitors

The family of an older adult reports increasing inability to perform basic activities of daily living. After evaluation, the client is diagnosed with Alzheimer's disease. What intervention will be implemented to slow cognitive decline? a) Psychotherapy b) Acetylcholinesterase inhibitors c) Antioxidant therapy d) Lipid-lowering agents

Acetylcholinesterase inhibitors Cognitive function in Alzheimer's disease (AD) can be enhanced by the use of medications. The acetylcholinsterase inhibitors donepezil, rivastigmine, and galantamine all increase concentration of acetylcholine in the cerebral cortex. -There has been no demonstrated improvement of cognitive function with use of lipid-lowering statins or antioxidant nutritional supplement therapy. -Psychotherapy is appropriate for depression.

Which of the following disorders of the skin is most likely to respond to treatment with systemic antibiotics?

Acne vulgaris

Which one of the following skin disorders seen in elderly persons is considered a premalignant lesion?

Actinic keratosis

In the adult, red blood cell production decreases and red marrow is gradually replaced by yellow bone marrow. Yellow marrow is composed largely of which of the following?

Adipose cells

We have both red and yellow bone marrow in our bodies. What is yellow bone marrow largely composed of?

Adipose cells

A client is brought to the emergency department and is diagnosed with an ischemic stroke confirmed by CT scan. The most important treatment for this client would be to:

Administer IV tissue-type plasmin activator (tPA)

A client is brought to the emergency department and is diagnosed with an ischemic stroke confirmed by CT scan. The most important treatment for this client would be to: a) Prepare the client for emergency surgery b) Monitor vital signs closely for improvement c) Administer IV tissue-type plasmin activator (tPA) d) Administer analgesics for the relief of pain

Administer IV tissue-type plasmin activator (tPA) tPA administration is the treatment of choice for an ischemic stroke after confirmation that it is not a hemorrhagic stroke. Monitor vital signs and provide pain relief to prevent complications

bursitis

Bursitis: inflammation of the bursae ~ most common in the shoulder; elbow; and hip

The nurse is caring for a client admitted to the emergency room with suspected meningitis. The nurse prepares to perform which nursing intervention upon physician orders, while diagnostic testing is being completed?

Administration of antibiotics The nurse should prepare to administer antibiotics as ordered by the physician while the diagnostic tests are being completed. Delay in initiation of antimicrobial therapy, most frequently due to medical imaging prior to lumbar puncture or transfer to another medical facility, can result in poor client outcomes.

Patient who has just returned to the unit after balloon valvuloplasty

After receiving information about four patients during change-of-shift report, which patient should the nurse assess first?

A client with a history of osteoarthritis in his hip, which he refers to as "bad hip," is also complaining of knee pain. The physiological principle behind this would include the fact that:

All joints of an extremity are innervated by the same peripheral nerves as they travel down the limb.

A client who has a fractured ankle is complaining of knee and hip pain. What is the most likely cause of these symptoms?

All nerves that cross articulations innervate each joint of an extremity.

For seizure disorders that do not respond to anticonvulsant medications, the option for surgical treatment exists. What is removed in the most common surgery for seizure disorders?

Amygdala The most common surgery consists of removal of the amygdala and an anterior part of the hippocampus and entorhinal cortex, as well as a small part of the temporal pole, leaving the lateral temporal neocortex intact. Only a portion of the hippocampus and entorhinal cortex, and temporal pole are removed.

For seizure disorders that do not respond to anticonvulsant medications, the option for surgical treatment exists. What is removed in the most common surgery for seizure disorders? a) Amygdala b) Temporal neocortex c) Hippocampus d) Entorhinal cortex

Amygdala The most common surgery consists of removal of the amygdala and an anterior part of the hippocampus and entorhinal cortex, as well as a small part of the temporal pole, leaving the lateral temporal neocortex intact. Only a portion of the hippocampus and entorhinal cortex, and temporal pole are removed.

Fatal degenerative disorder in which corticospinal neurons are lost first, followed by spinal motor neurons

Amyotrophic lateral sclerosis (ALS)

A 44 year-old woman who has a longstanding diagnosis of SLE has been able to control her symptoms with lifestyle modifications for several years, but has presented to her care provider due to recent exacerbation. Which of the following pharmacological treatment options is her care provider most likely to rule out first?

Antiplatelet aggregators

The nurse is conducting a staff inservice on increased intracranial pressure. The nurse determines that the participants are understanding the information when they identify that blood pressure increases in increased intracranial pressure because of which of the following?

An attempt to increase cerebral perfusion Blood pressure increases in increased intracranial pressure due to the body's attempt to increase tissue perfusion. When the pressure in the cranial cavity approaches or exceeds the mean arterial pressure, tissue perfusion becomes inadequate.

What medication teaching should be done for a woman of childbearing age with a seizure disorder?

Antiseizure drugs increase the risk for congenital abnormalities. For women with epilepsy who become pregnant, antiseizure drugs increase the risk for congenital abnormalities and other perinatal complications.

One of the best products for extremely dry skin on the elbows would be to apply which type of dressing?

An occlusive with petroleum material mixed in the cream

The nurse is teaching a client about appropriate interventions for back pain. Select all that apply

Analgesics Nonsteroidal anti-inflammatory drugs

A nurse is teaching the patient about extra-articular manifestations of rheumatoid arthritis (RA). Which of the following will the nurse include in the teaching plan?

Anemia

A patient has a diagnosis of chronic renal failure secondary to diabetic nephropathy. Which of the following hematologic changes may result from this patient's kidney disorder?

Anemia

HLA-B27 antigen may be linked to other genes that determine the pathologic autoimmune phenomenon in:

Ankylosing Spondylitis

A nurse is teaching a client newly diagnosed with a seizure disorder about medications. The most important information for the nurse to provide would be:

Antiepileptic medications should never be discontinued abruptly. Consistency in taking seizure medications is essential to obtaining and maintaining therapeutic blood levels of the medication. Abrupt withdrawal can cause seizure recurrence. Monitoring and assessment of drug levels are important.

A nurse is teaching a client newly diagnosed with a seizure disorder about medications. The most important information for the nurse to provide would be: a) Antiepileptic medications should never be discontinued abruptly. b) Pregnant women should reduce the dose of medication they are taking. c) All antiepileptic medications should be taken with food. d) Children can build up a tolerance to the medication quickly.

Antiepileptic medications should never be discontinued abruptly. Consistency in taking seizure medications is essential to obtaining and maintaining therapeutic blood levels of the medication. Abrupt withdrawal can cause seizure recurrence. Monitoring and assessment of drug levels are important. Each prescribed drug will provide information regarding administration to provide client safety.

A patient with a diagnosis of chronic kidney disease(CKD) May require the administration of which of the following drugs to treat the consequences of CKD?

Antihypertensive medications

What medication teaching should be done for a woman of childbearing age with a seizure disorder? a) Antiseizure drugs do not interact with oral contraceptives. b) Antiseizure drugs increase the risk for congenital abnormalities. c) All women of childbearing age should be advised to take a vitamin C supplement. d) Some antiseizure drugs can interfer with vitamin K metabolism.

Antiseizure drugs increase the risk for congenital abnormalities. For women with epilepsy who become pregnant, antiseizure drugs increase the risk for congenital abnormalities and other perinatal complications. - Many of the antiseizure medications interact with oral contraceptives and can interfere with vitamin D metabolism. -All woman should be advised to take folic acid supplementation.

A client develops fever, headache, and burning/itching in the periorbital area. After a few days, a vesicular rash appears around the eyelid margins. The health care provider will likely prescribe:

Antiviral medication for herpes zoster ophthalmicus

In which of the following locations would a clinician expect to find a ligament?

Around the capsule that forms the knee joint.

Systemic lupus erythematosus has been called the great imitator because it can affect many different body systems. What is among the most commonly occurring symptoms in the early stages of SLE?

Arthritis

articular cartilage

Articular cartilage: example of hyaline cartilage; unique in that its free surface is not covered with perichondrium

Which of the following describes the blood supply to a synovial joint?

Articulating areas are nourished indirectly by the synovial fluid.

When working with a patient with diffuse scleroderma who is exhibiting a 'stone face' expression, the nurse should consider which of the following to be a priority nursing diagnosis for this patient?

Aspiration, risk for related to swallowing impairments.

The nurse is caring for a client with a brain tumor when the client begins to vomit. Which intervention should the nurse do first? a) Document the finding as it is an expected symptom b) Assess for signs/symptoms of cerebral vascular accident c) Assess for other signs/symptoms of increased intracranial pressure d) Contact physician for anti-nausea medication orders

Assess for other signs/symptoms of increased intracranial pressure The tumor may be causing increased intracranial pressure. Vomiting, with or without nausea, is a common symptom of increased intracranial pressure and/or brain stem compression. The nurse's first action is to assess for other signs/symptoms of increased intracranial pressure. Once the assessment is completed, the nurse should contact the physician if indicated by the findings.

The nurse is caring for a client with a brain tumor when the client begins to vomit. Which intervention should the nurse do first?

Assess for other signs/symptoms of increased intracranial pressure. The tumor may be causing increased intracranial pressure. Vomiting, with or without nausea, is a common symptom of increased intracranial pressure and/or brain stem compression. The nurse's first action is to assess for other signs/symptoms of increased intracranial pressure. Once the assessment is completed, the nurse should contact the physician if indicated by the findings.

A patient presents with arthralgia and arthritis. Which of the following is the priority nursing action?

Assessing the patient's pain and history

A 47 year-old woman was diagnosed with amyotrophic lateral sclerosis three years ago and has experienced a progressive onset and severity of complications. She has been admitted to a palliative care unit due to her poor prognosis? What assessments and interventions should the nursing staff of the unit prioritize in their care?

Assessment of swallowing ability and respiratory status.

The two main categories of glial tumors include which of the following? a) Meningiomas b) Oligohydraminos c) Ependymomas d) Astrocytic

Astrocytic Glial tumors are divided into two main catgories: astrocytic and oligodendroglial.

A hospital patient has been complaining of increasing fatigue for several hours, and his nurse has entered his room to find him unarousable. The nurse immediately checked the patient's blood glucose level (and re-verified with a second blood glucose meter), which is 22 mg/dL (1.2 mmol/L). The nurse should prepare to administer which of the following? A. A snack that combines simple sugars, protein, and complex carbohydrates B. A 50% glucose solution intravenously C. Infusion of rapid-acting insulin D. An oral solution containing glucagon and simple sugars

B

A hospital patient with a diagnosis of type 1 diabetes has been administered a scheduled dose of regular insulin. Which of the following effects will result from the action of insulin? A. Promotion of fat breakdown B. Promotion of glucose uptake by target cells C. Promotion of gluconeogenesis D. Initiation of glycogenolysis

B

A pathologic stress fracture occurs in bones subjected to: A. Sudden direct force B. Weakening by disease C. Repeated excessive use D. Massive muscle contraction

B

A patient with a history of diabetes presents to the emergency department following several days of polyuria and polydipsia with nausea/vomiting. On admission, the patient labs show a blood glucose level of 480 mg/dL and bicarbonate level of 7.8 mEq/dL. The nurse suspects the patient has diabetic ketoacidosis (DKA). The priority intervention should include: A. Limit fluid intake to only 250 mL/4 hours. B. Begin a loading dose of IV regular insulin followed by a continuous insulin infusion. C. Give at least 50 units of regular insulin IV stat and recheck blood glucose in 2 hours. D. Push a stat dose of bicarbonate followed by a double-dose (loading) of metformin.

B

A patient's recent computed tomography (CT) scan has revealed the presence of hydrocephalus. Which of the following treatment measures is most likely to resolve this health problem? A. Aggressive diuresis B. Placement of a shunt C. Administration of hypertonic intravenous solution D. Lumbar puncture

B

An elderly patient has been brought to his primary care provider by his wife who is concerned about his recent decrease in coordination. Upon assessment, his primary care provider notes that the patient's gait is wide-based, unsteady, & lacking in fluidity, although his muscle tone appears normal. This patient requires further assessment for which of the following health problems? A. Muscle atrophy B. Cerebellar disorder C. Impaired spinal reflexes D. Lower motor neuron lesions

B

An elderly resident of an assisted-living facility has had his mobility & independence significantly impaired by the progression of his rheumatoid arthritis (RA). What is the primary pathophysiologic process that has contributed to this patient's decline in health? A. A mismatch between bone resoprtion & remodeling. B. Immunologically mediated joint inflammation. C. Excessive collagen production & deposition. D. Cytokine release following mechanical joint injury.

B

An elderly resident of an assisted-living facility has had his mobility and independence significantly impaired by the progression of his rheumatoid arthritis (RA). What is the primary pathophysiologic process that has contributed to this patient's decline in health? A. A mismatch between bone resorption and remodeling B. Immunologically mediated joint inflammation C. Excessive collagen production and deposition D. Cytokine release following mechanical joint injury

B

An increase in the level of RANKL would result in: A. fusing of the epiphysis & metaphysic on long bones. B. changes in the rate of bone remodeling. C. decreased production of PTH. D. increased vitamin D activation.

B

Assessment of a newborn infant reveals the presence of developmental dysplasia of the hip (DDH) that is currently demonstrated by subluxation of the baby's hip joint & a general laxity in the baby's ligaments. What measures should be emphasized in this infant's treatment? A. Corticosteroid therapy B. Close observation C. Open reduction D. Joint reconstruction

B

Cartilage is a firm but flexible type of connective tissue that is essential for: A. Calcium salt storage B. Growth of long bones C. Bone surface perdusion D. Reduced friction on tendons

B

For many patients, the first indication that they have osteoporosis is: A. bone pain that is not alleviated by rest. B. a bone fracture. C. craving high-calcium foods. D. decreased ROM in the hip & knee joints.

B

In contrast to structural scoliosis, postural scoliosis: A. Compresses vertebrae B. Corrects with bending C. Is apparent at birth D. Becomes painful

B

In order to initiate cellular events essential to bone healing, there must be ______ formation at the fracture site. A. Callous B. Hematoma C. Ossification D. Fibrin meshwork

B

Restoration of the integrity of myelin sheaths would likely result in a slowing or stopping of the progression of: A. Amyotrophic lateral sclerosis (ALS). B. Multiple sclerosis (MS). C. Duchenne muscular dystrophy (DMD). D. Paralysis caused by Clostridium botulinum.

B

Rheumatoid arthritis is a systemic inflammatory disease with joint manifestations described as: A. dysplasia. B. polyarticular. C. asymmetrical. D. osteophytes.

B

The cardinal symptoms of Parkinson disease include: A. Hypotonia B. Bradykinesia C. Paresthesia D. Lack of sweating

B

To maintain adequate serum calcium levels, PTH reduces: A. Activation of vitamin D B. Serum phosphate levels C. Calcium release from bone D. Intestinal absorption of calcium

B

What role do osteoblasts play in the physiology of bone tissue? A. differentiation into mature bone cells. B. synthesis & secretion of bone matrix. C. maintenance of calcium balance. D. resorption of the bone matrix.

B

Whereas PTH increases blood calcium levels, the hormone _____ lowers blood calcium levels & decreases bone resorption. A. Vitamin D B. Calcitonin C. Prolactin D. Phosphate

B

Which of the following patients presenting to the emergency department would most likely be diagnosed with a pathologic stress fracture? A. A teenager who fell of a ladder and hit the concrete driveway, landing on his hip B. A postmenopausal female who was diagnosed with breast cancer with metastasis to bone C. A competitive volleyball player diving to retrieve a volley and landing on his hip D. A weight lifter who bench-presses 200 kg lost balance and fell to the side, landing on his hip

B

Which of the following signs and symptoms should prompt a 29-year-old woman's primary care provider to assess for systemic lupus erythematosus (SLE)? A. Chronic nausea and vomiting that is unresponsive to antimetics B. Joint pain & increased creatinine & blood urea nitrogen C. A history of thromboembolic events & varicose veins D. Dysmenorrhea & recent spontaneous absorption

B

Which of the following vision deficits is a clinician justified in attributing to the normal aging process? A. Conjunctivitis B. Presbyopia C. Strabismus D. Angle-closure glaucoma

B

Which of the following characteristics differentiates cartilage from bone? A) Secretion of an extracellular matrix B) Avascularity C) Low tensile strength D) Rapid healing

B) Avascularity

Assessment of a newborn infant reveals the presence of developmental dysplasia of the hip (DDH) that is currently demonstrated by subluxation of the babys hip joint and a general laxity in the babys ligaments. What measures should be emphasized in this infants treatment? A) Corticosteroid therapy B) Close observation C) Open reduction D) Joint reconstruction

B) Close observation

An elderly resident of an assisted-living facility has had his mobility and independence significantly impaired by the progression of his rheumatoid arthritis (RA). What is the primary pathophysiologic process that has contributed to this patients decline in health? A) A mismatch between bone resorption and remodeling B) Immunologically mediated joint inflammation C) Excessive collagen production and deposition D) Cytokine release following mechanical joint injury

B) Immunologically mediated joint inflammation

A 55-year-old male patient has reported joint pain in his feet. Which of the following blood work results should prompt further testing to rule out primary gout? A) Increased C-reactive protein (CRP) B) Increased serum uric acid C) Increased polymorphonuclear leukocytes D) Increased serum cortisol

B) Increased serum uric acid

Which of the following signs and symptoms should prompt a 29-year-old womans primary care provider to assess for systemic lupus erythematosus (SLE)? A) Chronic nausea and vomiting that is unresponsive to antiemetics B) Joint pain and increased creatinine and blood urea nitrogen C) A history of thromboembolic events and varicose veins D) Dysmenorrhea and recent spontaneous abortion

B) Joint pain and increased creatinine and blood urea nitrogen

What role do osteoblasts play in the physiology of bone tissue? A) Differentiation into mature bone cells B) Synthesis and secretion of bone matrix C) Maintenance of calcium balance D) Resorption of the bone matrix

B) Synthesis and secretion of bone matrix

An increase in the level of RANKL would result in: A) fusing of the epiphysis and metaphysic in long bones. B) changes in the rate of bone remodeling. C) decreased production of PTH. D) increased vitamin D activation.

B) changes in the rate of bone remodeling.

In contrast to structural scoliosis, postural scoliosis: A) compresses vertebrae. B) corrects with bending. C) is apparent at birth. D) becomes painful.

B) corrects with bending.

Cartilage is a firm but flexible type of connective tissue that is essential for: A) calcium salt storage. B) growth of long bones. C) bone surface perfusion. D) reduced friction on tendons.

B) growth of long bones.

In order to initiate the cellular events essential to bone healing, there must be ________ formation at the fracture site. A) callous B) hematoma C) ossification D) fibrin meshwork

B) hematoma

A child has been hospitalized for the treatment of hematogenous osteomyelitis. The defining characteristic of this type of osteomyelitis is: A) the presence of dead bone tissue. B) introduction of microorganisms from the bloodstream. C) bacterial proliferation in the absence of the classic signs of infection. D) destruction of the vascular network in the endosteum.

B) introduction of microorganisms from the bloodstream.

If the nurse suspects a spinal cord injury patient has developed autonomic dysreflexia. Which of the following assessments would confirm this complication? Select all that apply.

BP 180/98. Pulse rate 49. Complains of a pounding headache.

breakdown products of proteins

BUN ( blood urea nitrogen)

A patient has a bacterial skin infection. Which medication can be applied to the lesion?

Bacitracin is used to treat a bacterial skin infection. Butenafine is an antifungal agent. Clotrimazole is used to treat tinea infections. Benzoyl peroxide is used to treat acne.

A client has been instructed to increase Vitamin D in his/her diet. What foods will the nurse make certain are included in the diet?

Baked fish and whole milk

metaplasia from squamous to columnar (increases risk for cancer)

Barrett's esophagus

The nurse caring for a client with an aneurysmal subarachnoid hemorrhage understands that the most common cause of this condition is which of the following?

Berry aneurysm Aneurysmal subarachnoid hemorrhage is a type of hemorrhagic stroke caused by the rupture of a cerebral aneurysm and resultant bleeding into the subarachnoid space. Most of these aneurysms are small saccular aneurysms, called berry aneurysms.

Which of the following would be considered functions of the skin? Select all that apply.

Besides providing a covering for the entire body surface, the skin performs many other functions, including protection against physical injury, sunlight, and microorganisms; prevention of loss of fluids from the internal environment; regulation of body temperature; continual reception of sensations from the environment, such as touch, temperature, and pain; and synthesis of vitamin D through the action of sunlight on the skin. Acid-base regulation occurs primarily in the lungs and kidneys. Metabolic rate is regulated by many hormones, not the skin.

A 15-year-old boy has begun acne treatment using a combination ointment of clindamycin and benzoyl peroxide. The nurse should teach the boy to expect maximum results in how long?

Best results of combination acne treatments require 8 to 12 weeks of therapy, and maintenance therapy is usually required.

A patient with rheumatoid arthritis (RA) tells the nurse that the disease-modifying antirheumatic drug (DMARD) methotrexate does not seem to be working anymore. Which of the following does the nurse relate can also be tried after DMARD failure?

Biologic response modifiers

When explaining to the client diagnosed with gout how the xanthine oxidase inhibitors work to help treat gout, the health care provider would include which of the following data? Allopurinol:

Blocks the production of uric acid by the body

A client has fractured his sternum when his chest hit the steering wheel during an accident. Which of the following statements most accurately describes the physiologic function of bone marrow?

Blood cell production takes place exclusively in red bone marrow, which predominates in infants and decreases (but not disappears) with age. Yellow bone marrow is composed primarily of adipose tissue and is not hematopoietically active. As the need for RBC production decreases during postnatal life, red marrow is gradually replaced with yellow bone marrow in most of the bones. In adults, red marrow persists in the vertebrae, ribs, sternum, and ilea.

Following a collision while mountain biking, the diagnostic work up of a 22 year-old male has indicated the presence of an acute subdural hematoma. Which of the following pathophysiological processes most likely underlies his diagnosis?

Blood has accumulated between the man's dura and subarachnoid space. A subdural hematoma develops in the area between the dura and the arachnoid space while epidural hematomas exist between the skull and dura.

Following a collision while mountain biking, the diagnostic work up of a 22 year-old male has indicated the presence of an acute subdural hematoma. Which of the following pathophysiological processes most likely underlies his diagnosis? a) Blood has accumulated between the man's dura and subarachnoid space. b) A traumatic lesion in the frontal or temporal lobe has resulted in increased ICP. c) Blood has displaced CSF in the ventricles as a consequence of his coup-contrecoup injury. d) Vessels have burst between the client's skull and his dura.

Blood has accumulated between the man's dura and subarachnoid space. A subdural hematoma develops in the area between the dura and the arachnoid space while epidural hematomas exist between the skull and dura. -Intracerebral hematomas are located most often in the frontal or temporal lobe and the ventricles are not directly involved in a subdural hematoma.

Neurotoxins such as the botulism organism can produce paralysis by what mechanism?

By blocking release of acetylcholine

bone marrow

Bone marrow: occupies the medullary cavities of the long bones throughout the skeleton; and the cavities of the cancellous bone in the vertebrae, ribs, sternum, and flat bones of the pelvis

medullary cavities

Bone marrow: occupies the medullary cavities of the long bones throughout the skeleton; and the cavities of the cancellous bone in the vertebrae, ribs, sternum, and flat bones of the pelvis

Which of the following statements about bone mass and the elderly is accurate?

Bone resorption and formation are no longer perfectly coupled, and bone mass is lost.

osteoblasts

Bone-building cells ~ synthesize and secrete the organic matrix of bone Responsible for the formation of the bone matrix through two stages: Ossification: formation of osteoid, or prebone Calcification: deposition of calcium salts in the osteoid tissue

appendicular skeleton

Bones of the limbs and limb girdles that are attached to the axial skeleton

The skeletal system is composed of bone and cartilage. Which of the following statements regarding the skeletal system is most accurate?

Bones provide protection for internal organs.

A teenager has been in a car accident and experienced acceleration-deceleration head injury. Initially, the client was stable but then started to develop neurological signs/symptoms. The nurse caring for this client should be assessing for which type of possible complication?

Brain contusions and hematomas

Skeletal tissue contains intercellular collagen fibers that provide ______ for tendons and ligaments. A) elastic recoil B) shape flexibility C) tensile strength D) inorganic calcium

C) tensile strength

A teenager has been in a car accident and experienced acceleration-deceleration head injury. Initially, the client was stable but then started to develop neurological signs/symptoms. The nurse caring for this client should be assessing for which type of possible complication?

Brain contusions and hematomas Contusions (focal brain injury) cause permanent damage to brain tissue. The bruised, necrotic tissue is phagocytized by macrophages, and scar tissue formed by astrocyte proliferation persists as a crater. The direct contusion of the brain at the site of external force is referred to as a acceleration injury, whereas the opposite side of the brain receives the deceleration injury from rebound against the inner skull surfaces. As the brain strikes the rough surface of the cranial vault, brain tissue, blood vessels, nerve tracts, and other structures are bruised and torn, resulting in contusions and hematomas.

A teenager has been in a car accident and experienced acceleration-deceleration head injury. Initially, the client was stable but then started to develop neurological signs/symptoms. The nurse caring for this client should be assessing for which type of possible complication? a) Status epilepticus b) Brain contusions and hematomas c) Momentary unconsciousness d) TIAs and cerebrovascular infarction

Brain contusions and hematomas Contusions (focal brain injury) cause permanent damage to brain tissue. The bruised, necrotic tissue is phagocytized by macrophages, and scar tissue formed by astrocyte proliferation persists as a crater. The direct contusion of the brain at the site of external force is referred to as a acceleration injury, whereas the opposite side of the brain receives the deceleration injury from rebound against the inner skull surfaces. As the brain strikes the rough surface of the cranial vault, brain tissue, blood vessels, nerve tracts, and other structures are bruised and torn, resulting in contusions and hematomas. -TIAs and cerebral vascular infarction (stroke) are often caused by atherosclerotic brain vessel occlusions that cause ischemic injuries. - In mild concussion head injury, there may be momentary loss of consciousness without demonstrable neurologic symptoms or residual damage, except for possible residual amnesia. -Status epilepticus is not related to this situation.

A teenager has been in a car accident and experienced acceleration-deceleration head injury. Initially, the client was stable but then started to develop neurological signs/symptoms. The nurse caring for this client should be assessing for which type of possible complication? a) Brain contusions and hematomas b) Momentary unconsciousness c) TIAs and cerebrovascular infarction d) Status epilepticus

Brain contusions and hematomas Contusions (focal brain injury) cause permanent damage to brain tissue. The bruised, necrotic tissue is phagocytized by macrophages, and scar tissue formed by astrocyte proliferation persists as a crater. The direct contusion of the brain at the site of external force is referred to as a acceleration injury, whereas the opposite side of the brain receives the deceleration injury from rebound against the inner skull surfaces. As the brain strikes the rough surface of the cranial vault, brain tissue, blood vessels, nerve tracts, and other structures are bruised and torn, resulting in contusions and hematomas. -TIAs and cerebral vascular infarction (stroke) are often caused by atherosclerotic brain vessel occlusions that cause ischemic injuries. -In mild concussion head injury, there may be momentary loss of consciousness without demonstrable neurologic symptoms or residual damage, except for possible residual amnesia. -Status epilepticus is not related to this situation.

A client has sustained a severe, diffuse brain injury that resulted in seriously compromised brain function. The client is at greatest risk for:

Brain death Severe brain injury that results in seriously compromised brain function can result in brain death.

Manifestations of brain tumors are focal disturbances in brain function and increased ICP. What causes the focal disturbances manifested by brain tumors?

Brain edema and disturbances in blood flow Intracranial tumors give rise to focal disturbances in brain function and increased ICP. Focal disturbances occur because of brain compression, tumor infiltration, disturbances in blood flow, and brain edema.

Manifestations of brain tumors are focal disturbances in brain function and increased ICP. What causes the focal disturbances manifested by brain tumors? a) Brain edema and disturbances in blood flow b) Brain compression and decreased ICP c) Tumor infiltration and decreased ICP d) Tumor infiltration and increased blood pressure

Brain edema and disturbances in blood flow Intracranial tumors give rise to focal disturbances in brain function and increased ICP. Focal disturbances occur because of brain compression, tumor infiltration, disturbances in blood flow, and brain edema. Blood pressure, either increased or decreased, is not a manifestation of a brain tumor.

deep

pain that isdiffuse, throbbing pain that originates in muscles, bones, & tendons & radiates to the surrounding tissues

A patient admitted to the emergency department with a change in mental status and a history of AIDS and primary central nervous system (CNS) lymphoma becomes extremely combative with the medical personnel. A family member is very upset with the patient's behavior. The nurse explains that these behaviors are most likely caused by which of the following? a) Recurrence of primary CNS lymphoma b) Decrease in intracranial pressure c) Onset of early dementia d) AIDS-induced encephalopathy

Brain edema and disturbances in blood flow Intracranial tumors give rise to focal disturbances in brain function and increased ICP. Focal disturbances occur because of brain compression, tumor infiltration, disturbances in blood flow, and brain edema. -Blood pressure, either increased or decreased, is not a manifestation of a brain tumor.

A client with confirmed low bone density asks the nurse if there is anything she can to decrease the risk of trauma. The best response would be:

Brisk walking three times per week on a flat surface

Which of the following disorders is a chronic respiratory disease that is characterized by irreversible dilation of medium-to small-sized bronchi and chronic infection? a. asthma b. bronchiectasis c. bronchitis d. cystic fibrosis

Bronchiectasis

A client has been involved in a motor vehicle accident and has sustained spinal cord damage. The client has voluntary motor function loss on one side and a loss of pain and temperature sensation from the contralateral side below the level of the lesion. What type of cord syndrome does the nurse recognize the client has sustained?

Brown-Sequard syndrome

one side of cord -Motor function lost on that side; pain/temperature sensation lost from other side

Brown-Séquard syndrome

When the suspected diagnosis is bacterial meningitis, what assessment techniques can assist in determining if meningeal irritation is present?

Brudzinski sign and Kernig sign Two assessment techniques can help determine whether meningeal irritation is present. Kernig sign is resistance to extension of the knee while the person is lying with the hip flexed at a right angle. Brudzinski sign is elicited when flexion of the neck induces flexion of the hip and knee.

When the suspected diagnosis is bacterial meningitis, what assessment techniques can assist in determining if meningeal irritation is present? a) Kernig sign and Chadwick sign b) Chvostek sign and Guedel sign c) Brudzinski sign and Chadwick sign d) Brudzinski sign and Kernig sign

Brudzinski sign and Kernig sign Two assessment techniques can help determine whether meningeal irritation is present. Kernig sign is resistance to extension of the knee while the person is lying with the hip flexed at a right angle. Brudzinski sign is elicited when flexion of the neck induces flexion of the hip and knee. The other answers are incorrect.

A child with a diagnosis of Legg-Calve-Perthes disease will exhibit: A. defective synthesis of type I collagen. B. congenital dislocation of the acetabulofemoral joint. C. necrosis of the proximal femoral head. D. intoeing due to metatarsus adductus.

C

A decrease in the serum level of which of the following substances is suggestive of liver injury? gamma-glutamyltransferase (GGT) albumin alanine aminotransferase (ALT) alkaline phosphatase (AP)

C

A female athlete has been diagnosed with amenorrhea due to intense training for a spot on the Olympic swimming team. As a health care provider, which of the following should be implemented to prevent premature osteoporosis? A. Encourage a minimum of 10 hours of sleep/night. B. Increase dietary intake of protein and iron. C. Calcium/vitamin D supplements to support bone mineral density. D. Watch sodium intake and eat a carb-consistent diet with lots of fruits.

C

A patient is devastated to receive a diagnosis of amyotrophic lateral sclerosis (ALS). The symptomatology of this disease is a result of its effects on upper and lower motor neurons. The health care provider caring for this patient will focus on which priority intervention for this patient? A. Ability to turn from side to side, thereby preventing skin breakdown B. Ability to empty bladder completely, thereby preventing autonomic dysreflexia C. Respiratory ventilation assessment and prevention of aspiration pneumonia D. Assessment of lower extremities to prevent deep vein thrombosis

C

A patient with a diagnosis of myasthenia gravis has required a mastecomy for the treatment of breast cancer. The surgery has been deemed a success, but the patient has gone into a myasthenic crisis on postoperative day one. Which of the following measures should the care team prioritize in this patient's immediate care? A. Positioning the patient to minimize hypertonia & muscle rigidity. B. Seizure precautions C. Respiratory support & protection of the patient's airway. D. Monitoring the patient for painful dyskinesias.

C

A patient with a diagnosis of myasthenia gravis has required a mastectomy for the treatment of breast cancer. The surgery has been deemed a success, but the patient has gone into a myasthenic crisis on postoperative day 1. Which of the following measures should the care team prioritize in this patient's immediate care? A. Positioning the patient to minimize hypertonia and muscle rigidity B. Seizure precautions with padded side rails and bed in lowest height C. Respiratory support and protection of the patient's airway D. Monitoring the patient for painful dyskinesias

C

A patient with a diagnosis of small cell lung carcinoma has developed bone metastases, a finding that has prompted a series of new interventions. What are the primary goals of the treatment regimen for this patient's bone cancer? A. Prevention of brain metastasis & early identification of osteonecrosis. B. Promotion of bone remodeling at tumor sites through calcium & vitamin D supplements. C. Prevention of pathologic fractures & maximization of function. D. Pain management & prevention of osteomyelitis.

C

A patient with a long history of cigarette smoking and poorly controlled hypertension has experienced recent psychomotor deficits as a result of hemorrhagic brain damage. The client's psychomotor deficits are likely the result of: A. Alzheimer disease B. Frontotemporal dementia (FTD) C. Vascular dementia D. Wernicke-Korsakoff syndrome

C

An adult female patient visits with her health care provider about pain in her hand. She describes it as an audible grinding and cracking sound, especially in her thumb. "I had to buy an automatic jar opener...I just can't grasp and open a jar...it just hurts too badly." The health care provider suspects the patient has a degenerative form of joint disease that is often evidenced by: A. Rheumatoid arthritis B. Systemic lupus erythematosus C. Osteoarthritis D. Ankylosing spondylitis

C

An elderly male patient has been brought to the emergency department after experiencing stroke-like symptoms a few hours ago, and has been subsequently diagnosed with an ischemic stroke. The care team is eager to restore cerebral perfusion despite the likely death of the brain cells directly affected by the stroke. What is the rationale for the care team's emphasis on restoring circulation? A. Failure to restore blood flow creates a severe risk for future transient ischemic attacks. B. Necrosis will continue unabated throughout the brain unless blood flow is restored. C. Cells of the penumbra may be saved from hypoxic damage if blood flow is promptly restored. D. Unless blood flow is restored, the patient faces the risk of progressing to hemorrhagic stroke.

C

Factors that may adversely affect bone healing include _______ of the bone. A. Immobilization B. Weight bearing C. Delayed union D. Tight alignment

C

Osteoarthritis is a degenerative form on joint disease that is often evidenced by: A. spongy bones. B. cartilage hypertrophy. C. crepitus & grinding. D. systemic inflammation

C

Skeletal tissue contains intercellular collagen fibers that provide _____ for tendons & ligaments. A. Elastic recoil B. Shape flexibility C. Tensile strength D. Inorganic calcium

C

The clinical course of osteoarthritis (OA) culminates in: A. osteonecrosis & loss of synovial fluid. B. formation of tophi in the synovial space. C. osteophyte formation & erosion of cartilage. D. separation of the epiphyseal plate.

C

The joint capsule consists of an outer fibrous layer & an inner synovium, which _____ to facilitate movement. A. senses position B. connects to tendons C. secretes synovial fluid D. covers articular cartilage

C

The most common cause of thyrotoxicosis is Graves disease. When assessing this client, the nurse should put priority on which of the following signs/symptoms? A. Complaints of muscle fatigue B. Facial myxedema with puffy eyelids C. Ophthalmopathy D. Pulse rate of 64 beats/minute

C

The patient has a traumatic complete spinal cord transection at the C5 level. Intact motor & somatosensory function will include ____ control. A. Bladder B. Finger flexion C. Diaphragm D. Trunk muscle

C

The tendons & ligaments of the joint capsule are sensitive to position & movement as a result of having ____ to help maintain muscle support. A. autonomic nerve fibers B. supporting bursa sacs C. reflexive proprioception D. elastic articular cartilage

C

Which of the following factors is most strongly associated with the pathogenesis of gallstones? A. Excess serum ammonia and urea levels B. Portal hypertension C. Abnormalities or stasis of bile D. High-cholesterol diet

C

Which of the following measures should a public health nurse recommend to middle-aged women to reduce their chances of developing osteoporosis later in life? A. Weight-control & daily use of low-dose corticosteroids. B. Genetic testing & ROM exercises. C. Calcium supplementation & regular physical activity. D. Increased fluid intake & use of vitamin D supplements.

C

While trying to explain the physiology behind type 2 diabetes to a group of nursing students, the instructor will mention which of the following accurate information? A. The destruction of beta cells and absolute lack of insulin in people with type 2 diabetes means that they are particularly prone to the development of diabetic complication. B. Because of the loss of insulin response, all people with type 2 diabetes require exogenous insulin replacement to control blood glucose levels. C. In skeletal muscle, insulin resistance prompts decreased uptake of glucose. Following meals (postprandial), glucose levels are higher due to diminished efficiency of glucose clearance. D. They have increased predisposition to other autoimmune disorders such as Graves disease, rheumatoid arthritis, and Addison disease.

C

how are the oxygenation needs of the articulating areas in a synovial joint met? A. capillaries release oxygenated blood into the synovial cavity at a controlled rate. B. the cartilage at the articulating areas uses anaerobic metabolism exclusively. C. oxygen is provided to the articulating area by synovial fluid rather than by blood. D. the epiphysis of long bones provide oxygen to the articulating areas.

C

Which of the following measures should a public health nurse recommend to middle-aged women to reduce their chances of developing osteoporosis later in life? A) Weight-control and daily use of low-dose corticosteroids B) Genetic testing and range of motion exercises C) Calcium supplementation and regular physical activity D) Increased fluid intake and use of vitamin D supplements

C) Calcium supplementation and regular physical activity

How are the oxygenation needs of the articulating areas in a synovial joint met? A) Capillaries release oxygenated blood into the synovial cavity at a controlled rate. B) The cartilage of the articulating areas uses anaerobic metabolism exclusively. C) Oxygen is provided to the articulating areas by synovial fluid rather than by blood. D) The epiphyses of long bones provide oxygen to the articulating areas.

C) Oxygen is provided to the articulating areas by synovial fluid rather than by blood.

A patient with a diagnosis of small cell lung carcinoma has developed bone metastases, a finding that has prompted a series of new interventions. What are the primary goals of the treatment regimen for this patients bone cancer? A) Prevention of brain metastasis and early identification of osteonecrosis B) Promotion of bone remodeling at tumor sites through calcium and vitamin D supplements C) Prevention of pathologic fractures and maximization of function D) Pain management and prevention of osteomyelitis

C) Prevention of pathologic fractures and maximization of function

Osteoarthritis is a degenerative form of joint disease that is often evidenced by: A) spongy joints. B) cartilage hypertrophy. C) crepitus and grinding. D) systemic inflammation.

C) crepitus and grinding.

Factors that may adversely affect bone healing include______ of the bone. A) immobilization B) weight bearing C) delayed union D) tight alignment

C) delayed union

A child with a diagnosis of Legg-Calv-Perthes disease will exhibit: A) defective synthesis of type I collagen. B) congenital dislocation of the acetabulofemoral joint. C) necrosis of the proximal femoral head. D) intoeing due to metatarsus adductus.

C) necrosis of the proximal femoral head.

The clinical course of osteoarthritis (OA) culminates in: A) osteonecrosis and loss of synovial fluid. B) formation of tophi in the synovial space. C) osteophyte formation and erosion of cartilage. D) separation of the epiphyseal plate.

C) osteophyte formation and erosion of cartilage.

The joint capsule consists of an outer fibrous layer and an inner synovium, which ______ to facilitate movement. A) senses position B) connects to tendons C) secretes synovial fluid D) covers articular cartilage

C) secretes synovial fluid

The nurse expects that loss of respiratory effort occurs with a spinal injury at which of the following levels?

C1-3

At what level of the cervical spine would a complete cord injury prevent flexion and extension of the fingers?

C8

A significant complication of the acute phase of burn injury is:

sepsis

Intracranial aneurysms that rupture cause subarachnoid hemorrhage in the client. How is the diagnosis of intracranial aneurysms and subarachnoid hemorrhage made?

CT scan The diagnosis of subarachnoid hemorrhage and intracranial aneurysms is made by clinical presentation, CT scan, and angiography.

The nurse working in an emergency room is caring for a client who is exhibiting signs and symptoms of a stroke. What does the nurse anticipate that the physician's orders will include?

CT scan The nurse should anticipate that the client will be ordered a CT scan to rule out hemorrhagic stroke that would preclude the administration of tissue plasminogen activator (tPA).

The nurse working in an emergency room is caring for a client who is exhibiting signs and symptoms of a stroke. What does the nurse anticipate that the physician's orders will include? a) CT scan b) MRI c) Intravenous antibiotics d) pain medication

CT scan The nurse should anticipate that the client will be ordered a CT scan to rule out hemorrhagic stroke that would preclude the administration of tissue plasminogen activator (tPA).

Intracranial aneurysms that rupture cause subarachnoid hemorrhage in the client. How is the diagnosis of intracranial aneurysms and subarachnoid hemorrhage made? a) CT scan b) Loss of cranial nerve reflexes c) MRI d) Venography

CT scan The diagnosis of subarachnoid hemorrhage and intracranial aneurysms is made by clinical presentation, CT scan, and angiography. - An MRI is not necessary for the diagnosis of subarachnoid hemorrhage and intracranial aneurysm. -Loss of cranial nerve reflexes is not diagnostic of subarachnoid hemorrhage and intracranial aneurysm, and neither is venography.

A client asks the nurse if drinking more milk will prevent the release of calcium from her bones. The best response by the nurse would be that the release of calcium from the bone into the extracellular fluid is regulated by:

Calcitonin

The physician is reviewing lab results for his client with cancer and finds the client to be hypercalcemic. Which of the following will the physcian prescribe?

Calcitonin

Calcitonin has what relationship to bone growth?

Calcitonin lowers blood calcium levels.

calcitonin

Calcitonin probably decreases the number and resorptive function of the osteoclasts

The major stimulus for calcitonin synthesis is the release of an increase in what serum element?

Calcium

Which of the following medications would the nurse anticipate being prescribed for the renal failure patient who has hyperphosphatemia?

Calcium carbonate

A female athlete has been diagnosed with amenorrhea due to intense training for a spot on the Olympic swimming team. As a health care provider, which of the following should be implemented to prevent premature osteoporosis?

Calcium/vitamin D supplements to support BMD.

Peripheral Neuropathy Involvement:

Can involve a single nerve (mononeuropathy) or multiple nerves (polyneuropathy).

A group of student nurses are reviewing the bones in the human body. What is the name for spongy bone?

Cancellous

The student is studying the types of mature bones. The spongy bone found in the interior of bones and composed of spicules is which of the following?

Cancellous

spicules

Cancellous (spongy) bone: found in the interior of bones and is composed of trabeculae or spicules forming a lattice-like pattern

spongy bone

Cancellous (spongy) bone: found in the interior of bones and is composed of trabeculae or spicules forming a lattice-like pattern

trabeculae

Cancellous (spongy) bone: found in the interior of bones and is composed of trabeculae or spicules forming a lattice-like pattern Trabeculae are lined with osteogenic cells and filled with red or yellow bone marrow

cancellous bone

Cancellous (spongy) bone: found in the interior of bones and is composed of trabeculae or spicules forming a lattice-like pattern Trabeculae are lined with osteogenic cells and filled with red or yellow bone marrow Cancellous bone is relatively light ~ structure provides tensile strength and weight-bearing properties

A 14-year-old girl has been thrown from the back of a pick-up truck. MRI shows complete cord injury at the level of C2. What is the main significance of an injury at this level of the spinal column?

Cannot breathe on own, needs ventilator assistance

Peripheral nerve disorders are not uncommon. What is an example of a fairly common mononeuropathy?

Carpal tunnel syndrome

A client is experiencing pain, tingling, and numbness of the thumb and first, second, third, and half of the fourth digits of the hand. She states that she has pain in the wrist and hand, which worsens at night, and she has noticed that they have become clumsy. The nurse recognizes these manifestations as:

Carpel tunnel syndrome

chondrocytes

Cartilage cells (chondrocytes) are located in lacunae ~ the lacunae are surrounded by an uncalcified, gel-like intercellular matrix of collagen fibers and ground substance

A gymnastics student lands awkwardly and hurts her ankle. After MRI scan, it was revealed that she has a torn cartilage. The health care worker states it may take up to 3 to 4 months for this injury to heal. The basic physiologic reason behind the prolonged recovery is due to the fact that cartilage is/has primarily:

Cartilage is avascular tissue, a fact that stands in contrast to bone tissue. Both bone and cartilage contain cells that secrete an extracellular matrix, which forms the structure of the tissue. Cartilage is more flexible than bone, but it also exhibits considerable tensile strength. Cartilage repair is a particularly slow process and may in fact not occur in some cases. In contrast to cartilage, the extracellular matrix of bone is mineralized, producing a hard tissue capable of providing support for the body and protection for its vital structures.

cartilage

Cartilage: essential component of the skeletal system Constitutes the articular cartilage of joints Found in the tendinous and ligamentous insertions; menisci; the symphysis pubis; and insertion of joint capsules Essential for growth before and after birth In the embryo, most of the axial and appendicular skeleton is formed first as a cartilage model and then replaced by bone In postnatal life, cartilage continues to be essential to growth of long bones; and persists as articulate cartilage in the adult

composition of cartilage

Cells and intercellular fibers embedded in an amorphous, gel-like material Cartilage is mostly surrounded by perichondrium which is a vascular sheath that provides nourishment for the chondrocytes

An elderly male patient has been brought to the emergency department after experiencing stroke-like symptoms a few hours ago, and has been subsequently diagnosed with an ischemic stroke. The care team is eager to restore cerebral perfusion despite the likely death of the brain cells directly affected by the stroke. What is the rationale for the care teams emphasis on restoring circulation?

Cells of the penumbra may be saved from hypoxic damage if blood flow is promptly restored

Patient is diagnosed with an ischemic stroke. The care team is eager to restore cerebral perfusion despite the likely death of the brain cells directly affected by the stroke. What is the rationale for the care team's emphasis on restoring circulation?

Cells of the penumbra may be saved from hypoxic damage if blood flow is promptly restored.

An elderly male patient has been brought to the emergency department after experiencing stroke like symptoms a few hours ago, and has been subsequently diagnosed with an ischemic stroke. The care team is eager to restore cerebral perfusion despite the likely death of the brain cells directly affected by the stroke. What is the rationale for the care team's emphasis on restoring circulation?

Cells of the penumbra may be saved from hypoxic damage of blood flow is promptly restored

somatosensory

Central processing of _________ information perception involves: -awareness of stimuli -localization & discrimination of its characteristics -interpretation of its meaning

A middle-aged woman is brought to the emergency room after a minor auto accident. Her gait is staggering and unsteady, her speech is slurred, and she displays slight nystagmus. The police officer who brought her in says she has not been drinking. Her blood pressure is very high. Which of the following health problems most likely underlies her present state?

Cerebellar damage caused by a cerebrovascular accident

An elderly patient has been brought to his primary care provider by his wife who is concerned about his recent decrease in coordination. Upon assessment, his primary care provider notes that the patient's gait is wide-based, unsteady, and lacking in fluidity, although his muscle tone appears normal. This patient requires further assessment for which of the following health problems? Muscle atrophy Cerebellar disorders Impaired spinal reflexes Lower motor neuron lesions

Cerebellar disorders

the patient's gait is wide-based, unsteady, and lacking in fluidity, although his muscle tone appears normal. This patient requires further assessment for which of the following health problems?

Cerebellar disorders

An 80-year-old patient with a history of heavy alcohol use is being seen by his provider for drowsiness, confusion, and headache. His family states that he fell and hit his head "several weeks ago." Which type of hematoma does the provider suspect?

Chronic subdural

The nurse is conducting a community education program on concussions. The nurse evaluates that the participants are understanding the education when they state that which of the following offers the brain protection from external forces?

Cerebral spinal fluid The brain is protected from external forces by the rigid confines of the skull and the cushioning afforded by the cerebrospinal fluid.

Which of the following physiologic changes results in menopause?

Cessation of ovarian function and decreased estrogen levels

A client has suffered a stroke that has affected his speech. The physician has identified the client as having expressive aphasia. Later in the day, the family asks the nurse to explain what this means. The most accurate response would be aphasia that is:

Characterized by an inability to easily communicate spontaneously or translate thoughts or ideas into meaningful speech or writing Expressive or nonfluent aphasia is characterized by an inability to easily communicate spontaneously or translate thoughts or ideas into meaningful speech or writing.

A 17-year-old male has developed phimosis to the point that he is having difficulty voiding. The nurse should prepare this teenager for:

Circumcision

Which of the following clients is at least risk for rapid bleeding?

Client with a subdural hematoma A subdural hematoma develops in the area between the dura and arachnoid space and is usually the result of a tear in small bridging veins that connect the surface of the cortex to dural sinuses. This is a slow source of bleeding, thus the client with a subdural hematoma is less likely to develop a rapid bleed than a client with an epidural hematoma, red stroke, and/or subarachnoid hemorrhage.

The nurse is working in the emergency room. One client's presenting symptoms include the worst headache ever, nuchal rigidity and nausea. Another client's presenting symptoms include fever, stiff back, and positive Kernig's sign. Which client should the nurse assess first?

Client with the worst headache, nuchal rigidity and nausea The nurse should assess the client with presenting symptoms of worst headache ever, nuichal rigidity and nausea because these are presenting signs of aneurysmal subarachnoid hemorrhage from a ruptured cerebral aneurysm. The client with the presenting symptoms of fever, stiff back, and positive Kernig's sign is experiencing symptoms of meningitis. Ruptured cerebral aneurysm is a medical emergency with a higher priority than meningitis.

A nurse is conducting a medication reconciliation for a 79-year-old man who has just relocated to the long-term care facility. The nurse notes that the man has been taking colchicine (Colcrys) on a regular basis. This aspect of the man's medication regimen should signal the nurse to the possibility that he has a diagnosis of

Colchicine (Colcrys), the prototype agent for the treatment and prevention of gout, is the most commonly administered antigout medication. Colchicine is not indicated in the treatment of osteoarthritis, IBD, tendonitis, or bursitis.

A nurse on a neurology unit is assessing a female brain-injured client. The client is unresponsive to speech, and her pupils are dilated and do not react to light. She is breathing regularly but her respiratory rate is 45 breaths per minute. In response to a noxious stimulus, her arms and legs extend rigidly. What is her level of impairment?

Coma

A nurse on a neurology unit is assessing a female brain-injured client. The client is unresponsive to speech, and her pupils are dilated and do not react to light. She is breathing regularly but her respiratory rate is 45 breaths per minute. In response to a noxious stimulus, her arms and legs extend rigidly. What is her level of impairment? a) Vegetative state b) Brain death c) Delirium d) Coma

Coma The continuum of loss of consciousness is marked by the degree of client responsiveness to stimuli, in addition to the preservation of brain stem reflexes. -Since this client still exhibits a pain response (the extended arms and legs indicate decerebrate posturing), even though her pupils are not responsive to light, she has sustained sufficient brain function that she fails to qualify as brain-dead or in a vegetative state.

compact bone

Compact (cortical) bone: forms the outer shell of a bone; has a densely packed, calcified intercellular matrix that makes cortical bone more rigid than cancellous bone Major component of tubular bones Found along the lines of stress on long bones and forms an outer protective shell on the bones

Which type of seizure begins in a localized area of the brain but may progress rapidly to involve both hemispheres?

Complex partial Complex partial type seizures begin in a localized area of the brain but may progress rapidly to involve both hemispheres.

A patient who is diagnosed with seizures describes feeling a strange sensation before losing consciousness. The family members report that the patient has been smacking their lips prior to having a seizure. Which type of seizure disorder presents with these symptoms?

Complex partial or focal seizure with impairment of consciousness

Wernicke-Korsakoff syndrome is a dementia that is associated with chronic alcoholism. It is caused by a deficiency in thiamine (vitamin B12). What is the most distinctive sign or symptom of this syndrome?

Confabulation Confabulation (i.e., recitation of imaginary experiences to fill in gaps in memory) is probably the most distinctive feature of the disease.

The moderate stage of the progressive degenerative Alzheimer-type dementia is manifested by behaviors that include:

Confusion

bones and bone structure

Connective tissue in which the intercellular matrix has been impregnated with inorganic calcium salts Has great tensile and compressible strength but is light enough to be moved by coordinated muscle contractions

When describing the covering on bones to the students, the instructor asks, "Why is periosteum an important part of the bone covering?" The student responses should mention which of the following? It:

Contains blood vessels that assist with providing nutrition to bone tissue

A client complains to the health care provider that he keeps getting hard skin on the sides of the great and little toes when he wears certain pairs of shoes. This is probably caused by ill-fitting shoes putting pressure on certain areas of the foot and would be called a:

Corns (helomas) are small, well-circumscribed, conical, keratinous thickenings of the skin. They usually appear on the toes from rubbing or ill-fitting shoes. The corn may be either hard (heloma durum) with a central hard, horny core or soft (heloma molle), as commonly seen between the toes. Blisters are circumscribed elevations of the skin caused by fluid under or within the epidermis. A callus is a hyperkeratotic plaque of skin that develops because of chronic pressure or friction. Hematomas are a mass of blood caused by a break in a blood vessel under the skin.

HLA-B27 antigen may be linked to other genes that determine the pathologic autoimmune phenomenon in: A. gout syndrome. B. theumatoid arthritis. C. osteoarthritis syndrome. D. ankylosing spondylitis.

D

The nurse is reviewing the health histories of four clients. Select the client most at risk for developing secondary osteoporosis. a. A 60-year-old female taking prednisone for asthma b. A 22-year-old female taking oral contraceptives c. An 80-year-old male who resides in an assisted living facility d. A 42-year-old male with a healing leg fracture

Correct response: a. A 60-year-old female taking prednisone for asthma Explanation: Corticosteroid (e.g., prednisone) use is the most common cause of drug-related osteoporosis, and long-term corticosteroid use in the treatment of disorders such as rheumatoid arthritis and chronic obstructive lung disease is associated with a high rate of fractures. The prolonged use of aluminum-containing antacids, which increase calcium excretion, and anticonvulsants, which impair vitamin D production, may also contribute to bone loss. Persons with human immunodeficiency virus (HIV) infection or acquired immunodeficiency syndrome (AIDS) who are being treated with antiretroviral therapy are also at risk. The other options would not cause osteoporosis.

The nurse is conducting a community prevention program on osteoporosis. The nurse determines that the participants are understanding the information when they identify that premature osteoporosis is being seen increasingly in female athletes because of an increased prevalence of which of these? a. Amenorrhea b. High protein intake c. Abnormal body fat d. Osteoarthritis

Correct response: a. Amenorrhea Explanation: The female athlete triad, a pattern of disordered eating that leads to amenorrhea and eventually premature osteoporosis, is being seen increasingly in female athletes because of an increased prevalence of eating disorders. Poor nutrition, combined with intense exercise training, can lead to very low percentage of body fat and an energy deficit that causes a lack of ovarian estrogen production and secondary amenorrhea. The lack of estrogen combined with the lack of calcium and vitamin D from dietary deficiencies results in a loss of bone density and increased risk of fractures. Older athletes are high risk for osteoarthritis, a degenerative joint disorder that is unrelated loss of bone density.

The nurse is caring for a patient with rheumatoid arthritis (RA). Which of the following assessment findings does the nurse expect? (Select all that apply.) a. Fatigue b. Increased appetite c. Flushed skin d. Weight loss e. Anorexia

Correct response: a. Fatigue d. Weight loss e. Anorexia Explanation: RA is characterized by weight loss, generalized aching, anorexia, fatigue, as well as joint changes such as pain and stiffness. Increased appetite and flushed skin are not symptoms of RA.

A daughter is concerned because her elderly parent has been diagnosed with osteomalacia. The daughter asks the nurse why this happened. The best response would be that: a. Intestinal absorption slows as a natural aging process b. Consumption of a diet high in calcium c. Absorption of too much vitamin D d. Not using any sunscreens to help with absorption

Correct response: a. Intestinal absorption slows as a natural aging process Explanation: The incidence of osteomalacia is high among the elderly because of diets deficient in calcium and vitamin D, a problem often compounded by the intestinal malabsorption that accompanies with aging. Melanin is extremely efficient in absorbing UVB radiation; thus, decreased skin pigmentation markedly reduces vitamin D synthesis, as does the use of sunscreens. Osteomalacia also may occur in persons on long-term treatment with medications such as anticonvulsants (e.g., phenytoin, carbamazepine, valproate) that decrease the activation of vitamin D in the liver.

Which of the following pathophysiological phenomena would be most indicative of ankylosing spondylitis? a. Loss of motion in the spinal column and eventual kyphosis. b. A progressive loss of range of motion in knee and hip joints. c.A facial "butterfly rash" and multi-organ involvement. d. Decreased bone density in long bones.

Correct response: a. Loss of motion in the spinal column and eventual kyphosis. Explanation: The characteristic trait of ankylosing spondylitis is progressive loss of spinal ROM and eventual kyphosis. Synovial joint involvement is not associated with the disease and a butterfly rash and multisystem involvement are associated with SLE. Decreased bone density does not normally accompany ankylosing spondylitis.

The nurse is caring for a patient with newly diagnosed systemic lupus erythematosus (SLE). Which of the following over-the-counter medications does the nurse recognize is useful in treating inflammation, arthritis, and pleuritis? a. Nonsteroidal anti-inflammatory drugs (NSAIDs) b. Hydroxychlorquine c. Cycloposphamide d. Belimumab

Correct response: a. Nonsteroidal anti-inflammatory drugs (NSAIDs) Explanation: Treatment with medications may be as simple as a drug to reduce inflammation, such as an NSAID. NSAIDs can control fever, arthritis, and mild pleuritis.

Hepatocellular cancer usually has a poor prognosis due in part to which of the following factors? A. Surgical options do not exist because removal of all or part of the liver is a threat to health. B. Liver cancer typically metastasizes at a much earlier stage than other cancers. C. Liver tumors are poorly differentiated due to the low density of hepatic tissue. D. The nonspecific symptomatology of liver cancer leads to a diagnosis at a late stage.

D

A 68 year-old woman has had her mobility and quality of life severely affected by rheumatoid arthritis (RA). Place the following pathophysiological events involved in her health problem in the correct order that they most likely occurred. Use all the options. a. Inflammatory response b. Interaction between rheumatoid factor (RF) and IgG c. T-cell mediated immune response d. Pannus invasion e. Destruction of articular cartilage

Correct response: a..Inflammatory response b. Interaction between rheumatoid factor (RF) and IgG c. T-cell mediated immune response d. Pannus invasion e. Destruction of articular cartilage Explanation: RA is thought to begin with a T-cell mediated immune response which precipitates interaction between IgG and RF that constitutes an immune response. Pannus invasion is one consequence of this interaction, the ultimate result of which is destruction of cartilage.

A nursing instructor is teaching students about metabolic bone diseases. She realizes a need for further instruction when one student states which of the following? a. "Osteopenia is a condition common to all metabolic bone diseases." b. "Osteopenia is not a diagnosis." c. "Osteopenia is a condition that is only common in the elderly." d. "Osteopenia can be caused by osteoporosis."

Correct response: c. "Osteopenia is a condition that is only common in the elderly." Explanation: Osteopenia is a condition that is common to all metabolic bone diseases. It is not a diagnosis but a term used to describe an apparent lack of bone seen on an x-ray. It can be seen in any age and can be caused by osteoporosis, osteomalacia, malignancies, and endocrine disorders.

The nurse performing a musculoskeletal assessment of a patient with osteoarthritis would evaluate the presence of which of the following as a normal finding in this patient? a. Spongy joints b. Cartilage hypertrophy c. Crepitus and grinding d. Systemic inflammation

Correct response: c. Crepitus and grinding Explanation: In osteoarthritis (OA) syndrome, crepitus and grinding may be evident when the osteoarthritic joint is moved. OA joint enlargement results from new bone formation and the joint feels hard, in contrast to the soft, spongy feeling characteristic of the joint in rheumatoid arthritis (RA). The person with ankylosing spondylitis typically reports low back pain, which becomes worse when resting, particularly when lying in bed. Cartilage atrophy develops as the pain of OA limits movements, but is rapidly reversible with a gradual increase in activity; impact exercise during the period of remobilization can cause serious cartilage damage. Laboratory studies for OA usually are normal because the disorder is not a systemic disease.

The nurse is assessing a patient with ankylosing spondylitis (AS). Which of the following does the nurse expect to find? a. Joint contractures b. Facial butterfly rash c. Lower back pain d. Bruises on the lower extremities

Correct response: c. Lower back pain Explanation: The patient with AS has an inflammatory erosion of the sites where tendons and ligaments attach to bone. The disease progresses with bilateral involvement of the sacroiliac joints and produces lower back pain. Joint contractures, butterfly rash, and bruises are not characteristic of this disease.

When teaching a group of nursing students about rheumatic disorders, a nurse emphasizes which of the following important differences when caring for the older adult? a. Need for pain relief b. Stiffness in the morning c. Risk for falls d. Muscle weakness

Correct response: c. Risk for falls Explanation: The pain, stiffness, and muscle weakness affect daily life, often threatening independence and quality of life. Symptoms of the rheumatic diseases can also have an indirect effect on and even threaten the duration of life for older adults. The weakness and gait disturbance that often accompany rheumatic diseases can contribute to the likelihood of falls and fractures.

A nurse is caring for a patient admitted with a malar rash on the nose and cheeks. The nurse recognizes that this rash is characteristic of which of the following disease processes? a. Rheumatoid arthritis b. Sarcoidosis c. Systemic lupus erythematosus d. Scleroderma

Correct response: c. Systemic lupus erythematosus Explanation: The acute skin lesions include the classic malar or "butterfly" rash on the nose and cheeks.

Ankylosing spondylitis is a disease that typically manifests in late adolescence and early adulthood. What is the characteristic of the pain in ankylosing spondylitis? a.Worse when active b. Worse when sitting c. Worse when lying d. Worse when standing

Correct response: c. Worse when lying Explanation: The pain, which becomes worse when resting, particularly when lying in bed, initially may be blamed on muscle strain or spasm from physical activity. The other answers are incorrect.

A 77-year-old woman has been admitted to the geriatric medical unit of the hospital for the treatment of pneumonia. The nurse providing care for the client notes the presence of nasal calcitonin, vitamin D, and calcium chloride on the client's medication administration record. The nurse should conclude that this client likely has a history of: a. Scleroderma b. Osteoarthritis c. Rheumatoid arthritis d. Osteoporosis

Correct response: d. Osteoporosis Explanation: Common pharmacologic treatments for osteoporosis include nasal calcitonin, vitamin D supplements, and calcium supplements. This combination of drugs does not address the etiology or manifestations of scleroderma, osteoarthritis, or rheumatoid arthritis.

The nurse is caring for a patient diagnosed with osteoarthritis (OA). Which of the following does the nurse teach the patient about the disease? a. "OA is a disease of the weight-bearing joints." b. "OA typically occurs in patients between 20 and 40 years of age." c. "OA is an autoimmune disorder." d. "OA is a genetic disorder."

Correct response: a. "OA is a disease of the weight-bearing joints." Explanation: OA is a slowly progressive destruction of articular cartilage of weight-bearing joints and fingers of older adults and the joints of younger people who have experienced trauma.

A client with confirmed low bone density asks the nurse if there is anything she can to decrease the risk of trauma. The best response would be: a. Brisk walking three times per week on a flat surface b. High-impact aerobic exercise for 1 hour three times per week c. Lawn bowling for 1 hour per week d. Running 1 mile per day with good athletic shoes

Correct response: a. Brisk walking three times per week on a flat surface Explanation: Weight-bearing exercises such as walking, jogging, rowing, and weight lifting are important in the maintenance of bone mass. The other options place the person at risk for injury if the bones are weakened.

In contrast to osteoporosis, osteomalacia causes _______ without the loss of bone matrix. A. Stress fractures B. Calcium excretion C. Bone cortex thinning D. Defective mineralization

D

A patient recently diagnosed with rheumatoid arthritis (RA) tells the nurse she is glad there is nothing "really wrong with her" but some joint swelling. Which of the following information should the nurse tell the patient about RA? a. "Joint involvement is typically the only manifestation of the disease in most clients." b. "Extra-articular manifestations may include anemia and deformities of the affected joints." c. "The disease can be reversed with medication and exercise." d. "Myocardial infarction develops in clients who have had the disease for an extended period."

Correct response: b. "Extra-articular manifestations may include anemia and deformities of the affected joints." Explanation: Although characteristically a joint disease, RA can affect a number of other tissues including red blood cells, blood vessels, lungs, and heart. Treatment goals for a person with RA are to prevent and/or reduce the pain, decrease stiffness and swelling, maximize mobility, and possibly halt the progression, but not cure the disease.

Which of the following signs and symptoms should prompt a 29-year-old woman's primary care provider to assess for systemic lupus erythematosus (SLE)? a. Chronic nausea and vomiting that is unresponsive to antiemetics b. Joint pain and increased creatinine and blood urea nitrogen c. A history of thromboembolic events and varicose veins d. Dysmenorrhea and recent spontaneous abortion

Correct response: b. Joint pain and increased creatinine and blood urea nitrogen Explanation: Renal involvement occurs in approximately one half to two thirds of persons with SLE, and arthralgia is a common early symptom of the disease. Although the manifestations of SLE are diffuse, these do not typically include alterations in hemostasis, gastrointestinal symptoms, dysmenorrhea, or miscarriage.

A nurse is assessing a patient with rheumatoid arthritis (RA). What does the nurse expect to find when evaluating the patient's joints? (Select all that apply.) a. Asymmetrical joint involvement b. Joint pain lasting for 30 minutes to several hours c. Decreased joint motion d. Nodules on joints

Correct response: b. Joint pain lasting for 30 minutes to several hours c. Decreased joint motion d. Nodules on joints Explanation: Patients with RA generally have joint pain lasting for 30 minutes to several hours, decreased joint motion, and nodules on joints. The involvement is symmetrical, occurring on both sides at once.

Due to her progressing osteoarthritis (OA), an 80 year-old woman is no longer able to perform her activities of daily living without assistance. Which of the following phenomena most likely underlies the woman's situation? a. Inappropriate T-cell mediated immune responses have resulted in articular cartilage degeneration. b. Loss of articular cartilage and synovitis has resulted from inflammation caused when joint cartilage attempted to repair itself. c. Excessive collagen deposits have accumulated in the woman's synovial joints. d. Bone overgrowth in synovial joints has resulted in fusing of adjacent bones that normally articulate.

Correct response: b. Loss of articular cartilage and synovitis has resulted from inflammation caused when joint cartilage attempted to repair itself. Explanation: The joint changes associated with osteoarthritis, which include a progressive loss of articular cartilage and synovitis, results from the inflammation caused when cartilage attempts to repair itself, creating osteophytes or spurs. These changes are accompanied by joint pain, stiffness, limitation of motion, and in some cases by joint instability and deformity. Immune etiology is more associated with rheumatoid arthritis and collagen deposits are characteristic of scleroderma. Bones do not tend to fuse in the pathogenesis of OA.

A patient presents with joint pain and suspected rheumatoid arthritis (RA). Which of the following is the appropriate nursing action? a. Increase in the client's activity to keep joints mobile b. Muscle-strengthening exercises to support joints c. Intravenous narcotics d. Admission to the intensive care unit for monitoring

Correct response: b. Muscle-strengthening exercises to support joints Explanation: Patients with RA can focus on muscle strengthening to support joints. Range-of-motion exercises and isometric exercises may be used to strengthen muscles. Rest, rather than increased activity, is beneficial. IV narcotics are not generally used, and admission to the intensive care unit is not needed. Heat and cold modalities can be used as well as teaching the patient about proper body mechanics.

Which of the following measures should a public health nurse recommend to middle-aged women to reduce their chances of developing osteoporosis later in life? a. Weight control and daily use of low-dose corticosteroids b. Genetic testing and range-of-motion exercises c. Calcium supplementation and regular physical activity d. Increased fluid intake and use of vitamin D supplements

Correct response: c. Calcium supplementation and regular physical activity Explanation: Although the use of vitamin D supplements may be of preventative value for some clients, the primary prevention measures for osteoporosis include calcium supplementation and regular exercise. Genetic testing and increased fluid intake are not relevant measures, and corticosteroids are a risk factor for osteoporosis.

A patient has a severe reaction to poison ivy. Which of the following medications will be most effective?

Corticosteroids are used to treat the inflammation present in many dermatologic conditions. They are most often applied topically but also may be given orally or parenterally. Emollients and analgesics would be ineffective. Antihistamines are not administered topically.

Knowing that she is a carrier for Duchene muscular dystrophy (DMD), a pregnant woman arranged for prenatal genetic testing, during which her child was diagnosed with DMD. As her son develops, the woman should watch for which of the following early signs that the disease is progressing? A. Impaired sensory perception & frequent wounds. B. Spasticity & hypertonic reflexes C. Muscle atrophy with decreased coordination. D. Frequent falls & increased muscle size.

D

Which one of the following blood tests reflects the glomerular filtration rate (GFR) and is used to estimate renal function?

serum creatinine

The nurse knows that which of the following treatment plans listed below is most likely to be prescribed after a computerized tomography (CT) scan of the head reveals a new-onset aneurysmal subarachnoid hemorrhage? a) Craniotomy and clipping of the affected vessel. b) STAT administration of tissue-type plasminogen activator (tPA). c) Administration of a diuretic such as mannitol to reduce cerebral edema and ICP. d) Monitoring in ICU for signs and symptoms of cerebral insult.

Craniotomy and clipping of the affected vessel. Surgery for treatment of aneurysmal subarachnoid hemorrhage involves craniotomy and inserting a specially designed silver clip that is tightened around the neck of the aneurysm. Administration of tPA would exacerbate bleeding and a diuretic would not address the issue of bleeding. Monitoring alone would be an insufficient response given the severity of the problem.

A transplant nurse is aware of the need to closely monitor a patient's serum levels of cyclosporine. In order to do this, the nurse should ensure that blood is drawn at what time?

Cyclosporine has a very narrow therapeutic index; therefore, prescribers use serum drug levels to regulate cyclosporine dosing, and close monitoring is necessary. They use blood levels measured 2 hours after a dose for dosage adjustments.

A 26-year-old woman has sought care for increasing pain at the back of her ankle & at the bottom of her foot over the past 2 weeks. The patient states that she is generally in good health, although she completed a course of antibiotics for a chlamydial infection 6 weeks earlier. This patient's recent history suggest the pssibility of: A. systemic sclerosis. B. ankylosing spondylitis. C. osteoarthritis. D. reactive arthritis.

D

A 77-year-old woman has been admitted to the geriatric medical unit of the hospital for the treatment of pneumonia. The nurse providing care for the patient notes the presence of nasal calcitonin, vitamin D, & calcium chloride on the patient's medication administration record. The nurse should conclude that this patient likely has a history of: A. scleroderma. B. osteoarthritis. C. rheumatoid arthritis. D. osteoporosis.

D

A client who has been taking 80 mg of prednisone, a glucocorticoid, each day has been warned by his primary care provider to carefully follow a plan for the gradual reduction of the dose rather than stopping the drug suddenly. What is the rationale for this directive? A. Sudden changes in glucocorticoid dosing may reverse the therapeutic effects of the drug. B. Stopping the drug suddenly may "shock" the HPA axis into overactivity. C. Sudden cessation of a glucocorticoid can result in adrenal gland necrosis. D. Stopping the drug suddenly may cause acute adrenal insufficiency.

D

A patient has started having uncontrolled seizures that are not responding to usual medications. Nursing working with the patient must pay special attention to which of the following priority aspects of this clients care? Assessment of: A. ECG for arrhythmias B. Urine output and continence C. Ability to grasp hands and squeeze on command D. Respiratory status and oxygen saturation

D

A patient has wrist inflammation causing compression of the median nerve in the carpal tunnel. Manifestations of this syndrome include: A. Little finger numbness B. Forearm paresthesia C. Loss of tendon reflexes D. Precision of grip weakness

D

A patient who experienced a traumatic head injury from a severe blow to the back of his head now lives with numerous function deficits, including an inability to maintain steady posture while he is in a standing position, although he is steadier when walking. Which of the following disorders most likely resulted from his injury? A. Cerebellar dystaxia. B. Cerebellar tremor. C. A lower motor neuron lesion. D. A vestibulocerebellar disorder

D

A patient with type 1 diabetes has started a new exercise routine. Knowing there may be some increase risks associated with exercise, the health care provider should encourage the patient to: A. Watch for too rapid weight loss B. Monitor for respiratory disorders C. Be careful that you're not experiencing a rebound hyperglycemia D. Carry a snack with carbs to prevent profound hypoglycemia

D

A thermal bone described as involving the entire epidermis & dermis is classified as: A. full third-degree. B. deep first-degree. C. partial second-degree. D. full-thickness second-degree.

D

Acute gastritis refers to a transient inflammation of the gastric mucosa that is most commonly associated with: Diarrhea Food allergies Gastric reflux Alcohol intake

D

Bones are covered, except at their articular ends, by a ______ membrane. A. canaliculi B. endosteum C. synovial D. periosteum

D

Diabetic and hypertensive retinopathy are both characterized by the appearance of: A. Macular edema B. Cloudy corneas C. Microinfarctions D. Intraretinal hemorrhages

D

Magnetic resonance imaging of a patient's knee has revealed the presence of bursitis. What is the primary purpose of bursae? A. to maintain close articulation between the 2 long bones at a synovial joint. B. to strengthen the attachment between skeletal muscles & bones. C. to strengthen the integrity of the articulating capsule. D. to prevent friction at a tendon in a synovial joint.

D

Metastatic bone disease is most closely associated with: A. cervical cancer & ovarian cancer. B. acute myelogenous leukemia (AML) & malignant melanoma. C. non-Hodgkin lymphoma & bladder cancer. D. breast cancer & prostate cancer.

D

Myasthenia gravis is characterized by muscle weakness caused by antibody-mediated destruction of: A. Periorbital muscles B. thymus gland cells C. Skeletal muscle fibers D. Acetylcholine receptors

D

Shortly after a closed fracture has occurred, early manifestations include localized: A. Tetany B. Deformity C. Necrosis D. Numbness

D

The condition that contributes to the pathology of all metabolic bone diseases is: A. impaired vitamin D synthesis. B. osteosarcoma. C. infection. D. osteopenia.

D

The intracranial volume that is most capable of compensating for increasing intracranial pressure is the: A. Brain cell tissue B. Intravascular blood C. Surface sulci fluid D. Cerebrospinal fluid

D

The transition of a melanoma from radial growth to vertical growth is associated with: A. failure of the integrity of the basement membrane. B. involvement of the nonkeratinizing cells. C. spontaneous resolution. D. an increased risk of metastasis.

D

Which of the following individuals is experiencing the effects of a primary endocrine disorder? A patient: A. With adrenal cortical insufficiency due to pituitary hyposecretion of ACTH B. Who has hypothyroidism as a result of low TSH production C. Whose dysfunctional hypothalamus has resulted in endocrine imbalances D. Who has low calcium levels because of the loss of his parathyroid gland

D

A long bone, such as the humerus of the upper arm, has which of the following structural characteristics? A) A perichondrium that overlies most of the bone surface B) A durable outer shell made of cancellous bone C) A diaphysis at each end D) An endosteum composed of osteogenic cells

D) An endosteum composed of osteogenic cells

A basketball player fell awkwardly when attempting to claim a rebound, a mishap that resulted in a tear to the anterior cruciate ligament of his left knee. What characteristic of ligaments makes them particularly susceptible to injury? A) Ligaments are incapable of accommodating lateral movement. B) Ligaments are exclusive to the knee, which is the most frequently injured joint. C) Ligaments are superficial to the synovial capsule and are consequently vulnerable to impacts. D) Ligaments are incapable of stretching when exposed to unusual stress.

D) Ligaments are incapable of stretching when exposed to unusual stress.

A patients clavicular fracture has healed in the weeks following a bicycle accident. Which of the following events takes place in the remodelling stage of bone healing? A) Formation of granulation tissue B) Development of fibrocartilage that resembles the appearance of the original bone C) Deposition of mineral salts into the callus D) Reduction in the size of the callus

D) Reduction in the size of the callus

In contrast to osteoporosis, osteomalacia causes _______ without the loss of bone matrix. A) stress fractures B) calcium excretion C) bone cortex thinning D) defective mineralization

D) defective mineralization

Shortly after a closed fracture has occurred, early manifestations include localized: A) tetany. B) deformity. C) necrosis. D) numbness.

D) numbness.

The condition that contributes to the pathology of all metabolic bone diseases is: A) impaired vitamin D synthesis. B) osteosarcoma. C) infection. D) osteopenia.

D) osteopenia.

A 77-year-old woman has been admitted to the geriatric medical unit of the hospital for the treatment of pneumonia. The nurse providing care for the patient notes the presence of nasal calcitonin, vitamin D, and calcium chloride on the patients medication administration record. The nurse should conclude that this patient likely has a history of: A) scleroderma. B) osteoarthritis. C) rheumatoid arthritis. D) osteoporosis.

D) osteoporosis.

Bones are covered, except at their articular ends, by a ______ membrane. A) canaliculi B) endosteum C) synovial D) periosteum

D) periosteum

A 26-year-old woman has sought care for increasing pain at the back of her ankle and the bottom of her foot over the past 2 weeks. The patient states that she is generally in good health, although she completed a course of antibiotics for a chlamydial infection 6 weeks earlier. This patients recent history suggests the possibility of: A) systemic sclerosis. B) ankylosing spondylitis. C) osteoarthritis. D) reactive arthritis.

D) reactive arthritis.

Cancellous bone is relatively light, but its lattice-like structure gives it: A) rigidity. B) thickness. C) a growth plate. D) tensile strength.

D) tensile strength.

A 41 year-old woman was diagnosed with multiple sclerosis (MS) seven years ago and is sharing her story with members of an MS support group, many of which have been diagnosed recently. Which of the following aspects of her health problem should the woman warn others to expect at some point in the progression of the disease? Select all that apply.

Debilitating fatigue Progressive loss of visual acuity Loss of mental acuity

A patient suffering global cerebral ischemia a week after a suicide attempt by hanging is in the intensive care unit receiving treatment. The parent asks the nurse why it is necessary to keep the patient paralyzed with medications and on the ventilator. The most appropriate response would be that these therapies do which of the following?

Decrease metabolic needs and increase oxygenation The general goal of treatment with global cerebral ischemia is to decrease metabolic needs and increase oxygenation to the injured cerebral tissue. Artificial ventilation provides appropriate oxygenation; keeping the patient paralyzed decreases the body's metabolic needs.

A patient suffering global cerebral ischemia a week after a suicide attempt by hanging is in the intensive care unit receiving treatment. The parent asks the nurse why it is necessary to keep the patient paralyzed with medications and on the ventilator. The most appropriate response would be that these therapies do which of the following? a) Increase oxygen demands and metabolic needs b) Decrease the patient's ability to attempt suicide again c) Decrease metabolic needs and increase oxygenation d) Decrease intracranial fluid volumes and pressures

Decrease metabolic needs and increase oxygenation The general goal of treatment with global cerebral ischemia is to decrease metabolic needs and increase oxygenation to the injured cerebral tissue. Artificial ventilation provides appropriate oxygenation; keeping the patient paralyzed decreases the body's metabolic needs.

A 70-year-old female client comes to the clinic with back pain. An x-ray reveals vertebral fractures and she is diagnosed with osteoporosis. Which of the following factors most likely contributed to her condition?

Decreased estrogen levels

John has been hit in the head with a baseball bat. The nurse knows which would be considered the earliest indicator of a brain injury?

Decreased level of consciousness

The health care provider is concerned that a client may be at risk for problems with cerebral blood flow. The most important data to assess would be:

Decreased level of oxygen Regulation of blood flow to the brain is controlled largely by autoregulatory or local mechanisms that respond to the metabolic needs of the brain. Metabolic factors affecting cerebral blood flow include an increase in carbon dioxide and hydrogen ion concentrations; cerebral blood flow is affected by decreased O2 levels and increased hydrogen ions, carbon dioxide, and PCO2 levels.

The health care provider is concerned that a client may be at risk for problems with cerebral blood flow. The most important data to assess would be: a) Decreased level of carbon dioxide b) Decreased hydrogen ions c) Decreased level of PCO2 d) Decreased level of oxygen

Decreased level of oxygen Regulation of blood flow to the brain is controlled largely by autoregulatory or local mechanisms that respond to the metabolic needs of the brain. Metabolic factors affecting cerebral blood flow include an increase in carbon dioxide and hydrogen ion concentrations; cerebral blood flow is affected by decreased O2 levels and increased hydrogen ions, carbon dioxide, and PCO2 levels.

An 80 year-old female with a diagnosis of osteoporosis receives daily supplements of calcitonin in the form of a nasal spray that she instills each morning. Which of the following phenomenon would her care providers expect to result from her supplementation?

Decreased serum calcium levels

Select the statement that best describes the pathophysiology of Parkinson disease.

Degeneration of the nigrostriatal dopamine system occurs.

A patient's most recent blood work reveals a blood urea nitrogen (BUN) level of 36 mg/dL (normal range 8-25) which of the following factors may have contributed to this finding?

Dehydration

Which of the following factors may adversely affect bone healing and therefore place the client at risk for long-term problems? Select all that apply.

Delayed union is the failure of a fracture to unite within the normal period (e.g., 20 weeks for a fracture of the tibia or femur in an adult). Malunion is caused by inadequate reduction or malalignment of the fracture. For healing to occur, the bone needs to be aligned and immobilized to maintain the alignment during bone remodeling. Individual factors that may delay bone healing are the client's age; current medications; debilitating diseases, such as diabetes and rheumatoid arthritis; local stress around the fracture site; circulatory problems and coagulation disorders; and poor nutrition. Weight bearing helps to maintain some area muscle tone while the bone is immobilized, avoiding muscle atrophy and loss of range of motion.

woven bone

Deposited more rapidly than lamellar bone; low tensile strength Found in developing fetus, part of healing fractures, and in areas near tumors / infections

After evaluating the patient, the physician thinks that his elderly patient is exhibiting signs and symptoms of normal pressure hydrocephalus. Which of the following symptoms would be seen? a) Papilledema b) Headache c) Vomiting d) Disturbances in gait

Disturbances in gait In adults, slowly developing hydrocephalus may produce deficits such as progressive dementia and gait changes. The other options are symptoms of increased ICP seen in acute onset hydrocephalus.

The nurse is caring for an elderly client with hemiplegia following a stroke. While planning the client's care, the nurse knows the client is at risk for which of the following conditions?

Disuse atrophy

A client affected by postural hypotension will likely display what symptoms?

Dizziness and pallor when moved to upright position

A football player who was admitted to the emergency department is awaiting results of a CT scan. The physician suspects injury of the hyaline cartilage. Select the statement that best describes hyaline cartilage.

Does not heal after injury

A patient with Alzheimer's disease (AD) is forgetful and has started to lose interest in social activities. Which of the following treatment routines would be beneficial for the patient?

Donepezil (Aricept) The cholinesterase inhibitor donepezil (Aricept) has been effective in slowing cognitive decline in early stages of AD.

A patient's recent diagnosis of Parkinson's disease has prompted his care provider to promptly begin pharmacologic therapy. The drugs that are selected will likely influence the patient's levels of:

Dopamine

Osteoporosis is a disease caused by demineralization of bone. What is the clinical method of choice for diagnosing osteoporosis?

Dual-energy x-ray absorptiometry (DXA or DEXA) of the spine and hip

A client who sustained a complete C6 spinal cord injury 6 months ago has been admitted to the hospital for pneumonia. The nurse observes the client with diaphoresis above the level of C6 and the blood pressure is 260/140 mm Hg. What is the first intervention the nurse should provide?

Elevate the head of the bed.

Which diagnostic finding has been strongly linked to systematic lupus erythematosus (SLE)?

Elevated anti-nuclear antibodies (ANA)

A child is being seen in the emergency department (ED) after ingesting crayons with lead in them. He is disoriented and having seizures. The provider suspects he has which of the following?

Encephalitis Less frequent causes of encephalitis include ingesting toxic substances such as lead. People experience neurologic disturbances such as lethergy, disorientation, seizures, focal paralysis, delirium and coma.

A child is being seen in the emergency department (ED) after ingesting crayons with lead in them. He is disoriented and having seizures. The provider suspects he has which of the following? a) Encephalitis b) Bacterial meningitis c) Viral meningitis d) Meningioma

Encephalitis Less frequent causes of encephalitis include ingesting toxic substances such as lead. -People experience neurologic disturbances such as lethergy, disorientation, seizures, focal paralysis, delirium and coma. - Bacterial and viral meningitis are caused by bacterial and viral infections. -Meningiomas are a type of brain tumor that are seen in the middle or later years of life.

Loss of coordinated movement and balance is likely due to damage to which of the following?

Extrapyramidal structures

A client has suffered a torn ligament. The client will most likely manifest:

Extreme pain and swelling

A patient is administered methotrexate for the treatment of severe rheumatoid arthritis. Administration of this drug should be performed with particular care because of the associated high risk of

Even in the low doses used in rheumatoid arthritis and psoriasis, methotrexate may cause hepatotoxicity. Consequently, many clinicians recommend serial liver biopsies for patients on long-term, low-dose methotrexate. This drug is not closely associated with bleeding disorders, MI, or stroke.

Excessive activity of the excitatory neurotransmitters and their receptor-mediated effects is the cause of which type of brain injury?

Excitotoxic Excitotoxicity is a final common pathway for neuronal cell injury and death. It is associated with excessive activity of excitatory amino acid neurotransmitters.

canaliculi

Extracellular fluid-filled passageways (canaliculi) permeate the calcified matrix, and connect with the lacunae of adjacent osteocytes Canaliculi serve as communicating channels for the exchange of nutrients and metabolism between the osteocytes and the blood vessels on the surface of the bone layer because diffusion does not occur through the calcified matrix of bone

The nurse assessing for the doll's head response (doll's eye response) in an unconscious client documents which of the following as an abnormal response?

Eyes turn right when head is turned right

The nurse assessing for the doll's head response (doll's eye response) in an unconscious client documents which of the following as an abnormal response? a) Eyes turn up when head is tilted down b) Eyes turn toward right when head is turned to left c) Eyes turn right when head is turned right d) Eyes turn down when head is tilted up

Eyes turn right when head is turned right The normal doll's head response (doll's eye response) is movement of the eyes in conjugate gaze to the opposite side or direction when the head is moved from side to side or up and down. An abnormal response is fixed position of the eyes or movement in the same direction as the head.

The underlying causative problem in Parkinsonism is which of the following?

Failure of dopamine release

Bradykinesia occurring in Parkinson's disease places the Parkinson's client most at risk for which of the following?

Falls and injury

Concussions are listed under the category of focal brain injuries.

False

Examples of superficial viral skin infections include: tinea capitis, tinea corporis, and tinea pedis.

False

Ligaments connect muscles to bones.

False

Neonatal respiratory distress syndrome occurs most commonly in full-term infants True False

False

Orchitis is an infection of the penis.

False

Rheumatic disorders have signs and symptoms that are limited to the skeletal system.

False

Rheumatoid arthritis is a chronic autoimmune system disease that affects a single joint.

False

The dermis is the outermost layer of the skin and covers the body.

False

The radiologist knows that it is normal to see lamellar and woven bone in the adult skeleton.

False

The structure of the skin renders it susceptible to bacterial and fungal infections but resistant to viral infections.

False

The uterus is a thin-walled muscular organ.

False

The nurse notes that the client is experiencing visible squirming and twitching movements that can be seen as flickers under the skin. The nurse would recognize this as:

Fasciculations

The nurse is caring for a client with a fracture in his appendicular skeleton. Which of the following bones is in this skeleton?

Femur

A client with a history of a seizure disorder has been observed suddenly and repetitively patting his knee. After stopping this repetitive action, the client appears confused but is oriented to person and place but not time. What type of seizure did this client most likely experience?

Focal seizure with impairment to consciousness Focal seizures with impairment of consciousness, sometimes referred to as psychomotor seizures, are often accompanied by automatisms or repetitive nonpurposeful activities such as lip smacking, grimacing, patting, or rubbing clothing. Confusion during the postictal period (after a seizure) is common.

A client with a history of a seizure disorder has been observed suddenly and repetitively patting his knee. After stopping this repetitive action, the client appears confused but is oriented to person and place but not time. What type of seizure did this client most likely experience? a) Focal seizure with impairment to consciousness b) Atonic seizure c) Tonic-clonic seizure d) Myoclonic seizure

Focal seizure with impairment to consciousness Focal seizures with impairment of consciousness, sometimes referred to as psychomotor seizures, are often accompanied by automatisms or repetitive nonpurposeful activities such as lip smacking, grimacing, patting, or rubbing clothing. Confusion during the postictal period (after a seizure) is common. -Atonic seizures are characterized by loss of muscle tone, and -myoclonic seizures involve brief involuntary muscle contractions induced by stimuli of cerebral origin. -With tonic-clonic seizures, formerly called grand mal seizures, a person has a vague warning (probably a simple focal seizure) and experiences a sharp tonic contraction of the muscles with extension of the extremities and immediate loss of consciousness.

compact bone

Forms the outer shell of a bone, and the diaphysis of long bones Has a densely packed calcified intercellular matrix that makes it more rigid than cancellous bone

cancellous bone

Found in the interior and epiphyseal ends of long bones Composed of trabeculae, or spicules, of bone that form a lattice-like pattern

osteoprogenitor cells

Found in the periosteum, endosteum, and epiphyseal plat of growing bone Undifferentiated cells that differentiate into osteoblasts Active during normal growth May be activated during adult life during healing of fractures and other injuries Participate in the continual replacement of worn-out bone tissue

perichondrium

Free surfaces of most hyaline cartilage ~ with the exception of articular cartilage ~ are covered by a layer of fibrous connective tissue, the perichondrium

Knowing that she is a carrier for Duchene muscular dystrophy (DMD), a pregnant woman arranged for prenatal genetic testing, during which her child was diagnosed with DMD. As her son develops, the woman should watch for which of the following early signs that the disease is progressing?

Frequent falls and increased muscle size

Knowing that she is a carrier for Duchene muscular dystrophy (DMD), a pregnant woman arranged for prenatal genetic testing, during which her child was diagnosed with DMD. As her son develops, the woman should watch for which of the following early signs that the disease is progressing? Impaired sensory perception and frequent wounds Spasticity and hypertonic reflexes Muscle atrophy with decreased coordination Frequent falls and increased muscle size

Frequent falls and increased muscle size

During a late night study session, a pathophysiology student reaches out to turn the page the page of her textbook. Which of the following components of her nervous system contains the highest level of control of her arm and hand action?

Frontal lobe

Following surgery for a large malignant brain tumor, the nurse should anticipate discussing which further treatment option with the family that may ensure that any remaining cancer cells will be killed?

Gamma knife radiation

The chart of a client admitted because of seizures notes that the seizure activity began simultaneously in both cerebral hemispheres. The nurse should interpret this to mean that the client experienced which of the following?

Generalized seizure When seizure activity begins simultaneously in both cerebral hemispheres, it is considered a generalized seizure.

Which statement is true regarding the development of juvenile idiopathic arthritis?

Generalized stunted growth can occur.

"Bow legs" is another name for:

Genu Varum

An individual affected by a basal-ganglia associated movement disorder is affected by dyskinesias. This is displayed as which of the following?

Grimacing and lip pursing

Which of the following disorders of neuromuscular function typically has the most rapid onset?

Guillain-Barr Syndrome

Immune-mediated attack on the sheaths of well-myelinated axons

Guillain-Barre Syndrome

Which of the following disorders of neuromuscular function typically has the most rapid onset? Duchenne muscular dystrophy (DMD) Guillain-Barré syndrome Parkinson disease Myasthenia gravis

Guillain-Barré syndrome

disorders of neuromuscular function typically has the most rapid onset?

Guillain-Barré syndrome

The client with a suspected diagnosis of osteoarthritis asks the health care provider how the diagnosis will be confirmed. The best response would be:

History and physical examination, x-ray studies, and laboratory findings that exclude other diseases

Which of the following clients' signs and symptoms would allow a clinician to be most justified in ruling out stroke as a cause? An adult:

Has had a gradual onset of weakness, headache and visual disturbances over the last two days. A cardinal trait of the manifestations of stroke is that the onset is sudden, and a gradual onset of symptoms over two days would suggest an alternative etiology.

A 9 year-old boy has been brought to the emergency department by his father who is concerned by his son's recent fever, stiff neck, pain and nausea. Examination reveals a petechial rash. Which of the following assessment questions by the emergency room physician is most appropriate? a) "Does your son have a history of cancer?" b) "Has your son had any sinus or ear infections in the last little while?" c) "Was your son born with any problems that affect his bone marrow or blood?" d) "Is your son currently taking any medications?"

Has your son had any sinus or ear infections in the last little while?" The most common symptoms of acute bacterial meningitis are fever and chills; headache; stiff neck; back, abdominal, and extremity pains; and nausea and vomiting. Risk factors associated with contracting meningitis include otitis media and sinusitis or mastoiditis. -Particular medications, a history of neoplasm and hematopoietic problems would be unlikely to relate directly to his symptoms of meningitis.

The student is examining a wedge of compact bone tissue and its blood distribution. The spaces in the cortex that move parallel through the long axis of the bone and contain blood vessels are known as which of the following?

Haversian canals

Haversian canals

Haversian canals: spaces in the bone of the cortex that move parallel through the long axis of the bone for a short distance, and then branch out to communicate with other similar canals Each canal carries 1-2 blood vessels, lymphatics, and some nerve fibers

A high school senior sustained a concussion during a football game. What signs and symptoms would indicate the presence of postconcussion syndrome in the days and weeks following his injury? ___ and __ __

Headaches and memory lapses

A high school senior sustained a concussion during a football game. Which of the following signs and symptoms would indicate the presence of postconcussion syndrome in the days and weeks following his injury?

Headaches and memory lapses

A college student has been experiencing frequent headaches that he describes as throbbing and complaining of difficulty concentrating while studying. Upon cerebral angiography, he is found to have an arteriovenous malformation. Which of the following pathophysiological concepts are likely responsible for his symptoms? a) Increased tissue perfusion at the site of the malformation. b) High pressure and local hemorrhage of the venous system. c) Hydrocephalus and protein in the cerebral spinal fluid. d) Localized ischemia with areas of necrosis noted on CT angiography.

High pressure and local hemorrhage of the venous system. Answer B is incorrect since arteriovenous malformation is associated with blood vessels and not the fluid within the ventricles of the brain. Answer D is incorrect in that there is blood flow to the area. Ischemia is associated with decreased arterial flow resulting in death to brain tissue. In arteriovenous malformations, a tangle of arteries and veins acts as a bypass between the cerebral arterial and venous circulation, in place of the normal capillary bed. However, the capillaries are necessary to attenuate the high arterial blood pressure before this volume drains to the venous system. As a result, the venous channels experience high pressure, making hemorrhage and rupture more likely; the lack of perfusion of surrounding tissue causes neurologic deficits such as learning disorders. Headaches are severe, and people with the disorder may describe them as throbbing (synchronous with their heartbeat). Increased tissue profusion means that more oxygenated blood is brought to the area which is not the case. The elevated arterial and venous pressures divert blood away from the surrounding tissue, impairing tissue perfusion.

Herniated disks occur when the nucleus pulposus is compressed enough that it protrudes through the annulus fibrosus, putting pressure on the nerve root. This type of injury occurs most often in the cervical and lumbar region of the spine. What is an important assessment technique for a herniated disk in the lumbar region?

Hip flexion test CT scan Straight-leg test

A 60 year-old male office worker presents to a clinic complaining of new-onset of lower back pain that has been worsening over the last six weeks. The nurse knows which of the following components of his physical assessment and history is most indicative of a serious pathological process (like aortic aneurysm or cancer)?

His onset of pain has been gradual and he has no prior history of lower back problems.

A dermatologist is explaining to a client the advantages of using a lotion that can draw out water to the skin surface. The nurse knows the technical term for this lotion is a/an:

Humectants are the additives in lotions, such as α-hydroxy acids and urea, that draw out water from the deeper skin layers and hold it on the skin surface. Emollients are fatty acid-containing lotions that replenish the oils on the skin surface but usually do not leave a residue on the skin. They have a short duration of action and need to be applied frequently. Urea is a nitrogenous substance that has been quite effective in reducing xerosis when combined with lotions. It is a humectant at lower concentrations (10%), but in higher concentrations (20%-30%), it is mildly keratolytic. Occlusives are thick creams that contain petroleum or some other moisture-proof material. They prevent water loss from the skin.

A patient with memory loss is concerned about the possibility that it may be inherited. Which of the following disorders is an inherited dementia?

Huntington's disease Huntington's disease is an inherited disorder with chorea and dementia.

A patient with memory loss is concerned about the possibility that it may be inherited. Which of the following disorders is an inherited dementia? a) Vascular dementia b) Wernicke-Korsakoff syndrome c) Frontotemporal dementia d) Huntington's disease

Huntington's disease Vascular dementia occurs as a result of brain injury from hemorrhage or occlusion. Frontotemporal dementia causes atrophy of the brain but is not hereditary. Wernicke-Korsakoff syndrome is a result of chronic alcoholism. Huntington's disease is an inherited disorder with chorea and dementia.

Hyaline cartilage is a firm but flexible type of connective tissue that is essential for

Hyaline cartilage is essential for growth before and after birth. As long bones grow in length, the deeper layers of cartilage cells in the growth plate multiply and enlarge, pushing the articular cartilage farther away from the metaphysis and diaphysis. Because cartilage has no blood vessels, this tissue fluid allows diffusion with blood vessels outside the cartilage. Cartilage dies if it becomes calcified. Bursae are located near joints and contain synovial fluid; their purpose is to prevent friction on a tendon.

Extreme cerebral edema may cause the brain to herniate into another compartment. Upward herniation from the infratentorial compartment against the aqueduct of Sylvius causes

Hydrocephalus

Only the kidneys can eliminate ____ from the body as a means of regulating body pH, when urine buffers are present.

Hydrogen (H+)

The nurse is suctioning a client with a C3 spinal cord injury when the client's heart rate drops from 86 bpm to 42 bpm. What intervention does the nurse understand should be provided prior to suctioning to prevent this vasovagal response from occurring?

Hyperoxygenate prior to suctioning

hyperthyroidism

Hyperthyroidism is associated with increased levels of vitamin D precursors; and hypothyroidism leads to decreased levels of this metabolite

hypothyroidism

Hyperthyroidism is associated with increased levels of vitamin D precursors; and hypothyroidism leads to decreased levels of this metabolite

The nurse is providing care for a patient who has a diagnosis of kidney failure. Which of the following lab findings is consistent with this patient's diagnosis?

Hypocalcemia

A clinician is assessing the muscle tone of a patient who has been diagnosed with a lower motor neuron (LMN) lesion. Which of the following assessment findings is congruent with the patient's diagnosis? Hypotonia Spasticity Tetany Rigidity

Hypotonia

A clinician is assessing the muscle tone of a patient who has been diagnosed with a lower motor neuron (LMN) lesion. What assessment findings is congruent with the patient's diagnosis?

Hypotonia (less muscle tone)

The nurse is planning an inservice on hypoxia versus ischemia in brain-injured clients. The nurse should include which of the following?

Hypoxia produces a generalized depressive effect on the brain. Hypoxia denotes a deprivation of oxygen with maintained blood flow (perfusion), whereas ischemia is a situation of greatly reduced or interrupted blood flow. Hypoxia produces a generalized depressant effect on the brain. Ischemia interferes with delivery of oxygen and glucose as well as the removal of metabolic wastes.

The nurse is planning an inservice on hypoxia versus ischemia in brain-injured clients. The nurse should include which of the following? a) Ischemia does not interfere with delivery of glucose. b) Ischemia denotes a deprivation of oxygen with maintained perfusion. c) Hypoxia denotes an interruption in blood flow. d) Hypoxia produces a generalized depressive effect on the brain.

Hypoxia produces a generalized depressive effect on the brain. Hypoxia denotes a deprivation of oxygen with maintained blood flow (perfusion), whereas ischemia is a situation of greatly reduced or interrupted blood flow. Hypoxia produces a generalized depressant effect on the brain. Ischemia interferes with delivery of oxygen and glucose as well as the removal of metabolic wastes.

Which of the following statements by the husband of a patient with Alzheimer disease demonstrates an accurate understanding of his wifes medication regimen?

Im really hoping these medications will slow down her mental losses

Which of the following would be considered physiologic characteristics of immature or woven bone? It: Select all that apply.

Immature or woven bone, consisting of trabeculae, looks like poorly organized bone. It is deposited more rapidly than lamellar bone, has low tensile strength, and serves as temporary scaffolding for support. It is found in the developing fetus, in areas surrounding tumors and infections, and as part of a healing fracture. Laminar or mature bone is composed largely of cylinder-shaped units of calcified matrix, called osteons, that are oriented parallel to the long axis of the bone. Functionally, osteons can be thought of as tiny weight-bearing pillars.

A 16 year-old boy has been diagnosed with ankylosing spondylitis. Which of the following etiologies is responsible for his health problem?

Immune response

An elderly resident of an assisted-living facility has had his mobility and independence significantly impaired by the progression of his rheumatoid arthritis (RA). What is the primary pathophysiologic process that has contributed to this client's decline in health?

Immunologically mediated joint inflammation

An individual experiencing spinal cord damage (SCD) is at high risk for developing deep vein thrombosis (DVT) and pulmonary emboli (PE) caused by which of the following?

Impaired mobility Impaired vasomotor tone Hypercoagulation of blood Stasis of blood flow

A family brings a client to the emergency department with increasing lethargy and disorientation. They think the client had a seizure on the drive over to the hospital. The client has been sick with a "cold virus" for the last few days. On admission, the clients' temperature is 102°F. Which other clinical manifestations may lead to the diagnosis of encephalitis?

Impaired neck flexion resulting from muscle spasm Like meningitis, encephalitis is characterized by fever, headache, and nuchal rigidity (impaired neck flexion resulting from muscle spasm), but more often clients also experience neurologic disturbances, such as lethargy, disorientation, seizures, focal paralysis, delirium, and coma

A family is sitting with a patient in the intensive care unit who sustained significant head injuries in a motorcycle accident. They are questioning the nurse about why the patient's eyes open but do not stay open for long. The nurse explains that the patient is probably in which of the following states?

In a stuporous state due to a reticular activated system (RAS) injury Injury to the RAS would be suspected due to the change in the level of consciousness. The RAS and functional cerebral hemispheres are necessary for arousal and wakefulness; damage to either will negatively affect a person's level of consciousness.

A family is sitting with a patient in the intensive care unit who sustained significant head injuries in a motorcycle accident. They are questioning the nurse about why the patient's eyes open but do not stay open for long. The nurse explains that the patient is probably in which of the following states? a) In a stuporous state due to a reticular activated system (RAS) injury b) In a stuporous state due to acidosis c) In an obtunded state due to possible brain injury d) In an obtunded state due to a concussion

In a stuporous state due to a reticular activated system (RAS) injury Injury to the RAS would be suspected due to the change in the level of consciousness. The RAS and functional cerebral hemispheres are necessary for arousal and wakefulness; damage to either will negatively affect a person's level of consciousness.

joint capsule

In a synovial joint, the articulating ends of the bones are not connected directly but are indirectly linked by a strong fibrous capsule (i.e., joint capsule) that surrounds the joint and is continuous with the periosteum (see Fig. 56.7)

When talking to a group of teenagers about ways to protect against skin damage from ultraviolet radiation, the nurse should discuss which of the following tissues/cells?

In addition to the keratinocytes, the epidermis has melanin pigment-producing melanocytes that protect against ultraviolet radiation and Langerhans cells that link the epidermis to the immune system. Subcutaneous tissue contains fat and binds the dermis to underlying body tissues. The third keratinocyte layer, the stratum granulosum, is composed of flat cells containing protein granules called keratohyalin granules.

Which of the following skin disorders is likely to result from the localized lack of melanin production by melanocytes?

In cases of vitiligo, depigmented areas may contain no melanocytes, greatly altered or decreased amounts of melanocytes, or, in some cases, melanocytes that no longer produce melanin. Melasma results from increased pigmentation, and neither bullae nor rashes are consequences of alteration in melanocyte function.

epiphysis

In growing bones, the part of the bone shaft that funnels out as it approaches the epiphysis is called metaphysis ~ composed of bony trabeculae that have cartilage cores In the child, the epiphysis is separated from the metaphysis by the cartilaginous growth plate After puberty, the metaphysis and the epiphysis merge, and the growth plate is obliterated

The nurse is assessing a client diagnosed with myasthenia gravis (MG). The nurse would expect the assessment to include:

Inability to move eyes in multiple positions

A client has developed global ischemia of the brain. The nurse determines this is:

Inadequate to meet the metabolic needs of the entire brain Global ischemia occurs when blood flow is inadequate to meet the metabolic needs of the entire brain. The result is a spectrum of neurologic disorders reflecting diffuse brain dysfunction.

A client has developed global ischemia of the brain. The nurse determines this is: a) Inadequate perfusion of the nondominant side of the brain b) Inadequate perfusion to the dominant side of the brain c) Inadequate perfusion of the right side of the brain d) Inadequate to meet the metabolic needs of the entire brain

Inadequate to meet the metabolic needs of the entire brain Global ischemia occurs when blood flow is inadequate to meet the metabolic needs of the entire brain. The result is a spectrum of neurologic disorders reflecting diffuse brain dysfunction.

Following a head injury on the football field, the medical team is assessing the player for injury. One of the earliest signs of decreased level of consciousness to assess for would be:

Inattention

The elderly nursing home resident has a fractured humerus. In order to promote healing, the physician recommends which of the following?

Including fish in the diet on a routine basis

A patient has recently undergone successful extracorporeal shock wave lithotripsy(ESWL) for the treatment of renal calculi. Which of the following measures should the patient integrate into his lifestyle to reduce the risk of recurrence?

Increased fluid intake and dietary changes

A female tennis player has suffered an injury to her shoulder that has affected her bursae in the joint. Which of the following consequences would be most expected from this aspect of her injury?

Increased friction on the tendons of the shoulder joint.

vitamin D function

Increased intestinal reabsorption of calcium and phosphate, increased bone resorption. Increases intestinal absorption of phosphate Increases renal excretion of phosphate Can increase bone resorption

calcitonin function

Increases renal excretion of phosphate Increases renal excretion of calcium Decreases bone resorption Decreases serum calcium levels with pharmacologic doses Decreases serum phosphate levels with pharmacologic doses

In a patient diagnosed with osteomalacia, which of the following would be recommended to aid with improvement of the condition?

Increasing dietary consumption of vitamin D

The nurse is teaching a client diagnosed with osteomalacia about treatments to improve the condition. The best information for the nurse to provide would be:

Increasing dietary consumption of vitamin D

The geriatrician providing care for a 74 year-old man with diagnosis of Parkinson disease has recently changed the client's medication regimen. What is the most likely focus of the pharmacologic treatment of the man's health problem?

Increasing the functional ability of the underactive dopaminergic system.

Myasthenic crisis may be precipitated by which of the following? Select all that apply.

Infection Pregnancy Surgery

Which of the following pathophysiologic processes occurs in cases of bacterial meningitis?

Inflammation allows pathogens to cross into the cerebrospinal fluid

The health care provider notes that a client has a history of tendonitis. Select the best description of tendonitis.

Inflammation of a tendon

synovium

Inner membrane, the synovium: Surrounds the tendons that pass through the joints and the free margins of other intra-articular structures such as ligaments and menisci Forms folds that surround the margins of articulations but do not cover the weight-bearing articular cartilage Folds permit stretching of the synovium so that movement can occur without tissue damage

A nurse is caring for a patient with discoid lupus. Which of the following systems does the nurse set as a priority for the nursing assessment?

Integumentary assessment

Reflexes are basically "hard-wired" into the CNS. Anatomically, the basis of a reflex is an afferent neuron that synapses directly with an effector neuron that causes muscle movement. Sometimes the afferent neuron synapses with what intermediary between the afferent and effector neurons?

Interneuron

Which of the following neurons connect sensory and motor neurons?

Interneurons

lamella

Interstitial lamella lies between the osteons as remnants of previous concentric lamellae Circumferential lamella follow the entire inner and outer shaft of a long bone (appearing much like the growth rings of a tree)

The nurse is explaining to a client that there are "cushions" in his back to absorb any jarring movement of the spine. Which of the following anatomical structures is the nurse referring to?

Intervertebral disks

intra-articular menisci

Intra-articular menisci: fibrocartilage structures that develop from portions of the articular disk that occupied the space between the articular cartilage surfaces during fetal development May extend part way through the joint and have a free inner border (as in the lateral and medial surfaces of the knee); or may extend through the joints separating it into two cavities (as in the sternoclavicular joint) The menisci of the knee joint may be torn as a result of an injury

Generalized convulsive status epilepticus is a medical emergency caused by a tonic-clonic seizure that does not spontaneously end, or recurs in succession without recovery. What is the first-line drug of choice to treat status epilepticus?

Intravenous lorazepam

A patient is diagnosed with salicylate overdose. Which of the following medications will be administered for the treatment of salicylate overdose?

Intravenous sodium bicarbonate produces alkaline urine in which salicylates are more rapidly excreted in patients with salicylism.

A child has been hospitalized for the treatment of hematogenous osteomyelitis. The defining characteristic of this type of osteomyelitis is:

Introduction of microorganisms from the bloodstream

irregular bones

Irregular bones: because of their shape, cannot be classified in any of the previous groups ~ includes bones such as the vertebrae and the jaw bones

A patient's emergency magnetic resonance imaging (MRI) has been examined by the physician and tPA has been administered to the patient. What was this patient's most likely diagnosis? ___ __

Ischemic stroke (TPA- tissue plasminogen activator...to plasmin... administered to break up clots)

What is the purpose of a bursae?

It is a fluid-filled sac that preventions friction on a tendon

hinge joints

Joints that can bend and straighten but cannot rotate; they restrict motion to one plane.

Which disease presents with an inflammatory myopathy primarily involving skin and muscle with a characteristic rash?

Juvenile dermatomyositis

The health care provider is assessing a child and suspects an inflammatory myopathy. The assessment data include muscle weakness and a characteristic rash, primarily involving the skin and muscle. The health care provider would document this as:

Juvenile dermatomyositis (JDMS)

A nurse is caring for a patient with ankylosing spondylitis. For which of these associated symptoms does the nurse assess?

Kyphosis

fibrous joints

Lack a synovial cavity, the bones are held together by fibrous connective tissue and allows for little or no movement. Example: Suture (composed of a thin layer of dense fibrous connective tissue which connects skull bones).

The goal of treatment in _____ is to reduce deformity and preserve the integrity of the femoral head.

Leff-Calve- Perthes disease

A nurse is caring for a client who twisted his ankle while running. Tests reveal damaged connective tissue that connects the movable bones of the joint. Based on this finding, the nurse prepares to teach the client about which of the following anatomical structure that is injured?

Ligaments

ligament

Ligaments: join one bone to its articulating mate Fibrous thickenings of the articular capsule Vary in size and shape, depending on their specific role Most ligaments are considered inelastic; but are pliable enough to permit joint movement Ligaments tend to tear (rather than stretch) when exposed to excess stress Torn ligaments are extremely painful; accompanied by local swelling

Based on assessment parameters for motor response on the Glasgow Coma Scale, to which client should the nurse assign a score of 5?

Localizes pain

osteocytes

Mature bone cells actively involved in maintaining the bony matrix

Due to her progressing osteoarthritis (OA), an 80 year-old woman is no longer able to perform her activities of daily living without assistance. Which of the following phenomena most likely underlies the woman's situation?

Loss of articular cartilage and synovitis has resulted from inflammation caused when joint cartilage attempted to repair itself.

Which of the following pathophysiological phenomena would be most indicative of ankylosing spondylitis?

Loss of motion in the spinal column and eventual kyphosis.

A client with a spinal cord injury has experienced contractures and destructive changes in the joints of the lower extremities. The nurse determines which of the following is the most likely cause?

Loss of proprioception and reflex control of the muscles

A client has developed spinal shock. The most important assessment for the nurse to perform would be:

Loss of tendon reflexes

The nurse is assessing a patient with ankylosing spondylitis (AS). Which of the following does the nurse expect to find?

Lower back pain

The emergency room doctor suspects a client may have bacterial meningitis. The most important diagnostic test to perform would be:

Lumbar puncture The diagnosis of bacterial meningitis is confirmed with abnormal CSF findings. Lumbar puncture findings, which are necessary for accurate diagnosis, include a cloudy and purulent CSF under increased pressure.

The emergency room doctor suspects a client may have bacterial meningitis. The most important diagnostic test to perform would be: a) Lumbar puncture b) Sputum culture c) CT of the head d) Blood cultures

Lumbar puncture The diagnosis of bacterial meningitis is confirmed with abnormal CSF findings. Lumbar puncture findings, which are necessary for accurate diagnosis, include a cloudy and purulent CSF under increased pressure. The other options do not confirm the diagnosis.

T cell-mediated demyelination sclerotic plaques CNS demylenation

MS

In addition to keratinocytes, the epidermis has _____ cells that protect against ultraviolet radiation.

Melanocytes

A client has been diagnosed with Guillain-Barre syndrome and is being treated in the intensive care unit. The client is experiencing rapidly ascending paralysis. What is the nurse's priority intervention when caring for this client?

Maintenance of the client's airway

What role do the basal ganglia play in cognition?

Monitor sensory information coming into the brain and apply it to information stored in the memory

hyaline cartilage

Most common type of cartilage; it is found on the ends of long bones, ribs, and nose have more collagen!!

A feature of rheumatoid arthritis that differentiates if from other forms of inflammatory arthritis in the development of:

Pannus

A demyelinating disease of the CNS, characterized by exacerbations and remissions over many years

Multiple Sclerosis (MS)

Restoration of the integrity of myelin sheaths would likely result in a slowing or stopping of the progression of:

Multiple Sclerosis (MS)

Restoration of the integrity of myelin sheaths would likely result in a slowing or stopping of the progression of: Amyotrophic lateral sclerosis (ALS) Multiple sclerosis (MS) Duchenne muscular dystrophy (DMD) Paralysis caused by Clostridium botulinum

Multiple sclerosis (MS)

Disorder of transmission at the neuromuscular junction that affects communication between the motor neuron and the innervated muscle cell:

Myasthenia Gravis

Antibiotics such as gentamicin can produce a disturbance in the body that is similar to botulism by preventing the release of acetylcholine from nerve endings. In persons with preexisting neuromuscular transmission disturbances, these drugs can be dangerous. What disease falls into this category?

Myasthenia gravis

Common manifestations of acute meningococcal meningitis, a highly contagious and lethal form of meningitis, include:

Neck pain

A child with a diagnosis of Legg-Calvé-Perthes disease will exhibit:

Necrosis of the proximal femoral head

A 60-year old has been recently diagnosed with multiple sclerosis, a disease in which the oligodendrocytes of the patient's central nervous system are progressively destroyed. Which physiologic process within the neurological system is most likely be affected by the disease process?

Nerve conduction

Hypoxic injury will result in which of the following effects on the brain?

Neuronal cell injury and death Neuronal cell injury and death is directly caused by hypoxic injury. The others are specific to several other brain injury types.

Hypoxic injury will result in which of the following effects on the brain? a) Can be focal or global with only one part of the brain being underperfused or all of the brain being compromised b) Depends on the brain's compensatory mechanisms and the extent of the swelling c) Neuronal cell injury and death d) Clouding of consciousness, bilaterally small pupils (approximately 2 mm in diameter) with a full range of constriction, and motor responses to pain that are purposeful or semipurposeful (localizing) and often asymmetric

Neuronal cell injury and death Neuronal cell injury and death is directly caused by hypoxic injury. The others are specific to several other brain injury types.

The brain stem contains gray matter that is made up of which of the following?

Neurons

Joint destruction in rheumatoid arthritis occurs by an obscure process. The cellular changes, however, have been documented. Place the process in the correct order.

Neutrophils, macrophages, and lymphocytes arrive Immune complexes phagocytized, releasing lysosomal enzymes Destructive changes in joint cartilage Inflammatory response Reactive hyperplasia of synovial cells and subsynovial tissues Vasodilation and joint swelling

The nurse is caring for a patient with newly diagnosed systemic lupus erythematosus (SLE). Which of the following over-the-counter medications does the nurse recognize is useful in treating inflammation, arthritis, and pleuritis?

Nonsteroidal anti-inflammatory drugs (NSAIDs)

A client diagnosed with Parkinson disease is displaying the following manifestations: tremor, rigidity, and slowness of movement. The nurse would interpret these as:

Normal manifestations of Parkinson disease

The nurse is preparing a client for oculovestibular reflex assessment (cold caloric test). The nurse explains that the test is used to elicit which of the following?

Nystagmus

When a nurse is assessing a patient with osteoarthritis, which of the following factors poses a risk factor for the disease?

Obesity

The nurse is conducting a health promotion class on osteoarthitis (OA). Which of the following statements should the nurse include?

Obesity is a strong risk factor for developing OA.

bone marrow

Occupies the medullary cavities of long bones and cavities of cancellous bones in the vertebrae, ribs, sternum, and flat bones of the pelvis.

woven bone

Often referred to as bundle bone Deposited more rapidly than lamellar bone Has low tensile strength ~ serves as temporary scaffolding for support Found in the developing fetus; in areas surrounding tumors and infections; and as part of a healing fracture

Following a fall 4 weeks prior that was caused by orthostatic hypotension, an 83 year-old male has fractured his femoral head. His care provider has stated that the healing process is occurring at a reasonable pace, and that the man will regain full function after healing and rehabilitation. Which of the following cells are most responsible for restoring the integrity of the man's broken bone?

Osteoblasts

What type of cell is responsible for building bone in the body?

Osteoblasts

A client has developed increased resorption of bone with removal of mineral content. This is most likely the result of:

Osteoclasts

prolactin

PTH and prolactin stimulate vitamin D3 precursors by the kidney

calcium

PTH maintains serum calcium levels by initiation of calcium release from the bone; by conservation of calcium by the kidney; by enhanced intestinal absorption of calcium through activation of vitamin D; and by reduction of serum phosphate levels (see Fig. 56.4)

The client experiences a twisting knee injury. Which type of pain would the provider expect the client to have?

Pain that is experienced as hip pain

Signs and Symptoms of Carpal Tunnel:

Pain, paresthesia, and numbness of the thumb and first two and one half digits of the hand; pain in the wrist and hand; decreased grip.

A client who has experienced a spinal cord injury still has use of the arms and has impaired motor and sensory function of the trunk, legs, and pelvic organs. The injury would be classified as:

Paraplegia

After being thrown off the back of a bull, the bull rider can move their arms but has loss of motor function in the lumbar and sacral segments of the spinal cord. This is usually referred to as being a:

Paraplegia

parathyroid hormone (PTH)

Parathyroid hormone (PTH) increases the number and resorptive function of the osteoclasts

The student is comparing the actions of parathyroid hormone (PTH) and calcitonin. Which of the following statements is most correct?

Parathyroid hormone produces a prompt increase in serum calcium levels.

What disease results from the degeneration of the dopamine nigrostriatal system of the basal ganglia?

Parkinson Disease

What disease results from the degeneration of the dopamine nigrostriatal system of the basal ganglia?

Parkinson disease

Results from loss of neurons in the substantia nigra

Parkinson's disease

The nurse is providing client education to a client newly diagnosed with Parkinson's disease. The nurse most accurately describes the disease progression:

Parkinson's is a chronic condition and treatment aims to manage symptoms.

An adolescent asks the nurse about peak bone mass. The most appropriate response would be:

Peak mass occurs in early adulthood.

Which of the following covers bones and has an inner layer that contains osteoprogenitor cells?

Periosteum

periosteum

Periosteum: covers the bones, except at their articular ends (see Fig. 56.1) Has an outer fibrous layer and an inner layer that contains the osteoprogenitor cells needed for bone growth and development Contains blood vessels; and acts as an anchorage point for vessels as the vessels enter and leave the bone

Any primary disorder of the peripheral nerves:

Peripheral Neuropathy

Coup and contrecoup cerebral confusion caused by blunt force trauma against a fixed object results in:

Permanent brain tissue damage

Magnetic resonance imaging of a client's knee has revealed the presence of bursitis. The nurse should anticipate performing which intervention for bursitis?

Placing an ice pack on the knee to decrease swelling

Synovial membranes can form sacs, called bursae. What is the function of bursae?

Prevent friction on a tendon

Which of the following physiological processes is performed by the kidneys and contributes to increased blood pressure?

Production and release of renin

One of the first signs that indicates an infant may have congenital hypothyroidism is:

Prolonged period of physiologic jaundice

Select the statement that best describes systemic lupus erythematosus (SLE). SLE is characterized by which of the following?

The formation of autoantibodies and immune complexes (type III hypersensitivity)

A 30 year-old woman has just given birth to boy. How will the mother's bone marrow differ from that of her son?

Proportionately, the infant will have more red marrow and less yellow marrow than his mother.

Which of the following assessment findings is most suggestive of nephrotic syndrome?

Proteinuria and generalized edema

A client has sustained an acute spinal cord injury in a fall from a tree stand during a hunting trip. The client will require surgical intervention for the unstable spinal cord. What does the nurse recognize is the goal of early surgical intervention for this client?

Provide internal skeletal stabilization.

A 26-year-old woman has sought care for increasing pain at the back of her ankle and the bottom of her foot over the past 2 weeks. The client states that she is generally in good health, although she completed a course of antibiotics for a chlamydial infection 6 weeks earlier. This client's recent history suggests the possibility of:

Reactive arthritis

A client was diagnosed with Salmonella 1 week ago and is now experiencing joints that are warm, swollen, and tender. The client also reports experiencing fever and weight loss. The health care provider would interpret these manifestations as:

Reactive arthritis

A patient admitted to the emergency department with a change in mental status and a history of AIDS and primary central nervous system (CNS) lymphoma becomes extremely combative with the medical personnel. A family member is very upset with the patient's behavior. The nurse explains that these behaviors are most likely caused by which of the following?

Recurrence of primary CNS lymphoma

A patient admitted to the emergency department with a change in mental status and a history of AIDS and primary central nervous system (CNS) lymphoma becomes extremely combative with the medical personnel. A family member is very upset with the patient's behavior. The nurse explains that these behaviors are most likely caused by which of the following? a) Recurrence of primary CNS lymphoma b) Onset of early dementia c) Decrease in intracranial pressure d) AIDS-induced encephalopathy

Recurrence of primary CNS lymphoma Primary CNS lymphoma has a higher incidence in people who are immunocompromised and who also have a high rate of recurrence in spite of treatment. The most common symptoms include behavioral and cognitive changes that occur in about 43% of the cases. There is no information in the question to support any of the other options.

The physician has scheduled a bone marrow biopsy for a client diagnosed with severe anemia. The purpose of the test will be to analyze the ability of bone marrow to produce:

Red blood cells

red bone marrow

Red bone marrow: contains developing RBCs and is the site of blood cell formation

While reviewing the role of the parathyroid hormone in the balance of calcium and phosphate levels, the nursing faculty will emphasize that the kidney responds to parathyroid stimulation by:

Reducing the reabsorption of phosphate.

While lifting weights during football season, a player hears a loud "pop." This is primarily associated with a loss of which function?

Reflexive proprioception

An athlete has become dehydrated during a long race in hot weather. Which of the following physiologic processes will occur in an attempt to protect the athlete's extracellular fluid volume?

Release of ADH from the posterior pituitary

The transmission of impulses at the neuromuscular junction is mediated by which of the following actions?

Release of neurotransmitter acetylcholine from autonomic nerve endings

A client has started having uncontrolled seizures that are not responding to usual medications. Nursing working with the client must pay special attention to which of the following priority aspects of this clients care? Assessment of:

Respiratory status and oxygen saturation Tonic-clonic status epilepticus is a medical emergency and, if not promptly treated, may lead to respiratory failure and death. Treatment consists of appropriate life support measures. Airway/breathing is always the priority in this emergency situation.

A client has started having uncontrolled seizures that are not responding to usual medications. Nursing working with the client must pay special attention to which of the following priority aspects of this clients care? Assessment of: a) Ability to grasp hands and squeeze on command b) ECG for arrhythmias c) Respiratory status and oxygen saturation d) Urine output and continence

Respiratory status and oxygen saturation Tonic-clonic status epilepticus is a medical emergency and, if not promptly treated, may lead to respiratory failure and death. Treatment consists of appropriate life support measures. Airway/breathing is always the priority in this emergency situation

A patient with a diagnosis of myasthenia gravis has required a mastectomy for the treatment of breast cancer. The surgery has been deemed a success, but the patient has gone into a myasthenic crisis on postoperative day 1. Which of the following measures should the care team prioritize in this patient's immediate care?

Respiratory support and protection of the patient's airway

A patient with a diagnosis of myasthenia gravis has required a mastectomy for the treatment of breast cancer. The surgery has been deemed a success, but the patient has gone into a myasthenic crisis on postoperative day one. Which of the following measures should the care team prioritize in this patient's immediate care? Positioning the patient to minimize hypertonia and muscle rigidity Seizure precautions Respiratory support and protection of the patient's airway Monitoring the patient for painful dyskinesias

Respiratory support and protection of the patient's airway

the patient has gone into a myasthenic crisis. What should the care team prioritize in this patient's immediate care?

Respiratory support and protection of the patient's airway

A patient is devastated to receive a diagnosis of amyotrophic lateral sclerosis (ALS). The symptomatology of this disease is a result of its effects on upper and lower motor neurons. The health care provider caring for this patient will focus on which priority intervention for this patient?

Respiratory ventilation assessment and prevention of aspiration pneumonia

A parent brought her 8-year-old child to the emergent care center to be examined following a fall off a playground set, resulting in a head injury. On discharge, the nurse explains to the parent that some symptoms may continue, including the inability to remember what happened before the fall. Which of the following terms best describes the diagnosis?

Retrograde amnesia The loss of memory following a concussion can be part of the postconcussion syndrome. The term for loss of memory before the accident is retrograde amnesia.

The nurse caring for a child with a deficiency of vitamin D knows that the deficiency places the child at risk for which of the following?

Rickets

When comparing a child's clinical manifestations with that of oligoarthritis versus systemic onset, the health care provider diagnoses this in your 4-year-old child with oligoarthritis based on which of the following clinical findings?

Right knee is warm and painful when putting it through normal range of motion.

Which of the following clients would be considered high risk for falling and fracturing a hip?

Risk factors for hip fracture include excessive consumption of alcohol and caffeine (rather than sugar), physical inactivity, low body weight, tall stature, use of certain psychotropic drugs, residence in an institution, visual impairment (rather than hearing), and dementia. Osteoporosis weakens the bone and is an important contributing factor. Most hip fractures result from falls.

To maintain adequate serum calcium levels, what does parathyroid hormone reduce?

Serum phosphate levels

A client is experiencing an episode of gout. The nurse recognizes that the condition is a result of:

Serum uric acid levels

functions of skeletal system

Serves as a framework for the attachment of muscles, tendons, and ligaments Protects and maintains soft tissues in their proper position Provides stability for the body Maintains the body's shape Acts as a storage reservoir for calcium Contains the hematopoietic connective tissue in which blood cells are formed

The radiology student is studying x-rays of the wrist. The instructor explains that these irregularly shaped bones are known as which of the following?

Short bones

A client presents to the emergency department following an accident where he fell off a chair. He reports hip numbness, increasing pain, and muscle cramping. The triage nurse suspects:

Shortly after a fracture has occurred, nerve function at the fracture site may be temporarily lost. The area may become numb, and the surrounding muscles may become flaccid. After this brief period, the pain sensation returns and with it muscle spasms and contractions of the surrounding muscles. There are pain, tenderness at the site of bone disruption, swelling, loss of function, deformity of the affected part, and abnormal mobility.

In contrast to synarthrosis joints, which synovial joints allow for movement in all directions?

Shoulder joints

The nurse taking a report on a client coming into the emergency room plans care for a client with brain dysfunction based on which of the following symptoms?

Stupor The most frequent sign of brain dysfunction is an altered level of consciousness such as stupor.

Which of the following complications of spinal cord injury is the most preventable in a paraplegic patient?

Skin breakdown

Select the statement that While reviewing the bones in anatomy class, the instructor discusses this type of bone that protects the underlying structures. A good example of a flat bone is:describes a synovial or diarthrodial joint. The joint:

Skull.

While reviewing the bones in anatomy class, the instructor discusses this type of bone that protects the underlying structures. A good example of a flat bone is:

Skull.

The elderly population needs special consideration in the treatment of the arthritic diseases. The NSAIDs, a first-line group of drugs used in the general population for arthritic diseases, may not be well tolerated by the elderly. What side effects of NSAIDs might be seen in the elderly?

Sleeplessness

A family brings their father to his primary care physician for a checkup. Since their last visit, they note their dad has developed a tremor in his hands and feet. He also rolls his fingers like he has a marble in his hand. The primary physician suspects the onset of Parkinson disease when he notes which of the following abnormalities in the patient's gait?

Slow to start walking and has difficulty when asked to "stop" suddenly

After numerous trips to the physician's office, a client is diagnosed with diffuse scleroderma based on which of the following clinical manifestations? Select all that apply.

Some persons with scleroderma have limited involvement and may develop the CREST syndrome, characterized by a combination of calcinosis (i.e., calcium deposits in the subcutaneous tissue that erupt through the skin), Raynaud phenomenon (a vascular disorder characterized by reversible vasospasm of the arteries supplying the fingers), esophageal dysmotility, sclerodactyly (localized scleroderma of the fingers), and telangiectasia (dilated skin capillaries). Protruding eyeball is related to hyperthyroidism, and inability to walk distances without cramping is known as claudication caused by poor circulation.

The emergency department nurse is caring for a patient who fell and had a head injury. Which of the following assessments would be noted during the early stage of intracranial pressure increase?

Stable vital signs The vital signs remain unchanged in the early stage of increased intracranial pressure.

The emergency department nurse is caring for a patient who fell and had a head injury. Which of the following assessments would be noted during the early stage of intracranial pressure increase? a) Vomiting b) Hemiplegia c) Nonreactive pupils d) Stable vital signs

Stable vital signs The vital signs remain unchanged in the early stage of increased intracranial pressure. The remaining options are characteristic of late signs.

A 30-year-old man has been diagnosed with mumps orchitis. The nurse should educate the client that which of the following complications may occur?

Sterility

A major complication of persistent esophageal reflux is a. Strictures b. Heartburn c. Chest Pain d. Hoarseness

Strictures

laminar bone

Strong, mature form of bone that is formed slowly and is highly organized Composed of cylindrical units (osteons / Haversian systems

lamellar bone

Strong, mature form of bone that is formed slowly and is highly organized Mature bone found in the adult skeleton ~ anything other than lamellar bone in an adult skeleton is abnormal Composed largely of cylindrical units, osteons or Haversian systems Osteons: consist of concentric lamellae of bone matrix surrounding a central canal, the Haversian canal, which contains the blood vessels and nerve supply for the osteon (see Fig. 56.3) In compact bone, the central Haversian canal runs essentially parallel to the long axis of the bone

What term is used to describe a level of consciousness that sees a client responding only to vigorous and repeated stimuli and has minimal or no spontaneous movement?

Stupor Stupor is unresponsive except to vigorous and repeated stimuli; responds appropriately to painful stimuli; lies quietly with minimal spontaneous movement; may have incomprehensible sounds and/or eye opening.

The nurse taking a report on a client coming into the emergency room plans care for a client with brain dysfunction based on which of the following symptoms? a) Wheezing b) Chest pain c) Stupor d) Pupils that react to light

Stupor The most frequent sign of brain dysfunction is an altered level of consciousness such as stupor. Pupils that react to light, wheezing and chest pain are not symptoms of brain function.

The MRA scan of a client with a suspected stroke reports ruptured berry aneurysm. The nurse plans care for a client with which of the following?

Subarachnoid hemorrhage The rupture of a berry aneurysm leads to a subarachnoid hemorrhage.

The MRA scan of a client with a suspected stroke reports ruptured berry aneurysm. The nurse plans care for a client with which of the following? a) Encephalitis b) Subarachnoid hemorrhage c) Thrombotic stroke

Subarachnoid hemorrhage The rupture of a berry aneurysm leads to a subarachnoid hemorrhage.

A 71 year-old male with a suspected diagnosis of osteoarthritis is being scheduled for a knee aspiration. The client is surprised to learn that his knee joint contains fluid and asks the physician ordering the procedure what the main role of the fluid is. Which of the following statements best underlies the explanation that the physician will provide?

Synovial fluid allows for joint movement by minimizing friction.

Most joints in the body are what types of joints?

Synovial joints

The joint capsule in a diarthrodial joint consists of two layers. Which of the following is one of these layers?

Synovium

A nurse is caring for a patient admitted with a malar rash on the nose and cheeks. The nurse recognizes that this rash is characteristic of which of the following disease processes?

Systemic lupus erythematosus

The pathogenicity of the tubercle bacillus is related to a. formation of a Ghon's focus lesion b. its inherent destructive capabilities c. rapid viral replication in host cells d. T-cell delayed hypersensitivity response

T-cell delayed hypersensitivity response

The physician is considering prescribing an anti-tumor necrosis factor (TNF) like infliximab for a rheumatoid arthritis patient. Which of the following statements is accurate about the advantages of using a TNF inhibitor?

TNF inhibitors help slow the disease progression and improve your ability to perform routine ADL functions.

tendon

Tendons: attach skeletal muscle to bone Relatively inextensible because of their richness in collagen fibers

Cancellous bone is relatively light. What does its lattice-like structure give it?

Tensile strength

A patient is receiving acetaminophen (Tylenol) for fever. The patient also has inflammation in the knees and elbows with pain. Why will acetaminophen (Tylenol) assist in reducing fever but not in decreasing the inflammatory process?

The action of acetaminophen on prostaglandin inhibition is limited to the central nervous system. Aspirin and other nonselective NSAIDs inhibit COX-1 and COX-2. Acetaminophen does not produce an antiplatelet effect. Prostaglandins do not affect gastric secretions.

A nurse practitioner has discussed the possible risks and benefits of isotretinoin treatment with a patient who has acne vulgaris. The nurse should be aware that this medication achieves a reduction in the signs and symptoms of acne by what method?

The antiacne effects of isotretinoin include suppression of sebum production, inhibition of comedone formation, and inhibition of inflammation. The drug does not stimulate immune function or reduce the function of sweat glands.

appendicular skeleton

The appendicular skeleton: composed of the bones of the upper and lower extremities, including the shoulder and the hip

axial skeleton

The axial skeleton: composed of the bones of the skull, thorax, and vertebral column ~ forms the axis of the body

The nurse is explaining how vasogenic brain edema occurs to a client's family. The most appropriate information for the nurse to provide would be:

The blood-brain barrier is disrupted, allowing fluid to escape into the extracellular fluid. Vasogenic brain edema occurs with conditions that impair the function of the blood-brain barrier and allow the transfer of water and protein from the vascular space into the interstitial space.

A college baseball player has seen his season cut short by a rotator cuff injury. Rotator cuff injuries are frequent because of:

The complexity and flexibility of the shoulder also mean that it is one of the more unstable joints. This makes it extremely vulnerable to injuries, including sprains, dislocations, and degenerative processes such as rotator cuff disorders. The glenohumeral joint contains ligaments, although menisci do not exist in the shoulder. The anatomy of the humerus does not account for the weakness of the joint.

The nurse is explaining how vasogenic brain edema occurs to a client's family. The most appropriate information for the nurse to provide would be: a) Normal physiologic circumstances result in decreased adsorption of CSF. b) There is a decrease in the amount of fluid volume in the brain. c) The blood-brain barrier is disrupted, allowing fluid to escape into the extracellular fluid. d) There is an increase in the production of cerebral spinal fluid volume.

The blood-brain barrier is disrupted, allowing fluid to escape into the extracellular fluid. Vasogenic brain edema occurs with conditions that impair the function of the blood-brain barrier and allow the transfer of water and protein from the vascular space into the interstitial space. Increased production of CSF and decreased absorption result in hydrocephalus. It occurs in conditions such as hemorrhage, brain injury, and infectious processes. (less

The nurse is explaining how vasogenic brain edema occurs to a client's family. The most appropriate information for the nurse to provide would be: a) There is a decrease in the amount of fluid volume in the brain. b) There is an increase in the production of cerebral spinal fluid volume. c) Normal physiologic circumstances result in decreased adsorption of CSF. d) The blood-brain barrier is disrupted, allowing fluid to escape into the extracellular fluid.

The blood-brain barrier is disrupted, allowing fluid to escape into the extracellular fluid. Vasogenic brain edema occurs with conditions that impair the function of the blood-brain barrier and allow the transfer of water and protein from the vascular space into the interstitial space. Increased production of CSF and decreased absorption result in hydrocephalus. It occurs in conditions such as hemorrhage, brain injury, and infectious processes.

A clinician is conducting an assessment of a male client suspected of having a disorder of motor function. Which of the following assessment findings would suggest a possible upper motor neuron (UMN) lesion?

The client displays increased muscle tone.

Which of the following clients would experience an accelerated rate of cell division in the stratum germinativum layer?

The deepest layer, the stratum germinativum, consists of a single layer of basal cells that are attached to the basal lamina in the basement membrane zone. The basal cells are the only epidermal cells that are mitotically active. It normally takes 3 to 4 weeks for the epidermis to replicate itself. The rate of cell division in the stratum germinativum is greatly accelerated when the outer layers of the epidermis are stripped away as occurs in abrasions and burns. The outer epidermis, which is avascular, is composed of four to five layers of stratified squamous epithelial cells, predominantly keratinocytes, which are formed in the deepest layer of the epidermis and migrate to the skin surface to replace cells that are lost during normal skin shedding. These are most affected by incontinence. Stretch marks are similar to stratum granulosum cells that have a dark staining granule of keratohyalin.

A client has been recovering from a stroke for several weeks and has been reluctant to participate in physical therapy. As a result, the client has experienced disuse atrophy. The nurse should recognize that the client is experiencing the consequences of which physiologic process?

The diameter of the client's muscle cells has decreased

A nurse is providing education to a client newly diagnosed with Guillain-Barre' syndrome. Which statement reflects accurate information about the course of the disease?

The disorder may present with rapid life-threatening symptoms or may present as a slow insidious process.

two 16-gauge IV catheters

The emergency department (ED) nurse receives report that a seriously injured patient involved in a motor vehicle crash is being transported to the facility with an estimated arrival in 5 minutes. In preparation for the patient's arrival, the nurse will obtain

A long bone, such as the humerus of the upper arm, has which of the following structural characteristics?

The endosteum is the membrane that lines the spaces of spongy bone, the marrow cavities, and the haversian canals of compact bone. It is composed mainly of osteoprogenitor cells that contribute to the growth and remodeling of bone and that are necessary for bone repair. The principle functions of the periosteum and endosteum are the nutrition of bone tissue and continuous supply of new osteoblasts for repair and growth of bone. The perichondrium lines cartilage, not bone, and the outer shell of long bones is compact, or cortical, bone. The diaphysis is the medial section of long bone; an epiphysis is found at each end of the bone.

A college baseball player has seen his season cut short by a rotator cuff injury. Rotator cuff injuries are frequent because of:

The inherent instability of the shoulder

The nurse is providing patient education related to intra-articular corticosteroid injections. Which of the following should the nurse include?

The injections will be given only 3-4 times per year because they can increase joint destruction.

Which of the following is the best example of a diarthrodial joint?

The interphalangeal joint between the proximal and middle phalanges of the fingers.

A nurse is discussing how vascular dementia differs from Alzheimer disease with a client. Select the most appropriate information. a) The main presenting symptom of vascular dementia is slow psychomotor function. b) Alzheimer disease most often presents with a rapid onset of memory loss followed by slowly progressive dementia that has a course of several years. c) Vascular dementia develops between the ages of 35 and 45. d) Vascular dementia is caused by neuritic (senile) plaques.

The main presenting symptom of vascular dementia is slow psychomotor function. The main presenting symptom of vascular dementia is slow psychomotor function. Alzheimer disease has been linked to neuritic plaque and has a slow onset. Vascular dementia usually manifests at age 50 or beyond.

An adult male patient is being administered immunosuppressant agents on a long-term basis. Which of the following assessments should be made routinely with the use of long-term immunosuppressant therapy?

The most common malignancies among transplant recipients are skin cancers and lymphomas. The patient should be assessed annually for skin cancer. The patient will not require a yearly bronchoscopy. The patient will not require a yearly bladder biopsy. The male patient should have a yearly prostate examination, but this examination is not required for immunosuppressant therapy.

A client with a history of osteoarthritis in his hip, which he refers to as "bad hip," is also complaining of knee pain. The physiological principle behind this would include the fact that:

The nerve supply to joints is provided by the same nerve trunks that supply the muscles that move the joints. These nerve trunks also supply the skin over the joints. As a rule, all the joints of an extremity are innervated by the same peripheral nerves as they travel down an extremity. The blood supply to a joint arises from blood vessels that enter the subchondral bone at or near the attachment of the joint capsule and form an arterial circle around the joint. The synovial membrane has a rich blood supply, and constituents of plasma diffuse rapidly between these vessels and the joint cavity. Because many of the capillaries are near the surface of the synovium, blood may escape into the synovial fluid after relatively minor injuries. Healing and repair of the synovial membrane usually are rapid and complete. This is important because synovial tissue is injured in many surgical procedures that involve the joint.

Alopecia related to chemotherapy.

The nurse is caring for a Sikh client who is undergoing radiation and chemotherapy for treatment of cancer. Which of the following would be most problematic for this client due to her religious beliefs?

referred

pain that originates in a visceral organ but is perceived elsewhere in the body wall that is innervated by neurons entering the same segment of the nervous system

Monitor the client's response to prescribed medications.

The nurse is caring for a client newly admitted with heart failure secondary to dilated cardiomyopathy. Which intervention would be a priority?

Fluctuation of the fluid level within the water seal chamber

The nurse is caring for a postoperative client who has a chest tube connected to suction and a water seal drainage system. Which of the following indicates to the nurse that the chest tube is functioning properly?

Obtain Do Not Resuscitate (DNR) order from the physician based on client beliefs

The nurse is caring for an Hmong client. Which of the following should be a priority intervention for the nurse to employ?

"Terminal cancer is a result of a curse."

The nurse is caring for clients of the Hmong culture and is assessing their understanding of the development of terminal cancer. Which of the following statements would the nurse expect based on cultural beliefs?

____distributing enzymes into the duodenum.

pancreas

"Have you had dental work done recently?"

The nurse obtains a health history from an older patient with a prosthetic mitral valve who has symptoms of infective endocarditis (IE). Which question by the nurse is most focused on identifying a risk factor for IE?

ventricular tachycardia

The nurse obtains a monitor strip on a patient who has had a myocardial infarction and makes the following analysis: P wave not apparent, ventricular rate 162, R-R interval regular, P-R interval not measurable, and QRS complex wide and disorted, QRS duration 0.18 second. The nurse interprets the patient's cardiac rhythm as

QRS complex followed by fibrillation and another QRS complex -Obtain the crash cart and go to the client's room.

The nurse on the cardiac unit notes the following rhythm of a client. The best initial response for the nurse would be which of the following?

Albumin level and weight loss

The nurse reviews the electronic health record for a patient scheduled for a total hip replacement. Which assessment data shown in the accompanying figure increase the patient's risk for complications after surgery?

pancreas distributing enzymes into the duodenum. Gallstones block the gall duct...the pancreas enzymes build up and eats itself is called

pancreatitis

A client with a diagnosis of small cell lung carcinoma has developed bone metastases, a finding that has prompted a series of new interventions. What are the primary goals of the treatment regimen for this client's bone cancer?

The primary goals in treatment of metastatic bone disease are to prevent pathologic fractures and promote survival with maximum functioning, allowing the person to maintain as much mobility and pain control as possible. Osteonecrosis, osteomyelitis, and brain metastasis are not typical sequelae of bone metastasis. Tumor sites do not normally undergo remodeling, and dietary supplements alone would not foster this process.

increase functional residual capacity and improve oxygenation

The purpose of adding positive end expiratory pressure (PEEP) to positive pressure is to:

synovial fluid

The synovium secretes a slippery fluid with the consistency of uncooked egg white, synovial fluid Normal synovial fluid is clear or pale yellow; does not clot Composition of synovial fluid is altered in many inflammatory and pathologic joint disorders Acts as a lubricant; facilitates the movement of the articulate surfaces of the joint Synovial fluid supports joint movement by minimizing friction Synovial fluid is particularly important to optimal knee function

Though the client's primary care provider has downplayed the symptoms, a geriatrician suspects that an 82 year-old female has polymyalgia rheumatica. Which characteristic symptomatology would most likely have led the specialist to suspect this health problem?

The woman complains of aching and morning stiffness in her neck, shoulder and pelvis.

A client presents with a fracture of a short bone. What is the most likely location of the client's fracture?

The wrist or ankle

A client has a fracture of an epiphyses. How will the nurse explain this fracture?

This is a fracture of the end of the bone.

A patient suffering a thrombotic stroke is brought into the emergency department by ambulance and the health care team is preparing to administer a synthetic tissue plasminogen activator for which of the following purposes?

Thrombolysis Synthetic tissue plasminogen activators work with the body's natural tissue plasminogen activators to convert plasminogen to plasmin, which breaks down clots to allow for the reestablishment of blood flow. There are two causes of strokes: hemorrhagic and thrombotic, with thrombotic strokes occurring much more frequently. Thrombolytics play a large role in increased outcomes seen with thrombotic strokes.

A patient suffering a thrombotic stroke is brought into the emergency department by ambulance and the health care team is preparing to administer a synthetic tissue plasminogen activator for which of the following purposes? a) Hemolysis b) Hemostasis c) Thrombolysis d) Thrombogenesis

Thrombolysis Synthetic tissue plasminogen activators work with the body's natural tissue plasminogen activators to convert plasminogen to plasmin, which breaks down clots to allow for the reestablishment of blood flow. There are two causes of strokes: hemorrhagic and thrombotic, with thrombotic strokes occurring much more frequently. Thrombolytics play a large role in increased outcomes seen with thrombotic strokes.

diaphysis

Typical long bone has a shaft (diaphysis) and two ends, called epiphyses (see Fig 56.2)

A client who is being seen in the outpatient clinic reports a single episode of unilateral arm and leg weakness and blurred vision that lasted approximately 45 minutes. The client is most likely experiencing:

Transient ischemic attack (TIA) Transient ischemic attacks are brief episodes of neurologic function resulting in focal cerebral ischemia not associated with infarction that usually resolve in 24 hours. The causes of transient ischemic attack are the same as they are for stroke. Embolic stroke usually has a sudden onset with immediate maximum deficit.

A client who is being seen in the outpatient clinic reports a single episode of unilateral arm and leg weakness and blurred vision that lasted approximately 45 minutes. The client is most likely experiencing: a) Transient ischemic attack (TIA) b) Lacunar infarct c) Cardiogenic embolic stroke d) Thrombotic stroke

Transient ischemic attack (TIA) Transient ischemic attacks are brief episodes of neurologic function resulting in focal cerebral ischemia not associated with infarction that usually resolve in 24 hours. The causes of transient ischemic attack are the same as they are for stroke. Embolic stroke usually has a sudden onset with immediate maximum deficit. Lacunar infarcts produce classic recognizable "lacunar syndromes" such as pure motor hemiplegia, pure sensory hemiplegia, and dysarthria with clumsy hand syndrome.

Cancer of the breast is the most common cancer in females.

True

Cerebral edema, or brain swelling, is characterized by an increase in volume secondary to an abnormal fluid accumulation.

True

Certain types of sexually transmitted human papillomaviruses are risk factors for a neoplasia, which can be a precursor lesion of invasive carcinoma.

True

Joints are the weakest part of the skeletal system.

True

Malignant melanoma, which is a malignant tumor of melanocytes, is a rapidly progressive and metastatic form of skin cancer.

True

Many drugs are labeled as ototoxic, or damaging to inner ear structures.

True

Many skeletal disorders of early infancy are caused by intrauterine positioning and resolve as the child grows.

True

Osteoclasts are "bone chewing" cells that function in the resorption of bone, removing the mineral content and the organic matrix.

True

Osteomalacia is a generalized bone condition in which inadequate mineralization of bone results from a calcium or phosphate deficiency, or both.

True

Ringing of the ears is known as tinnitus.

True

Sebaceous glands, which release a lubricant for the skin and hair, are located over the entire skin surface except for the palms, soles, and sides of the feet.

True

The condition of having undescended testes is also known as cryptorchidism.

True

The dermis and epidermis play a role in immune function.

True

The epidermis is well supplied by a vascular system.

True

Global or diffuse brain injury is manifested by changes in the level of consciousness.

True Global brain injury nearly always results in altered levels of consciousness, ranging from inattention to stupor or coma.

Global or diffuse brain injury is manifested by changes in the level of consciousness. a) False b) True

True Global brain injury nearly always results in altered levels of consciousness, ranging from inattention to stupor or coma.

bone

Two types of mature bones: compact and cancellous bones (see Fig. 56.1)

A 22-year-old male college student has presented to his campus medical clinic distraught at the emergence of numerous small blisters on the shaft of his penis. On examination, the clinician notes that the lesions are between 2 and 4 mm in diameter and are filled with serous fluid. The clinician would document the presence of:

Vesicles

Disorders of the pyramidal tracts, such as a stroke, are characterized by:

paralysis

The nurse is providing education to a patient recently diagnosed with multiple sclerosis. The nurse instructs the patient regarding which symptoms commonly occurring with multiple sclerosis? Select all that apply.

Visual loss or blurring Difficulty swallowing Paresthesias of extremities

Which vitamin has little or no activity until it has been converted to physiologically active compounds?

Vitamin D

vitamin D

Vitamin D (and its metabolites): not a vitamin but a steroid hormone Two forms of vitamin D with identical biological activity, but differing by the presence of a double-bond: Vitamin D2 (ergocalciferol) Vitamin D3 (cholecalciferol) Term vitamin D is used to indicate both forms Necessary for calcium absorption Highly valuable hormone in immune system modulation; bone mineralization; and suppression of malignant cells Has little or no activity until it has been converted to be physiologically active, and metabolized to compounds that mediate its activity

vitamin D2 (ergocalciferol)

Vitamin D (and its metabolites): not a vitamin but a steroid hormone Two forms of vitamin D with identical biological activity, but differing by the presence of a double-bond: Vitamin D2 (ergocalciferol) Vitamin D3 (cholecalciferol) Term vitamin D is used to indicate both forms Necessary for calcium absorption Highly valuable hormone in immune system modulation; bone mineralization; and suppression of malignant cells Has little or no activity until it has been converted to be physiologically active, and metabolized to compounds that mediate its activity

vitamin D3 (cholecalciferol)

Vitamin D (and its metabolites): not a vitamin but a steroid hormone Two forms of vitamin D with identical biological activity, but differing by the presence of a double-bond: Vitamin D2 (ergocalciferol) Vitamin D3 (cholecalciferol) Term vitamin D is used to indicate both forms Necessary for calcium absorption Highly valuable hormone in immune system modulation; bone mineralization; and suppression of malignant cells Has little or no activity until it has been converted to be physiologically active, and metabolized to compounds that mediate its activity

Osteomalacia is a bone disease caused by one of two reasons: inadequate calcium absorption or phosphate deficiency. In the elderly, what is the least expensive and most effective long-term treatment for osteomalacia?

Vitamin D-rich diet and exposure to midday sun

Volkmann canals

Volkmann canals: also contain blood vessels ~ are spaces in the cortex that run perpendicular to the long axis of the cortex to connect adjacent Haversian canals

A patient has been diagnosed with a cerebral aneurysm and placed under close observation before treatment commences. Which of the following pathophysiologic conditions has contributed to this patient's diagnosis?

Weakness in the muscular wall of an artery

A patient has been diagnosed with a cerebral aneurysm and placed under close observation before treatment commences. Which of the following pathophysiologic conditions has contributed to this patients diagnosis?

Weakness in the muscular wall of an artery

Uncontrolled hypertension

When discussing risk factor modification for a patient who has a 5-cm abdominal aortic aneurysm, the nurse will focus teaching on which patient risk factor?

ammonia is the byproduct of __ __ in the urine ammonia can't live in the body because it's toxic... the liver breaks it down into ___... neurological/____ caused by the build up of ammonia

amino acids urea encephalopathy

Administer a sedative before cardioversion is implemented

Which action should the nurse take when preparing for cardioversion of a patient with supraventricular tachycardia who is alert and has a blood pressure of 110/66 mm Hg?

The patient's serum creatinine level is elevated

Which data collected by the nurse caring for a patient who has cardiogenic shock indicate that the patient may be developing multiple organ dysfunction syndrome (MODS)?

Urine output is 65 mL over the past hour.

Which finding is the best indicator that the fluid resuscitation for a 90-kg patient with hypovolemic shock has been effective?

Serum potassium 2.9 mEq/L

Which laboratory result for a patient with multifocal premature ventricular contractions (PVCs) is most important for the nurse to communicate to the health care provider?

"PEEP prevents the lung air sacs from collapsing during exhalation."

Which statement by the nurse when explaining the purpose of positive end-expiratory pressure (PEEP) to the patient's caregiver is accurate?

monitoring the surgical incision for signs of infection.

While waiting for heart transplantation, a patient with severe cardiomyopathy has a ventricular assist device (VAD) implanted. When planning care for this patient, the nurse should anticipate

Unlike disorders of the motor cortex and corticospinal (pyramidal) tract, lesions of the basal ganglia disrupt movement

Without causing paralysis (think parkinson's)

Ankylosing spondylitis is a disease that typically manifests in late adolescence and early adulthood. What is the characteristic of the pain in ankylosing spondylitis?

Worse when lying

yellow bone marrow

Yellow bone marrow: composed largely of adipose cells

first

______-order neurons & their receptors detect painful stimuli

second

______-order neurons in spinal cord process information & transmit it to reticular formation & thalamus

third

______-order neurons project pain information to somatosensory cortex where perception & subjective meaning of pain take place

phantom limb pain

a neurologic pain that follows amputation of a limb

analgesia

absence of pain on noxious stimulation or relief of pain without loss of consciousness

cluster headache

accompanied by conjunctival injection (redness) or lacrimation (tearing), ipsilateral nasal congestion, eyelid edema, or forehead/facial sweating

Myasthenia gravis is characterized by muscle weakness caused by antibody-mediated destruction of ___ ___

acetylcholine receptors

Myasthenia gravis is characterized by muscle weakness caused by antibody-mediated destruction of:

acetylcholine receptors

___ ___ transmitters are blocked or less receptors

acetylcholine receptors (myasthenia gravis)

Myasthenia gravis is characterized by muscle weakness caused by antibody-mediated destruction of: periorbital muscles. thymus gland cells. skeletal muscle fibers. acetylcholine receptors.

acetylcholine receptors.

In addition to regulating body fluids and electrolytes, the kidneys function in maintaining bone calcium levels by:

activating vitamin D

glutamate

acts quickly but lasts only a few sec

A 16-year-old girl has been admitted to the emergency department after ingesting 20 g of acetaminophen (Tylenol) in a suicide attempt. The care team would recognize that this patient faces a severe risk of: __ __ __

acute fulminant hepatitis

severe, nagging pain that does not last long and comes and goes

acute gastritis

cluster headache

affects men more than weomen

cluster (headache)

affects men more than women; episodic occurrences

Atelectasis is most commonly caused by a. airway obstruction b. pulmonary embolism c. inflammation of the pleura d. chronic forceful coughing

airway obstruction

primary (somatosensory) cortex

akapostcentral gyrus

competitive with fatty acid metabolism (breakdown of triglyceride) build up of triglyceride and causes fatty liver...because is breaking down alcohol instead of fat.

alcoholic liver disease

epiphyseal growth plate

allows for bone growth until puberty

The most serious complication of _____ treatment is avascular necrosis of the femoral head as a result of forced abduction.

any

When the secretions of these glands combine with bacteria on the skin surface, they produce body odor.

apocrine glands

proprioceptive receptors

are found in muscles, tendons & joint capsules

dermatome maps

are used to assess level/extent of damage to segmental nerves &/or spinal cord

PAG (periaqueductal gray) region

area of the midbrain with ↑[opioid receptors] & with descending connections to the brainstem & spinal cord

A patient with a spinal cord injury at T8 would likely retain normal motor and somatosensory function of her

arms

A patient with a spinal cord injury at T8 would likely retain normal motor and somatosensory function of her:

arms

A patient with a spinal cord injury at T8 would likely retain normal motor and somatosensory function of her: arms. bowels. bladder. perineal musculature.

arms.

The most commonly occurring early symptoms of systemic lupus erythematosus (SLE) include:

arthralgias and arthritis

joint

articulation or junction between two or more bones or cartilage

joints

articulations

pain threshold

associated with tissue damage & point at which stimulus is received

tendons

attach skeletal muscle to bone

elastic cartilage

auricles auditory canal larynx

Inflammatory bowel diseases are accompanied by systemic manifestations that include a. autoimmune anemia b. rheumatoid arthritis c. thrombocytopenia d. lactose intolerance

autoimmune anemia

Inflammatory bowel diseases are accompanied by systemic manifestations that include ___ __

autoimmune anemia

The nurse is caring for a client experiencing a seizure. During the seizure the nurse notes that the client repetitively rubs his/her clothing. When contacting the client's physician, the nurse notes that the client exhibited:

automatisms. The nurse reports that the client exhibited automatisms, defined as repetitive nonpurposeful activities such as lip smacking, grimacing, patting and/or rubbing clothing.

high blood pressure and slow heart rate

autonomic dysreflexia

Skeletal tissue contains intercellular collagen fibers that provide ______ for tendons and ligaments. A) elastic recoil B) shape flexibility C) tensile strength D) inorganic calcium

c. tensile strength

The very deep gasping breathing pattern which is often observed in persons with ketoacidosis can be viewed as a compensatory mechanism in which the increased ventilation attempts to a. increase the oxygen content in blood in an effort to burn the excess glucose b. blow off excess carbon dioxide in an effort to increase pH c. increase the respiratory loss of ketone bodies as a means of controlling pH d. increase the oxygen supply to the brain

b. blow off excess carbon dioxide in an effort to increase pH.

Whereas PTH increases blood calcium levels, the hormone _____ lowers blood calcium levels and decreases bone resorption. A) vitamin D B) calcitonin C) prolactin D) phosphate

b. calcitonin

An increase in the level of RANKL would result in: A) fusing of the epiphysis and metaphysic in long bones. B) changes in the rate of bone remodeling. C) decreased production of PTH. D) increased vitamin D activation.

b. changes in the rate of bone remodeling

Thyroid hypofunction in the newborn produces a condition known as _____ a. acidotic coma b. cretinism c. hyperpyrexia d. myxedema e. thyrotoxicosis

b. cretinism

Cartilage is a firm but flexible type of connective tissue that is essential for: A) calcium salt storage. B) growth of long bones. C) bone surface perfusion. D) reduced friction on tendons.

b. growth of long bones

Which of the following characteristics differentiates cartilage from bone? A) Secretion of an extracellular matrix B) Avascularity C) Low tensile strength D) Rapid healing

b. secretion of extracellular matrix

To maintain adequate serum calcium levels, parathyroid hormone reduces: A) activation of vitamin D. B) serum phosphate levels. C) calcium release from bone. D) intestinal absorption of calcium.

b. serum phosphate levels

What role do osteoblasts play in the physiology of bone tissue? A) Differentiation into mature bone cells B) Synthesis and secretion of bone matrix C) Maintenance of calcium balance D) Resorption of the bone matrix

b. synthesis and secretion of bone matrix

extrapyramidal

balance and coordination (- rigidity, immobility, but not necessarily paralysis. movement- can't control)

thalamus

basic sensation of pain occurs at level of _______

(somatosensory) association area

behind primary somatosensory cortex

The patient has right upper quadrant pain caused by acute choledocholithiasis. If the common bile duct becomes obstructed, manifestations will also include

bilirubinuria

s1-s5-

bladder control

osteoclast

bone cell that absorbs and removes unwanted bony tissue

osteoprogenitor cells

bone stem cells

irregular bones

bones of the vertebrae and face

Cardinal symptoms of Parkinson's disease include:

bradykinesia

The cardinal symptoms of Parkinson disease include

bradykinesia

Upper motor neuron damage..where? what?

brain hyper reflexia (...spine reflex is intact, but brain can't shut it off )

In adults, the predominant effect of a prolonged elevated growth hormone level is a. short stature with obesity b. high androgen hormone levels c. increased blood glucose levels d. insulin-like growth factor (IGF) depletion

c. increased glucose levels

How are the oxygenation needs of the articulating areas in a synovial joint met? A) Capillaries release oxygenated blood into the synovial cavity at a controlled rate. B) The cartilage of the articulating areas uses anaerobic metabolism exclusively. C) Oxygen is provided to the articulating areas by synovial fluid rather than by blood. D) The epiphyses of long bones provide oxygen to the articulating areas.

c. oxygen is provided to the articulating areas by synovial fluid rather than by blood

The tendons and ligaments of the joint capsule are sensitive to position and movement as a result of having _______ to help maintain muscle support. A) autonomic nerve fibers B) supporting bursa sacs C) reflexive proprioception D) elastic articular cartilage

c. reflexive proprioception

The joint capsule consists of an outer fibrous layer and an inner synovium, which ______ to facilitate movement. A) senses position B) connects to tendons C) secretes synovial fluid D) covers articular cartilage

c. secretes synovial fluid

An elderly patient has been brought to his primary care provider by his wife who is concerned about his recent decrease in coordination. Upon assessment, his primary care provider notes that the patients gait is wide-based, unsteady, and lacking in fluidity, although his muscle tone appears normal. This patient requires further assessment for which of the following health problems?

cerebellar disorders

The intracranial volume that is most capable of compensating for increasing intracranial pressure is the:

cerebrospinal fluid

unmyelinated

chemical or chronic mechanical/thermal stimuli

constant damage and scar tissue formation (chronic)...end stage liver disease

cirrhosis

second-order neurons

communicate with spinal reflexes & ascending tract from spinal cord → thalamus

cortical bone

compact bone

slower

conduction of thermal information is (faster/slower) than the rapid tactile tracts

The moderate stage of the progressive degenerative Alzheimer-type dementia is manifested by behaviors that include:

confusion

cartillaginous joints (amphiarthoses)

connected by hyaline cartilage or fibrocartilage and have limited motion

hyperpathia

continued stimulation causes pain

A basketball player fell awkwardly when attempting to claim a rebound, a mishap that resulted in a tear to the anterior cruciate ligament of his left knee. What characteristic of ligaments makes them particularly susceptible to injury? A) Ligaments are incapable of accommodating lateral movement. B) Ligaments are exclusive to the knee, which is the most frequently injured joint. C) Ligaments are superficial to the synovial capsule and are consequently vulnerable to impacts. D) Ligaments are incapable of stretching when exposed to unusual stress.

d. ligaments are incapable of stretching when exposed to unusual stress

Bones are covered, except at their articular ends, by a ______ membrane. A) canaliculi B) endosteum C) synovial D) periosteum

d. periosteum

Cancellous bone is relatively light, but its lattice-like structure gives it: A) rigidity. B) thickness. C) a growth plate. D) tensile strength.

d. tensile strength

Magnetic resonance imaging of a patient's knee has revealed the presence of bursitis. What is the primary purpose of bursae? A) To maintain close articulation between the two long bones at a synovial joint B) To strengthen the attachment between skeletal muscles and bones C) To strengthen the integrity of the articulating capsule D) To prevent friction at a tendon in a synovial joint

d. to prevent friction at a tendon in a synovial joint

The demyelination and degeneration of nerve fibers characteristic of multiple sclerosis is the result of ___ ____

decreased oligodendrocytes

The demyelination and degeneration of nerve fibers characteristic of multiple sclerosis is the result of:

decreased oligodendrocytes

The demyelination and degeneration of nerve fibers characteristic of multiple sclerosis is the result of: decreased oligodendrocytes. corticospinal injuries. atherosclerotic destruction. oligodendrocytic infection.

decreased oligodendrocytes.

Hair follicle end organs

detect impending touch on skin surface

The patient has a traumatic complete spinal cord transection at the C5 level. Intact motor and somatosensory function will include ______ control.

diaphragm

The patient has a traumatic complete spinal cord transection at the C5 level. Intact motor and somatosensory function will include ______ control. bladder finger flexion diaphragm trunk muscle

diaphragm

c1-c4

diaphragm paralysis

With bronchiectasis, persistent airway obstruction and chronic infection results in bronchial a. dilation b. rupture c. infarction d. thickening

dilation

The mechanisms of liver damage in viral hepatitis include __ __ __

direct cellular injury

Thermal sensation

discriminated by cold, warmth, & pain receptors

dyesthesia

distortions of somesthetic sensation

dyesthesia

distortions of somesthetic sensation (usually associated with a partial loss of sensory innervation)

A patient is beginning to recover from acute tubular necrosis. The recovery phase of ATN is characterized by:

diuresis

The patient is beginning to recover from acute tubular necrosis. The recovery phase of ATN is characterized by

diuresis

A patient's recent diagnosis of Parkinson disease has prompted his care provider to promptly begin pharmacologic therapy. The drugs that are selected will likely influence the patient's levels of:

dopamine

A patients recent diagnosis of Parkinson disease has prompted his care provider to promptly begin pharmacologic therapy. The drugs that are selected will likely influence the patients levels of:

dopamine

A patient's recent diagnosis of Parkinson disease has prompted his care provider to promptly begin pharmacologic therapy. The drugs that are selected will likely influence the patient's levels of: dopamine. acetylcholine. serotonin. adenosine.

dopamine.

Which of the following integumentary problems most often accompanies chronic kidney disease?

dry skin and pruritus

paresthesias

example: -pins-and-needles sensation following compression of a peripheral nerve

glutamate

excitatory NT in spinal cord

fast-conducting (neospinothalamic tract)

experienced as bright, sharp or stabbing pain

pain tolerance

extremely variable & depends on person's culture & previous experiences

trigeminal neuralgia

facial tics or grimaces characterized by stabbing, paroxysmal attacks of pain

myelinated

fast pain

dorsal column (medial lemniscal pathway)

fast-conducting

Acute pancreatitis involves activated pancreatic enzymes that escape into surrounding tissues to cause _______ in the abdominal cavity. a. fatty deposits b. autodigestion c. bowel obstruction d. abscess formation

fatty deposits

hyaline cartilage is important for

fetal bone growth development

pockets in the intestines...add more __ and avoid __

fiber seeds (diverticulitis)

C pain

fibers release both glutamate & substance P

Syndemoses are separated by a:

fibrous disk

if an organ perforates in the peritoneum, a profound inflammatory response...lots of __ ___

fibrous exudate (peritonitis)

gomphosis

fibrous joint tooth and alveolar process of jaw bone

Ligaments

fibrous thickenings of the articular capsule that join one bone to its articular mate

three types of joints

fibrous, cartilaginous, synovial

The "cold viruses" are rapidly spread from person to person. The greatest source of spread is a. fingers b. sneezing c. plastic toys d. eye mucosa

fingers

A sudden traumatic complete transection of the spinal cord results in ____ ___ below the level of injury.

flaccid paralysis

A sudden traumatic complete transection of the spinal cord results in _______ below the level of injury.

flaccid paralysis

Meissner corpuscles

found in nonhairy skin regions (lips, fingertips) -highly-developed sense of touch

somatosensory

four major modalities of _______ experience: 1. discriminative touch 2.proprioception 3.temperature 4.nociception

synovial joints

freely movable joints articulating bone ends are separated by a joint cavity containing synovial fluid

Factors that contribute to the formation of gallstones or acute cholelithiasis include a. chronic pancreatitis b. raid elimination of bile c. gallbladder inflammation d. excessive alcohol intake

gallbladder inflammation

19. Extreme cerebral edema may cause the brain to herniate into another compartment. Upward herniation from the infratentorial compartment against the aqueduct of Sylvius causes:

hydrocephalus

A client begins to exhibit signs and symptoms of a stroke at a community health fair. Emergency care for the client includes: a) going to an urgent care center. b) seeing his/her physician. c) going to the nearest stroke center. d) going to the nearest emergency room.

going to the nearest stroke center. Salvaging brain tissue, preventing secondary stroke, and minimizing long-term disability are the treatment goals for an acute ischemic stroke. The care of patients with stroke has shifted away from the "nearest hospital" to certified stroke centers. These are hospitals that have been certified by some external agency, most commonly the state or Joint Commission on Accreditation of Healthcare Organizations, the federal agency overseeing all facilities that care for Medicare patients.

causes ulcers

h Pylori

primary (somatosensory) cortex

has a spatial orientation, the sensory homunculus

intracranial lesions

headaches that disturb sleep, are triggered by exertion, or are accompanied by drowsiness, visual or limb disturbances, or altered mental status suggest underlying ______ ______

Ruffini end organs

heavy continuous touch or pressure

High risk factors for development of adult renal cell cancer include:

heavy smoking

Which of the following patient complaints most clearly suggests a need for diagnostic testing to rule out renal cell carcinoma?

hematuria

blood moves into the liver from the ___ __ ___from the intestines and the spleen. The __ __ transports waste to the ___

hepatic portal vein (digested red blood cells...breaks into bilirubin) bile duct gallbladder

A late indicator of increased intracranial pressure is ___ __ ___ ___

high mean arterial pressure

A late indicator of increased intracranial pressure is:

high mean arterial pressure

nose is made of

hyaline cartilage

types of cartilage

hyaline, elastic, fibrocartilage

synchondroses

joints in which bones are connected by hyaline cartilage and have limited motion Ribs are attached to the sternum by synchondroses

The earliest sign of altered level of consciousness, or arouseability, include

inattention

The earliest signs of decreased level of consciousness include:

inattention

the skeletal system

includes joints, cartilages, and ligaments

parathyroid hormone

increase serum phosphate and calcium can crystallize soft tissues! Increases intestinal absorption of calcium Increases intestinal absorption of phosphate Decreases renal excretion of calcium Increases renal excretion of phosphate (increased serum phosphate and calcium can crystallize soft tissues) Increases bone resorption Decreases bone formation Promptly increases serum calcium levels Prevents increase in serum phosphate levels

Metabolic factors that increase cerebral blood flow include:

increased carbon dioxide level

hyperalgesia

increased painfulness

prostaglandins

induce pain

Which of the following pathophysiologic processes occurs in cases of bacterial meningitis?

inflammation allows pathogens to cross into the cerebrospinal fluid

NSAIDs

inhibit production of PGs; inhibits COX 1/2 enzymes

receptor

innervates a certain area of the periphery

(somatosensory) association area

integrates sensory information with past learning to produce a meaningful experience

phantom limb pain

involvement of forebrain (site of pain integration) has also been suggested

glutamate

is an excitatory NT released by the Aδ fibers within the spinal cord

somatosensory cortex

is needed to add precision/discrimination to the pain

substance P

is released more slowly & increases in concentration over seconds/minutes

A patient had excessive blood loss and prolonged hypotension during surgery. His postoperative urine output is sharply decreased and his blood urea nitrogen (BUN) is elevated. The most likely cause for the change is acute:

ischemic tubular necrosis

Abnormally high accumulation of bilirubin in the blood causes ___

jaundice

how is vitamin D activated

kidneys by hydroxylation

tension-type

most common type of headache

proprioception

movement of limbs & joints

gate control theory

neural gating mechanism exists at the segmental spinal cord level to account for interactions between pain and the other sensations

substance P

neurotransmitter from C fibers -slow release

tension-type

not sufficiently severe to interfere with daily activities

Checking the pupillary response to light

nurse is assessing a client's cranial nerves as part of a neurological examination. Which of the following actions should the nurse take to assess cranial nerve III?

neuralgia

occurs along the distribution of a cranial or spinal nerve

The fulminating degree of FES is:

often fatal

endorphins

opiod peptides that serve as endogenous

4 types of bone cells

osteoprogenitor cells, osteoblasts, osteocytes, osteoclasts

complex regional pain syndrome

pain & mobility problems that are more severe than the injury warrants

allodynia

pain after non-noxious stimulus

nociception

pain or itch

pattern theory

pain receptors share endings or pathways with the other senses but that different patterns of activity by the same neurons signal painful or nonpainful stimuli

acute

pain that is form from tissue damage; characterized by ANS responses (e.g., nausea)

The initiating event in the development of nephrosis is a derangement in the glomerular membrane that causes increased permeability to

plasma proteins

Major causes of mechanical bowel obstruction include a. chemical irritation b. ruptured appendix c. abdominal distention d. postoperative adhesions

postoperative adhesions

A patient has wrist inflammation causing compression of the median nerve in the carpal tunnel. Manifestations of this syndrome include:

precision grip weakness

More complex patterns of movements, such as throwing a ball or picking up a fork, are controlled by the ______ cortex in the frontal lobe. premotor primary motor reflexive supplementary

premotor

Ischemic lesions of the skin and underlying structures caused by unrelieved pressure are:

pressure ulcers

endochondral ossification

process in which bone forms by replacing hyaline cartilage

As the nurse is performing a physical assessment of a client, the client begins to have seizure activity including loss of consciousness and limb jerking. The nurse's first priority is to: a) preserve brain functioning. b) treat underlying disease. c) protect the patient from injury. d) stop the seizure.

protect the patient from injury. The first priority for the nurse when a client begins to experience seizure activity is to protect the client from injury during the seizure. -Stopping or preventing the seizure, preserving brain functioning and/or treating underlying disease are important goals of treatment but secondary to protecting the client from injury.

muscle spasm (guarding)

protective reflex rigidity to guard affected body part

Glomerular damage causes ____

proteinuria (or, technically dehydration would be correct also)

proprioceptive receptors

provide information on stationary & dynamic (kinesthesia) aspects of movement

hormonal control of bone formation and metabolism

pth calctionin vit D

hyaline cartilage

pure cartilage ~ pearly white ~ opaque type of cartilage seen in the articulate joints of supermarket "soup bones" Most abundant cartilage Forms much of the cartilage of the fetal skeleton In the adult, forms the costal cartilages that join the ribs to the sternum and vertebrae; many of the cartilages of the respiratory tract; the articular cartilages; and the epiphyseal plates Free surfaces of most hyaline cartilage ~ with the exception of articular cartilage ~ are covered by a layer of fibrous connective tissue, the perichondrium

first-order neurons

receptor → CNS

complex regional pain syndrome

regional post-traumatic pain problem affecting one or more limbs

tactile sensation

relays information about touch, pressure & vibration

The nurse observes a new nurse performing the test for Kernig's sign on a client. The new nurse performs the test by providing resistance to flexion of the knees while the client is lying with the hip flexed at a right angle. The nurse should explain to the new nurse that:

resistance should be provided with the knee in a flexed position. The test for Kernig's sign for meningeal irritation is performed by providing resistance to flexion of the knees while the client is lying with the hip flexed at a right angle.

Transient ischemic attacks (TIAs) are characterized by ischemic cerebral neurologic deficits that:

resolve within one hour of onset 5 min and then clot passes...not quite a stroke...precursors to a stroke

pain receptors

respond to 'freezing cold', < 5°C or 'burning hot', > 45°C

primary cartilaginous joint

seen in early development, growth plate

dermatome

segmental region of the body supplied by a single dorsal root ganglion

proprioception

sense of joint/limb position without visual cues

neuralgia

severe, brief, often repetitive attacks of lightning-like or throbbing pain

cluster (headache)

severe, unilateral, supraorbital, or temporal pain lasting 15-180 minutes

cluster headache

severe, unilateral, supraorbital, or temporal pain lasting 15-180 minutes

ball and socket joint

shoulder and hip

A patient's recent computed tomography (CT) scan has revealed the presence of hydrocephalus. Which of the following treatment measures is most likely to resolve this health problem? ___

shunt

Ruffini end organs

signal continuous states of deformation

unmyelinated

slower onset, longer duration

Lower motor neuron damage..where? What?

spinal cord paralysis...flaccid ( atrophy)

paresthesias

spontaneous, unpleasant sensations

paresthesias

spontaneous, unpleasant sensations seen with severe irritation

In women, pelvic floor weakness and loss of the posterior proximal urethrovesical (PVU) junction cause which type of incontinence?

stress

Secondary cartilaginous joint

strong slightly moveable held by fibrocartilage

arthrology

study of joints

lumbar ___ bladder

sympathetic

spastic bladder- ____broken so ____ in charge...makes you ___ and have___

sympathetic parasympathetic relax incontinence

hyperpathia

syndrome where sensory threshold is raised

third-order neurons

thalamus → somatosensory cortex (parietal lobe, postcentral gyrus)

An elderly female patient has been hospitalized for the treatment of acute pyelonephritis. Which of the following characteristics of the patient is most likely implicated in the etiology of her current health problem?

the patient recently had a urinary tract infection

pain

theories include: specificity theory, pattern theory, gate control theory

nociceptive

these action potentials are transmitted through myelinated Aδ & unmyelinated C fibers

The most common cause of ischemic stroke is: a) vasospasms. b) cryptogenesis. c) arterial vasculitis. d) thrombosis.

thrombosis. Ischemic stroke includes those caused by large artery thrombosis (20%), small artery thrombosis (25%), cardiogenic embolism (20%) and cryptogenic (undetermined cause)(30%), making thrombosis the most common cause (45%).

Emphysema is characterized by hyperinflation of the lungs that produces increased a. total lung capacity b. alveolar wall thickness c. alpa1-antitrypsin enzyme d. submucosal gland hypertrophy

total lung capacity

slower conducting (paleospinothalamic tract)

transmits diffuse, dull, aching, & unpleasant sensations to brainstem

fast-conducting (neospinothalamic tract)

transmits sharp-fast pain to thalamus

growth plate is affected by

trauma nutritional and metabolic changes

cluster headache

uncommon

trigeminal neuralgia

unilateral sensory distribution of one or more branches of CN V

hyperesthesia

unpleasant hypersensitivity

pain

unpleasant sensory & emotional experience associated with actual or potential tissue damage

A patient is devastated to receive a diagnosis of amyotrophic lateral sclerosis (ALS). The symptomatology of this disease is a result of its effects on:

upper and lower motor neurons

A patient is devastated to receive a diagnosis of amyotrophic lateral sclerosis (ALS). The symptomatology of this disease is a result of its effects on: upper motor neurons. the vestibulocerebellar system. upper and lower motor neurons. neuromuscular junctions.

upper and lower motor neurons.

Although urinary obstruction and urinary incontinence have almost opposite effects on urination, they both can result from

urethral structure changes

The most damaging effects of urinary obstruction are the result of _____ and unrelieved obstruction of urine outflow.

urinary stasis

dorsal column (medial lemniscal pathway)

used for transmission of discriminative touch & proprioception

Unlike disorders of the motor cortex and corticospinal (pyramidal) tract, lesions of the basal ganglia disrupt movement: without causing paralysis. posture and muscle tone. and cortical responses. of upper motor neurons.

without causing paralysis.

bundle bone

woven bone

short bones

wrists and ankles

Another name for dry skin is:

xerosis

A client presents to the orthopedic clinic for evaluation since the primary care provider thinks the client may have rheumatoid arthritis (RA). Which statement by the client correlates with the diagnosis of RA? Select all that apply.

• "I'm having a hard time opening doors since it hurts so bad." • "Look, I didn't button all my shirt buttons...it just hurts too much and look at the swelling in my hands." • "Look how my hand is deformed. My doctor calls it 'hyperextension.'"

A client presented to the emergency department after getting "hit in the head with a baseball" while watching his grandson play. An x-ray of the head reveals poor quality of bone. The ED physician suspects the client has Paget disease. Which of the following signs/symptoms helps confirm this diagnosis? Select all that apply.

• "I've had a lot of headaches lately." • "Every now and then, I get a ringing in my ears." • "I have gotten dizzy and had to sit down while shopping."

When a nurse is assessing a patient with osteoarthritis, which of the following does the nurse note are consistent with this disorder? Select all that apply.

• Aching diffuse pain in the involved area • Grating sensation palpated upon movement • Presence of Heberden nodes

In what anatomical areas are irregularly shaped bones located? Select all that apply.

• Ankles • Wrists

A patient who has had ankylosing spondylitis for several years is admitted to the hospital. Which of the following interventions will best assist this patient while in the hospital? (Select all that apply.)

• Assist the client to ambulate in the morning. • Medicate the client for muscle spasms.

An 85-year-old patient with a medical history of diabetes, thrombocytopenia, and hypertension is on the cardiac step-down unit following the development of atrial defibrillation with a heart rate of 120-140. The atrial fibrillation was successfully cardioverted to a normal sinus rhythm and the patient was preparing to be discharged from the hospital when suddenly the patient developed right-sided hemiplegia and dysphasia. The nurse understands that this patient has many modifiable risk factors for stroke including which of the following? Select all that apply.

• Atrial fibrillation • Diabetes • Hypertension

When trying to explain the difference between vasogenic versus cytotoxic cerebral edema, the physiology instructor mentions that cytotoxic edema displays which of the following functions in the brain? Select all that apply.

• Causes ischemia to build up lactic acid due to anaerobic metabolism • Allows cells to increase volume to the point of rupture, damaging neighboring cells

Hydrocephalus is caused by which of the following? Select all that apply.

• Decreased absorption of cerebral spinal fluid • Obstruction of cerebral spinal fluid Hydrocephalus is defined as an abnormal increase in cerebral spinal fluid volume. The two causes include decreased absorption and obstruction of cerebral spinal fluid.

When trying to explain the difference between vasogenic versus cytotoxic cerebral edema, the physiology instructor mentions that cytotoxic edema displays which of the following functions in the brain? Select all that apply. a) May cause herniation by displacing a cerebral hemisphere b) Impaired blood-brain barrier that allows water/proteins to leave vessels and go into the interstitial space c) Allows cells to increase volume to the point of rupture, damaging neighboring cells d) Mainly allows edema to form in the white mater of the brain e) Causes ischemia to build up lactic acid due to anaerobic metabolism

• Causes ischemia to build up lactic acid due to anaerobic metabolism • Allows cells to increase volume to the point of rupture, damaging neighboring cells Cytotoxic edema involves an increase in intracellular fluid. Ischemia results in the inadequate removal of anaerobic metabolic end products such as lactic acid, producing extracellular acidosis. If blood flow is reduced to low levels for extended periods or to extremely low levels for a few minutes, cellular edema can cause the cell membrane to rupture, allowing the escape of intracellular contents into the surrounding extracellular fluid. This leads to damage of neighboring cells. Vasogenic edema occurs with conditions that impair the function of the blood-brain barrier and allow transfer of water and proteins from the vascular into the interstitial space. It occurs primarily in the white matter of the brain, possibly because the white matter is more compliant than the gray matter. Vasogenic edema can result in displacement of a cerebral hemisphere and various types of brain herniation.

A patient has an abrupt onset of mental slowing and depression. Which of the following conditions in the patient's history would indicate vascular dementia as a cause of these changes? Select all that apply. a) Cigarette smoking b) Peptic ulcer c) Diabetes insipidus d) Cardiac dysrhythmias e) Cerebrovascular accident

• Cerebrovascular accident • Cardiac dysrhythmias • Cigarette smoking The hallmarks of vascular dementia are mental slowing and depression. They usually occur as a result of brain injury from hemorrhage or occlusion. Common disorders associated with this diagnosis are cerebrovascular accident, cardiac dysrhythmias, cigarette smoking, hypertension, hyperlipidemia, diabetes mellitus, and autoimmune disorders.

The nurse assessing a patient with a traumatic brain injury assesses for changes in which of the following? Select all that apply. a) Sensory function b) Level of consciousness c) Metabolic function d) Motor function e) Cognition

• Cognition • Level of consciousness • Motor function • Sensory function Brain injuries can cause changes in level of consciousness and alterations in cognition, motor, and sensory function; therefore, the nurse assessing a patient with a traumatic brain injury should assess for changes in these areas.

Osteoarthritis is the most common cause of arthritis and a significant cause of disability in the elderly. What joint changes occur in osteoarthritis? (Select all that apply.)

• Creation of spurs • Loss of articular cartilage • Synovitis

The nurse is assessing a patient for osteoarthritis (OS) of the knee. Which should the nurse consider to be expected findings? Select all that apply.

• Crepitus • Limitation of motion • Quadricep atrophy • Localized pain

When teaching a community education class on osteoarthritis (OA), the nurse should include which of the following in discussing which individuals are at risk for this problem? Select all that apply.

• Elderly individuals • Older women • Obese people

The nurse is assessing a patient with rheumatoid arthritis. Which of the following would the nurse expect to find? Select all that apply.

• Erythema • Swelling • Warmth • Tenderness

Which symptoms are required for a diagnosis of psoriatic arthritis? Select all that apply.

• Evidence of nail changes • Characteristic skin changes are evident

Which patient reports would support a diagnosis of a torn ligament? Select all that apply.

• Extreme pain • Swelling

The nurse is caring for a patient with rheumatoid arthritis (RA). Which of the following assessment findings does the nurse expect? (Select all that apply.)

• Fatigue • Anorexia • Weight loss

A rheumatology nurse is teaching a patient about extra-articular signs and symptoms of rheumatoid arthritis (RA). Which of the following does the nurse include in the teaching session? Select all that apply.

• Fatigue • Loss of appetite • Aching • Stiffness

The nurse is caring for a patient with rheumatoid arthritis (RA). Which of the following assessment findings does the nurse expect? (Select all that apply.)

• Fatigue • Weight loss • Anorexia

There are several types of brain injuries that can occur. What are the primary (or direct) brain injuries? (Select all that apply.)

• Focal lesions of laceration • Contusion • Diffuse axonal • Hemorrhage

During pathophysiology class, a nursing student asks the faculty what purpose do the osteoprogenitor cells serve. From the following list, identify the purpose of these cells. Select all that apply.

• Growth and remodeling of bone. • Repair of bone.

Which of the following would be considered physiologic characteristics of immature or woven bone? It: Select all that apply.

• Has low tensile strength • Serves as temporary scaffolding for support • Is found in parts of a healing fracture

When teaching a patient about areas of the body typically affected by gout, the nurse should include which of the following areas? Select all that apply.

• Heel • Metatarsals • Instep

The nurse contacts the physician regarding a client's early signs of diminishing level of consciousness based on which of the following? Select all that apply. a) Stupor b) Inattention c) Disorientation d) Blunted responsiveness

• Inattention • Disorientation • Blunted responsiveness The earliest signs of diminution in level of consciousness are inattention, mild confusion, disorientation, and blunted responsiveness.

A basketball player fell awkwardly when attempting to claim a rebound, a mishap that resulted in a tear to the anterior cruciate ligament (ACL) of his left knee. The school nurses will anticipate the player to exhibit which of the following clinical manifestations? Select all that apply.

• Intense pain • Edema (swelling) of the knee

A brain tumor causing clinical manifestations of headache, nausea, projectile vomiting and mental changes is likely located in which part of the brain? Select all that apply. a) Temporal lobe. b) Frontal lobe. c) Intra-axially. d) Brain stem. e) Extra-axially.

• Intra-axially. • Extra-axially. • Frontal lobe. Tumors within the intracranial (intra-axially) cavity is fixed and cause s/s of increased ICP like headache, n/vomiting, mental changes, papilledema, visual disturbances, alterations in sensory and motor function... Outside the brain tissue (extra-axially) but within the cranium, tumors may reach large sizes without producing s/s. After they reach a sufficient size, s/s of increased ICP appear. -Temporal lobe tumors often produce seizures as their first symptom. -Brain stem tumors commonly produce upper/lower motor neuron s/s such as weakness of facial muscles and ocular palsies. -Frontal lobe tumors also grow to a large size and cause s/s of increased ICP.

An older adult is being evaluated for dementia. Which of the following assessments do not indicate normal aging? Select all that apply.

• Is easily agitated when routines are not followed • Is unable to explain the meaning of a proverb • Needs cues to perform hygiene activities Normal aging often includes slower processing of short term memory, and intact long-term memory with recognition cues for recall. Dementia may inhibit cognitive function so the client is unable to perform normal activities such as bathing and dressing. They become agitated, experience difficulty in higher order thinking and problem solving, and may see things that are not there.

What are the criteria categories required for the diagnosis of rheumatoid arthritis (RA)? Select all that apply.

• Joint involvement • Serology • Acute phase reactants • Duration of symptoms

Which symptoms would support the diagnosis of a stroke involving the posterior cerebral artery? Select all that apply.

• Loss of central vision • Repeating of verbal responses Posterior cerebral artery stroke would produce visual defects and the repeat of verbal and motor responses.

Huntington disease is a genetic disorder that does not usually manifest itself until the client is in his or her 40s or 50s. What are the most common early psychological changes that occur with Huntington disease? (Select all that apply.)

• Moodiness • Impulsive behavior • Personality changes Depression and personality changes are the most common early psychological manifestations. Memory loss is often accompanied by impulsive behavior, moodiness, antisocial behavior, and a tendency toward emotional outbursts.

When a nurse is caring for a patient with gout, which of the following medications are used? Select all that apply.

• Nonsteroidal anti-inflammatory drugs (NSAIDs) • Colchicine • Indomethacin

Scleroderma is an autoimmune disease of connective tissue that is characterized by hardening of the skin. What diseases do almost all people with scleroderma develop? (Select all that apply.)

• Polyarthritis • Raynaud phenomenon

Select the correct statements that describe the function of PTH (parathyroid hormone). Select all that apply.

• Prevents serum calcium from decreasing and phosphate from increasing above normal • Maintained by serum levels of ionized calcium • Maintains serum calcium by bone resorption

The nurse is caring for a patient with systemic lupus erythematosus who has developed interstitial nephritis. Which of the following assessment findings is consistent with lupus nephritis? Select all that apply.

• Proteinuria • Periorbital edema

Following a car accident of a male teenage that did not have their seatbelt on; he arrived in the emergency department with a traumatic brain injury. He has severe cerebral edema following emergent craniotomy. Throughout the night, the nurse has been monitoring and reporting changes in his assessment. Which of the following assessments corresponds to a supratentorial herniation that has progressed to include midbrain involvement? Select all that apply.

• Pupils fixed at ~ 5 mm in diameter. • Respiration rate of 40 breaths/min. • Decerebrate posturing following painful stimulation of the sternum. With midbrain involvement, pupils are fixed and midsize (5 mm in diameter), and reflex adduction of the eyes is impaired; pain elicits decerebrate posturing; and respirations change from Cheyne-Stokes to neurogenic hyperventilation.

When assessing a patient with gout, for which of these signs and symptoms does the nurse assess? Select all that apply.

• Redness and swelling of the affected joint • Pain and swelling in one joint • Intake of alcohol

Which of the following are potential causes of osteoarthritis? (Select all that apply.)

• Rheumatoid arthritis • Postraumatic disorders • Metabolic disorders • Collagen disorders

The nurse assessing a patient with a traumatic brain injury assesses for changes in which of the following? Select all that apply.

• Sensory function • Motor function • Cognition • Level of consciousness Brain injuries can cause changes in level of consiousness and alterations in cognition, motor, and sensory function; therefore, the nurse assessing a patient with a traumatic brain injury should assess for changes in these areas.

A nurse is caring for a patient with rheumatoid arthritis (RA). For which of these extra-articular manifestations does the nurse assess? Select all that apply.

• Splenomegaly • Lymphadenopathy • Elevated erythrocyte sedimentation rate (ESR)

A physician is attempting a differential diagnosis of a 30 year-old female who is suspected of having systemic lupus erythematosus (SLE). Which of the following aspects of the physician's assessment and the client's history would be considered potentially indicative of SLE? Select all that apply.

• The client has a "butterfly rash" on her nose and cheeks. • She complains of intermittent joint pain. • The client has been hospitalized twice in the past for pleural effusions. • Blood work indicates low red cells, white cells and platelets.

The nurse is providing discharge instructions on the antiseizure medication valproic acid to a woman of childbearing age. Which of the following should the nurse include? Select all that apply. a) The medication may interact with oral contraceptives. b) The medication should be stopped in the third trimester of pregnancy c) The medication may predispose her to osteoporosis. d) The medication should be taken consistently. e) The client should also take a folic acid supplement daily.

• The client should also take a folic acid supplement daily. • The medication may predispose her to osteoporosis. • The medication should be taken consistently. • The medication may interact with oral contraceptives. The nurse should include in the teaching that the client should also take folic acid supplementation daily, that the medication may predispose the client to osteoporosis, and that it may interact with oral contraceptives. The nurse should also caution the client to take the medication on a consistent basis to prevent recurring seizures.

Which statements are true regarding the skeletal system? Select all that apply.

• There is a type of skeleton referred to as appendicular. • The skeletal system includes both bone and cartilage tissue. • Cartilage is prevalent in the prenatal fetal skeleton. • The skull is considered a part of the axial skeleton.

A client newly diagnosed with rheumatoid arthritis is crying and tells the nurse she does not know how to manage the disease. Select the most important information for the nurse to provide. Select all that apply.

• Treatment goals • Education • Posture • Physical rest • Therapeutic use of heat and cold

Which substances are not involved in preventing the release of calcium from the bone? Select all that apply.

• Vitamin D • Phosphate

Which of the following intervention should the nurse recommend to best minimize the risk of nutritional rickets in an infant whose culture supports long-term breast feeding without the introduction of foods? Select all that apply.

• Vitamin D supplements • Regular exposure to sunlight

The nurse is caring for a client who has received tissue plasminogen activator (tPA). The nurse's plan of care should include education relating to which of the following? Select all that apply.

• Warfarin (Coumadin) therapy • Reduction of risk factors • Signs and symptoms of occurrence

The nurse is caring for a client who has received tissue plasminogen activator (tPA). The nurse's plan of care should include education relating to which of the following? Select all that apply. a) Warfarin (Coumadin) therapy b) Aspirin (ASA) therapy c) Reduction of risk factors d) Signs and symptoms of occurrence

• Warfarin (Coumadin) therapy • Reduction of risk factors • Signs and symptoms of occurrence Clients receive tPA as treatment for ischemic stroke. In her plan of care the nurse should include education related to: warfarin (Coumadin therapy), signs and symptoms of stroke recurrence (most likely in first week post-tPA administration) and ways for the client to reduce risk factors for another stroke.

Papulosquamous dermatoses, such as psoriasis, are a group of skin disorders characterized by: A. scaling papules. B. granular scabbing. C. raised red borders. D. nodular ulcerations.

A

The main cause of decubitus ulcers (bedsores) is ______ that impairs the flow of blood & lymph. A. external pressure B. shearing forces C. tissue edema D. thrombosis

A

What neoplasm of the skin is associated with the poorest prognosis? A. Malignant melanoma B. Basal cell carcinoma C. Intraepidermal squamous cell carcinoma D. Invasive squamous cell carcinoma

A

Which of the following disorders of the skin is most likely to respond to treatment with systemic antibiotics? A. Acne vulgaris B. Urticaria C. Atopic dermatitis D. Verrucae

A

A 44-year-old man has been brought to the emergency department by emergency medical services with severe electrical burns resulting from a workplace accident. The most immediate threat to this patient's survival at this time is: A. infection. B. hemodynamic instability. C. acute pain. D. decreased protein synthesis & impaired healing.

B

A client visits the OB/GYN clinic complaining of low abdominal pain, purulent cervical discharge, and painful intercourse. The health care worker (HCW) diagnoses pelvic inflammatory disease. The HCW educates the client about that fact that this disease may be associated with:

D. STI polymicrobial infection

A client comes to the college campus nurse complaining of unilateral pain, swelling, and redness on his scrotal area. The nurse knows these clinical manifestations are likely caused by:

D. Epididymitis

When talking to a group of teenagers about ways to protect against skin damage from ultraviolet radiation, the nurse should discuss which of the following tissues/cells?

D. Melanocytes

Endometriosis is characterized by painful hemorrhagic lesions in the pelvis, which may develop into which of the following potential complications?

D. Pelvic adhesions

Psoriasis is characterized by _____ with an overlying silvery-white scale.

red plaques

The direction of surgical incisions is often determined by the organizational pattern in this cutaneous layer.

reticular dermis

Dysplastic nevi are precursors of malignant melanoma that are: A. larger than other nevi. B. oval epidermal nests. C. dermal cords of cells. D. brown rounded papules.

A

Manifestations of superficial dermatophytosis of the skin include: A. scaling. B. erythema. C. vesicles. D. infiltration.

A

Dry, itchy plaques on her elbows & knees prompted a 23-year-old woman to seek care. The clinician has subsequently diagnosed the patient with psoriasis, a disorder that results from: A. increased epidermal cell turnover. B. an IgE-mediated immune reaction. C. hormonal influences on sebaceous gland activity. D. human papillomaviruses (HPV).

A

Which of the following clients is at greatest risk for developing balanitis xerotica obliterans?

A male who has an uncircumcised penis

Which of the following statements about temperature regulation and skin is accurate?

Arteriovenous anastomoses between an artery and a vein within the skin layer are important for temperature regulation.

A 79-year-old patient has been confined to bed after a severe hemorrhagic stroke that has caused hemiplegia. Which of the following measures should his care team prioritize int eh prevention of pressure ulcers? A. Prophylactic antibiotics B Repositioning the patient on a scheduled basis C. Applying protective dressings to vulnerable areas D. Parenteral nutrition

B

A yeast-like Candida albicans fungal infection can be differentiated from a tinera fungal infection by the presence of: A. circular patches. B. satellite lesions. C. fungal spores. D. raised borders.

B

Warts develop when _____ are invaded by human papillomavirus (HPV). A. melanocytes B. keratinocytes C. sebaceous glands D. subcutaneous cells

B

Which of the following skin disorders seen in elderly persons is considered a premalignant lesion? A. Cherry angiomas B. Actinic keratosis C. Solar lentigines D. Telangiectases

B

A teenager reports ugly warts that have invaded her hands. She wants them gone before prom season. The nurse will likely be explaining which of the following treatment measures to this teenager?

B. Applying a keratolytic agent like salicylic acid

While breaking in a new pair of shoes, a client develops a large (1.0 cm) blister filled with clear fluid. The dermatologist diagnoses this as:

B. Bullae

A client comes to the OB/GYN clinic complaining of difficulty in emptying the bladder and frequency and urgency of urination. After examination, the health care provider will likely diagnose:

B. Cystocele

A man's winter vacation to a tropical destination has been accompanied by repeated sunburns. What process accounts for the damaging effects of the sun's radiation? A. Initiation of a autoimmune response B. Compensatory increases in melanin production. C. Damage to epidermal cell DNA & free radical production. D. Hyperkeratinization & the formation of microscopic, subcutaneous lesions.

C

Allergic & hypersensitivity dermatoses are characterized by: A. target lesions. B. dark erythema. C. epidermal edema. D. silver-white scale.

C

Onychomycosis initially causes the toenail to appear ____ as a fungus digest the nail keratin. A. black B. cracked C. opaque D. eroded

C

Which of the following actions involves the greatest risk of skin shearing? A. Inserting a peripheral intravenous catheter B. Rolling the patient from a supine to side-lying position C. Pulling the patient up in bed D. Helping the patient ambulate after surgery

C

A 41-year-old male client is planning on having a vasectomy. While explaining this surgery to the client, the health care worker will include which physiologic principle as the basis for this contraception technique?

C. "Sperm can no longer pass through the ductus deferens."

Papulosquamous dermatoses, such as psoriasis, are a group of skin disorders characterized by:

C. Scaling papules

A nurse is conducting a healthy living workshop for a group of women in their 20s. Which of the following screening recommendations should the nurse provide to the participants?

Clinical breast examination every 3 years until age 40

A patient has been admitted to the intensive care unit of the hospital after developing toxic epidermal necrolysis (TEN) consequent to the administration of the sulfonamide antibiotic. What pathophysiologic phenomenon is likely the greatest immediate threat to this patient's health? A. The development of bacterial cellulitis on compromised skin surfaces. B. Fluid & electrolyte imbalances resulting from the loss of skin integrity. C. A cascading autoimmune response that may result in shock. D. The presence of diffuse lesions & skin sloughing on the patient's mucous membranes.

D

Which of the following clients would be predisposed to developing a yeast-like Candida albicans fungal infection?

D. An immunosuppressed cancer client with maculopapular satellite lesions

Which of the following statements about screening for prostate cancer is most accurate?

D. BPH and prostatitis can confound prostate screening results.

Verrucae or warts are common benign papilloma caused by:

DNA-containing HPV

_____ is the metabolic bone disorder characterized by a failure or delay in calcification of the cartilaginous growth plate in children.

Rickets

The most common bacteria to cause pyogenic osteomyelitis is:

S. aureus

In synovial joints, the articulating ends of the bones are indirectly linked by:

a strong fibrous capsule

A highly pruritic chronic inflammatory skin disease that usually begins early in life is called:

atopic dermatitis

Circumscribed elevations of the skin caused by fluid under or within the epidermis are:

blisters

Osteosarcoma is aggressive and highly malignant with early metastasis seen in the:

bone

The hormone _____ lowers blood calcium levels and decreases bone resorption.

calcitonin

_____ possess antigen-presenting functions and play an important role in the immunobiology of the dermis.

dermal dendrocytes

Rheumatoid arthritis is a systemic inflammatory disease with joint manifestations described as:

fatigue weakness

The composition of ____-cartilage allows it to function as shock absorber.

fibro

In a strain or sprain, repair is accomplished by _____ from the inner tendon sheath.

fibroblasts

The high degree of hydration and the movement of the water in the matrix, allows _____-cartilage to respond to varying pressure loads.

hyaline

Bone is connective tissue that is impregnated with _____ so it has great tensile and compressive strength without excessive weight.

inorganic calcium salts

Osteomalacia is caused by _____ without loss of bone matrix.

insufficient calcium

Shortly after a closed fracture has occurred, early manifestations include _____ or "local shock".

numbness/flaccid muscles

With the exception of the _____, each type of bone cell originates from the same basic cell type.

osteoclast

Aseptic destruction of a bone segment due to interrupted blood flow is:

osteonecrosis

The _____ layer of the dermis is richly vascularized.

papillary


Kaugnay na mga set ng pag-aaral

Markets and Social Security Chapter 6

View Set

Chapter 6 Medication Order Entry and Fill Process

View Set

Ch. 8 Female Reproductive System

View Set